Sunteți pe pagina 1din 242

TEXT FLY WITHIN

THE BOOK ONLY

<>
W

c^
DO
^

u<OU_1 64808

>[g
""

CD

^D

CHAPTER I
i.

Definitions.

Scalar

and Vector Quantities.

The mathematicians and

physicists

have got to deal

with two different kinds of quantities. Some of them are


specified by a single real number called the magnitude
or the measure of that quantity and are not related to any
Such quantities aie Scalars. The
direction in space.

examples of such quantities are mass,


charge, temperature, density etc. etc.

body may be

specified

ratio of the given

Thus

mass

for scalars

ment and a

real

by a

positive

volume, electric
Thus the mass of a

number m equal

to the

to the unit mass.

we

should have a unit of

number m that expresses the

measure-

ratio of

the

given scalar quantity to that of the unit.

There are however other types

of

quantities

which

have got magnitude as well as a definite direction in


space. Such type of quantities are called vector quantities
or simply vectors.
The most familiar examples of this
are
type
velocity, acceleration, force,
displacement etc.
If we say that the speed of a train is 45 m. p. h., our

statement is not complete so long as we do not specify


the direction in which the train is moving. Similatly we
cannot content ourselves by simply giving the magnitude

we have got to specify the particular direction


which it acts. Thus a scalar quantity cannot completely

of a force;
in

specify

a vector quantity.

Vector Analysis

Representation and Notation qf Vectors*


Symbolically a vector is often denoted by two letters
with an arrow over them; the tail of the arrow is called
the origin of the vector whereas its head is called the

terminus
origin
is

or the end.

Thus

and B the terminus.

given by the length

Such

in the vector

AB A

The magnitude

AB and

direction

its

is

is

of

called the

the vector

from A

to

J5.

vectors are called line

Thus a

vectors*

vector

be

may

represented by a directed line


segment i.e. a given portion of

a given line on which

two

the

Fig.

1.

end points origin and the terminus are specified, i.e. they
cannot be interchanged, for it will change the direction
of the? vector.

In addition to the above notation of vectors by giving


their origin and
terminus ws shall use sigle letters

(Clarendon

letters)

in

The corresponding

heavy (bold
letters

italic

face)

a, b, c

type like a, b, c*
denote the magni-

tude of the vectors.

Thus
by

if

^B=a,

then

AB
|

i.e.
\

its

magnitude

is

denoted

a.

The above

right so far as the

printing of
the books is concerned but it is quite inconvenient for the
teachers and the student to use the above notation on the

notation

is all

black-boards and their note-books respectively. Alternatively


we may adopt the Greek letters a, /3, y, 8 to denote the
vectors and the letters
respectively.

d to denote their magnitudes


have to deal with large number of

a, b, c 9

When we

Introduction
vectors

we may

use the capital letters A, B, C, D,


vectors and the corresponding small letters a, b, c,

for their
i.e.

placing bars on

magnitudes or by

by V,

IT,

letters for vectors

notations

of

F. .for

d, *,/...
the letters

however, we shall

In this book,

etc.

follow the standard

bold-face

using

and the corresponding

italic

typed

letters

for

their magnitudes.
**

Free and Localised Vectors.

When we

are at

any point, then

when

restricted to

vector

it is

said to be

the origin of the


a free vector, but

a certain specified point, then the


said to be a localized vector.

it is

is

to choose

liberty

vector, at

Equal Vectors. Two vectors are said to be equal when


they have the same length (magnitude) and the same
direction and the equality of two vectors is written as
usual a=b. Thus equal vectors may be represented by
parallel lines of equal length drawn in the same sense of
direction irrespective of the origin.

Like Vectors.

Vectors are said to be

like

when they

have the same sense of direction.

Collinear Vectors. Any number of vectors are said to


when they are parallel to the same line what-

be collinear

ever their magnitudes

may

Any number

Coplanar Vectors.
to be coplanar

when they

Co-initial Vectors.
it

about parallel to

a=AB may

be.

are parallel to the

itself

AB

same

are said
plane.

not altered by shifting


Hence any vector
space.

vector
in

be drawn from any

moving the segment

of vectors

parallel to

is

assigned origin
by
so that the point

itself

Vector Analysis

and the point

coincides with

say

P;

falls

on some other point

then AB=*OP*=a.

In this

way

all

vectors in

be replaced by

may

space

drawn from the same assigned origin by moving


them parallel to themselves till their origin coincides with
All such vectors which have
the same assigned orgm 0.
the same point as the origin are called co-initial
vectors

vectors*

Zero Vector.
vector

If

then

coincide,

the origin and terminal points of a


it
is said
to be a zero vector.

Evidently its length is zero and its direction is indetermiA zero vector is denoted by the bold face typed o.

nate.

All zero vectors

->

AA,

BB

are equal and they can

etc.

Unit Vector.
its

be expressed as

vector

is

said to be

magnitude be of unit length.

a whose module

is

a,

a unit vector

if

there be

any vector
then the corresponding unit vector
If

A
in that direction is

denoted by a which

unity.

Thus we have a = ia

Note.

Any two unit


we do

be equal so long as
are also the same.

a=

or

vectors

may

has

its

magnitude

a
.

not be confused to

not ascertain that their directions

Thus only two

like unit vectors are

A A
equal. Again

if

we

say that

a=b,

it

would mean that

a=b

and that afc, the first relation specifying same direction


and the second one specifying equal magnitude.
Position vector.

The

position vector of

any point P

with reference to an origia

is

the vector OP.

Thus

taking

Introduction

we can find the position vector of every point


Conversely, corresponding to any given vector
->
a point P such that 0P=r.

as origin
in space.

r there

is

whose direction is the


a given vector a but whose magnitude is

Reciprocal vector.

same

as that of

reciprocal of the magnitude

vector

of the

given vector

called

is

the reciprocil of a and

is

vector
reciprocal
r

..

Thus

written as a~*.

a""

t
1

A
A

aaa.

a=

<x.a
-.

a2

a
=--a*

Now since

the magnitude of a unit vector


reciprocal is again a unit, we conclude

whose

is

a unit

that the

reciprocal of a unit vector is the unit vector itself.

Negative vector.

vector whose

magnitude

is

same

the

as that of a given vector a but opposite direction


called the negative vector of a and is written as
a.
If

represented by

OA

is

is

then

-a

is

represented by AO.

Addition of vectors.

a.

Let there be any


two given vectors a and
b.

Now

point
vectors

choose any
and draw the

a and b

so that

the terminus of a coincides with the origin of

Fig. 2

->

->

Then

the vector given

by OB

is

defined as the

sum

of

Vector Analysis

the given vectors

a and b and

The above sum


position of 0.

is

written as

independent of the choice for the


should be clearly understood here that

It

is

->
the magnitude of

of

OB

is

OA

AB

and

as

we know

the

of

magnitudes

that any side of a triangle

than the sum of the other two.

OB

sum

not equal to

will only

be equal to the
>

OA and OB when OA and OB have


vector

OB
-

vector

the

same

represents the combined effect


-

OA and OB.

is

less

The magnitude of vector


sum of the magnitudes of

The above law

is

direction.

(i.e.

called

The

resultant) of

triangle

law

of addition by which the vector quantities are compounded,


If

a-+b~0

showing that

a but whose

i.e.

-a

is

when

(M=a

direction

and AB=*b,

OBs=a+b.

Now

coincide then

b=-b+a

a vector which has the same


is

Vector addition
If

and

length as

opposite to that of a.

is

Commuative,

then

complete

parallelogram whose
two sides are OA and AB.

the

Since the opposite sides of


a parallelogram are equal
and parallel, we can say that

and AB**OC**b.

i.e.

a-fb=*b+a.

Introduction

a-fb=b+a. ^

or

Vector addition

is associative

(a+b)+c=:a+(b+c):

e.

i.

OA

Let

and

OA^c.
Join

to the

terminus of

last vector C.

-*

Similarly

is

Thus we see that (a+b)+c=OC-a+(b4-c) where OC


sum of the three given vectors and is written as

the

a+b+c.
Thus

the

sum of any number of

order in which they are added

vectors is

independent of the

and of their grouping

to

form partial

sums.

Sum of any number


sum

of

a, b, c, d,

line

any

number

of

vectors

e say, then form a broken

whose segments

direction

of vectors.

represent

In the above figure

in length and
these vectors.
-

OA= a, AB=b,

>

BCssc,

etc.;

the origin of

then the vector


first

vector to

joining
terhiinal

If

we

are to find the

Vector Analysis
the last vector will represent the

of

point

a+b+c+d+e

vactor

sum

OA+AB+BC+CD+DE=OE.

i. e.

The above sum

will

be zero

if

the terminal point

the last vector coincides with the origin of the

first

of

vector.

04+45+ J5C+CZ)+Z)0=>0.
From above we

observe that

04+44=04
Now 44=o

and

or

00=o

00+04=04.

and

we put 04=a,

if

then

a+o=a and o + a=a.


EX.I,

7/a^b,

c,

represent

the

show that necessary and


quadrilateral be a parallelogram is that

quadrilateral,
the

sides

of a

sufficient

condition

that

a+c

and that this

consecutive

b+d=0.

implies

Since the^ origin of the first


vector coincides with the end point
of the last vector,

we have

AB+BC+CD+DA^Q
a+b+c+d=0.

or

..

(1)

the figure be a parallelogram,


then 4J3=CDand they are parallel.
If

/.

CD

4Bs=

as they are in opposite

a+c=0. Hence from


giving that BC and DA are
i. e.

figure
it

(i),

is

easily.

parallelogram.

we

get

directions

b+d=0.

:.

or

a= -c

C= - DA

equal and parallel. Hence the


can prove the converse of

We

Introduction

Subtraction of vectors ,

3,

We

have already defined negative vectors as a vector


the
same magnitude as that of a given vector but
having
opposite direction.

Thus

if

AB~b,

then AC=*

b,

where

AC=AB.
The
traction of

b may
of

operation of vector subtwo given vectors a and

be regarded as the operation

addition

vecctor

of

a and

-b

and written as

a-b=-a+(-b).
Similarly,

b-a=b + (-a).
Fig.No.7

Expression of a vector in terms of the position


vectors of its end points.
Let us choose any point
as origin and the position
vectors of

extremities

the

A and B

of a vector

respect

to

this

AB

with

origin be

and b; then

Similarly

BA~BO+OA~OA+(-OB)

Vector Analysis

10

Ex.

2.

//"a

and

of a regular hexagon

FA, AC, AD)

AE

are the

ABCDEF,

and

CE

vectors

express

in terms of

forming consecutive sides


the vectors

a and

CD, DE, EF,

b.

(Utkal B. Sc. Hons. 53)

a)
(2)

~*

~*

because
twice

AD

its

parallel to

is

BC

and

length.

Fig.No.8
or

(a+b)+CD=2b.

[by

(1)

&

(2)]

.(3)

.*.

F-4

is

equal to CD.bat in opposite direction,

(4)

CD f DE^b - a - a

2b-a.
Ex. 3.

The position

vectors

=b - 2a,

[by

(3)

and

(4)]

[by

(2)

and

(4)]

are

of Jour points A, B, C,

Introduction

2a+3b, a- ab

a, b,

11

Express

respectively.

the vectors

--DB BC

AC,

0m/

C4

i/z

Let

tarww

0/a

0/zd b.

be the origin so that

OA =a, OJSb, OC=2a+3b

->

and OZ)=a-2b.

2 (a+b),

-(a+3b).

Ex.

->>>->
AF

Five forces AB, AC,

4.

AD, AE,

act at the vertex

->
of a regular hexagon

where

is the

ABCDEF.

Prove that the resultant

is

6AO

centroid of the hexagon.

-----

Refer figure Ex.

2.

If

be the resultant, then

R=AB+AC+AD+AE+AF

-->->--->-->

=*AB+(AD+DC)+AD+(AD+DE)+AF.
-

Now AF
directions

AB

and D-E cancel each other.

/.
i.

e.

and DC are two equal vectors in opposite


and hence they cancel each other. Similarly

R*=3AD**6.AO where

the centroid of the hexagon.

is

the

mid. point of

AD

Vector Analysis

Multiplication of vectors by scalars.

4.
If

OA=SL and m

vector m.

OA

ma. is

of the given vector

is

real

any positive

defined to be a

number then the

vector in the direction

but whose module

is

times the module

of the given vector.

manner the vector -m..OA=-ma is a


the direction opposite to that of a and having

In a similar
vector

in

module m times

The

that of a.
of a

division

vector

by a

real

number m may be

of that vector by
considered as the product
r
J

Multiplication of vectors by scalars is


tive,, associative

and

distributive,

w.a
m (na)s=(w) a
(m+ri)

commuta-

e.

a.m,
(wa),

a~;

m(a+1
where m and

are

any scalar numbers and a and b are

any two vectors.

The
we

first

three results follow

Let
so that

be

CM=a, AB~b

OB=a+b.
+ive

then choose

on

from the definition and

are going to prove here the last result.

A'

If

number,
and B

OA and OB produced

respectively,

so

that

OA'=*m.OA and OB'^m.OB.

13

Introduction

.'.

Ofl'=i.(a+b)

04'o.ro.a,

Since A'B'

is

parallel to

AB

or

.'.

4'5'=7n

^4^.

A'B'^m b

.(2)

Now

OA'+A'B'^OB'

OA+m AB~m OB
m a+w b-m (a+b).

or

iw $

or

AB,

(1)

[from

(1)

and

(2)]

Note In case
be negative then

we should choose
point

A'

produced

A_

on

^40

(and

not

^ <&+h

OA

produced) such
D
that 0/1' ism times
Fig Ho. 11
OA but in direction opposite to that of 0-4. Tto above
result can be similarly proved by the help of the diagram
given.

Any of two

like vectors

can be expressed as a

multiple of the other.

->

A
CZ)=b=& b
A A
that a=b. .(1)
A
A

Let 4Z?=sa==fl . a and


be two

like vectors so

A
.

_A
-

^
=s

a from

^a

<*<

(1).

Hence proved.

14

Vector Analysis

we can show

Similarly

that

a==~ b

i.

e.

AB**~ CD.

Linear Combination of Vectors.

5.

a vector r can be expressed as

If

where

.are

any scalar numbers, then r


a linear combination of the vectors a, b, c. .
x, y, z.

is

said to be

6.

Components of Vectors.

We

have already defined that

when any number

of

the same line, they are said to be


and when they are parallel to the same plane

vectors are parallel to


collinear

they are said to be coplanar.


Collinear Vector :
(i)
given vector

Let

can be expressed as

Any vector r
xa where x is a

04 =a and OP=r.

"*
Since

OP

with a

scalar.

"*
is

collinear with

OA,

scalar multiple of OA,

i.

e.

it

can be expressed as a

-*

OP=x OA
.

r=*.a.

or

Coplanar vectors
given (non-collinear)
as a

collinear

linear

andy

Any

vectors

vector

and

combination of the given vectors

uniquely

i.

expressed

e.rxa+yb where
(^ aj B.

are scalar s.

Let

r coplanar with any two


can be

Sc. 1960)

04= a and OBb be two


~

non-collinear vectors

and

OP^v

be a vector coplanar with a and

P draw PM and PN
OB
and OA respecparallel
tively and meeting OA in Af and
OB in JV.

Now

through
to

Fig.No.

13-

15

Introduction

OM being collinear
-

ON

with

0^=#a,

->
being collinear with OB=*yb.

Hence proved.

The above combination

is

unique*

In order to prove that the linear combination

...... (1)

r-xa+jb
is

unique,

let

that

us suppose

can be expressed

in

another

form as

r-*'a+/b

....

[from

(1)

and

(2)
(2)]

or

If

/>

be not equal to zero, then a

is

a scalar multiple

of b.

The above form shows that a and b are collinear


which contradicts that a and b are non-collinear. Hence
we must have

pQ or x

Similarly by writing
or

is

coefficients

a we can prove

that <?=()

above

unique.

Therefore if two

same two

b=

Hence we prove that the

y-j'asQ or ^=j'.

combination

#'s=0 or #=*'.

equal

non-collinear

vectors

vectors

are

a and

b,

expressed in

terms of the

the corresponding scalar

are equal.

The above result will not be


vectors a and b be collinear [see note
Note

From

above

we

also

true

in

case the two

after case (Hi).]

observe

that

if there

exists

16

Vector Analysis
of the form

relation

a and

If there -be

two

pa+qbO

p=0, q~0

b, then

(p

between two

several vectors

vectors

non-collinear

non-collinear

vectors

and q being scalars).

rt r2 r 3 ... coplanar with

a and

then

b,

may

they

be

expressed as

If

r be

Above
vectors are

case

r=0

their

sum, then

relation

shows that the components

of a

sum

of

the sums of components of these vectors. In


then each of its components must be zero (as a

and b are non-collinear). Therefore the vector equation


rs=0 te equivalent to the two scalar equation?.

Ex.

5.

Prove that the following lectors are coplanar

3a-7b-4c, 3a-2b+c, a+b+2c,


a, b,

c being any

vectors.

these vectors are coplanar, we should be able to


express one of them as a linear combination of the other
two. Let us suppose that the given vectors are coplanar.
If

/.

.8a-7b-4c*(3a-2b+c)+j>(a+b + 2c), where


Comparing the coefficients of a, b and c,

A:

and y are scalars.


we get 3x+jy=3,

3.
first two, we find that #=2, and ;>
and.y satisfy the third equation as well.
vectors are coplanar.

(iii)

uniquely

Non-coplanar vectors.
expressed

Solving the
These values of x

-Zx+y=*-l, *+-2y=-4.

as

Hence the given

Any

a linear combination

vector

of three

can

be

given (non-

17

Introduction
coplanar) vectors a, b,

i.e.

C where

raa+^b-f

y and z

x,

---*

scalar*.

Let

are

(Pb. 60)

CM=a, OB~b, OC=c

be

any

three

non-coplanar

OP=r.

vectors and let

Fig.NoJ4

The
three

0, OC taken in
Now
C04
and AOB.
BOG,

three lines 0^4,

planes

AOB

OB and OC

cutting OA,

the

to

planes parallel respectively

in

Pdraw
BOC, COA and

through

planes
,

Af and

JV"

thus giving us a parallelepiped whose diagonal

Also

OL

collinear

with 0-4

>

Similarly

respectively

is

OP.

->

->
is

determine

pairs

i.e.

a;

.'.

OL==^a.

>

OM=jb and

-->->
Now r^OP

The above combination

is

unique*

In order to prove that the linear combination

r**xa+.>b+c,
where

...... (1)

c are non-coplanar vectors, is unique,


r can be expressed in another form as
that
suppose
a, b,

r*'a+/b+*'c

let

us

...... (2)

18

Vector Analysis

+j*+*c~*'4/b+<;'c. [from (1) and


(*-*') a+(j>-/)b+te-*')c0
La+Ajfb+JVc0.

.'.

/.

or
If

L be

then

not equal to zero,

a=

j-

b- ^

Above shows that a can be expressed as a


two non-collinear vectors b and c.

and we are given that


there

is

com-

But

it is

b and c should be coplanar [


a, b and c are non-coplanar.

L=0

or

#'s=0,

i.e.

#==#'.

Similarly M=Q

or

^-y==0,

i.e.

y=y'

JV=0

or

^-^=0,

i.e.

Hence we

'three

Hence

z~z'.

prove that the above combination

Therefore if two equal

same

5 P. 14]

So we must have

a contradiction.

and

c.

linear

bination of

essential here that a,

(2)]

vectors

non-coplanar vectors,

the

are

is

unique.

expressed in terms

corresponding

of the

scalar coefficients

are equal.

It should be understood here that the

above

result will not

necessarily be true when a, b and c are coplanar as


case c can be expressed in terms of a and b,

in that

c=/>a+tfb.

i.e.,

b,

/.

r!

rL - ra

*!

and r2 are equal.


/.

since

we must have

a and b are two

vectors supposed non-collinear,

their coefficients each zero.

and

The above

relations do not necessarily

*i*2,

J>i-J>2

and *i-*..

mean

that

19

Introduction

Note

From above we

also observe that

if

a, b,

c are

three non-copalanar vectors and there exists a relation of


the form

a+Afb+JVc=0
Note

the relation

In

c are called the

jb,

M,

JV being scalar)

r=xa+jb+c,

the vectors *a,

(L,

L=0, M=0, JV=0.

then

of the vector r and

components

are called the coordinates of the point


the vectors a, b, c.
there

If

#,

y, z

with reference to

be several vectors rl9 r 2 r3

terms of three non-coplanar vectors


be expressed as

If

a, b, c,

.. expressed
in
then they may

r be their sum, then


xx )

r=(Z
Above

a+(r^) b+(r

^)

c.

shows that the components of a


the sums of the components of those
vectors.
In case r=0, then each of its components should
be zero (as a, b, c are three non-coplanar vectors). Hence

sum

of

the

vector

relation

vectors

are

equation

r=0

is

to

equivalent

three scalar

equations.

2
Ex.

6.

//a,

#!=(),

b5

Z j)'!0, Z ^=0.

be any three

non-zero,

non-coplanar vectors,

Jind the linear relation between the following system of vectors

7a+6c,

a+b+c, 2a-b-fc, a-b~c.

Let 7a+(5c=* (a+b+c)+;> (2a-b+c)+* (a-b-c).

Comparing the

coefficients of a,

Solving these three equations

-!.

d)onaiedby

Mr. N. Sreekanth

b and c we,
we

find

find that

that #

2t

Vector Analysis

20

Hence the required

linear relation

is

7a+6c2(a+b+c)+3(2a-b+c)-(a-b-c).
Linear dependence of vectors.

6.
If

there exists a relation of the type

*a+jb+sc+...0
where

x, y,

(1)

are scalars (not all zero), then the system

z,

of vectors a, b, c.

is

said to be linearly

dependent*

the system of vectors is not linearly dependent, then


said to be linearly independent and in that case
If

it is

*-0,j-O *-0...
f

If

r=*a+j>'b-f-c+...
r-f*a4-rb-|-c.. .=0, then the system of vectors

i.e.

r, a, b, c.

. . .

is

linearly dependent.

Now we know
exist

that between two collinear vectors there

a relation of

the

form r=*a.

three coplanar vectors there exists

raa+jb.

between

Similarly

relation of the

form

Also between any four vectors there exists a

relation of the type

Therefore

rxa+jb+^c.

we can say

that

or three coplanar vectors or any

any two collinear vectors,


more vectors form

four or

a linearly dependent system.

Combining the

results

proved

in

and

6,

we can

say that

The

necessary

and

sufficient

condition

that

two

that

three

vectors

be

linearly independent is that they be collinear.

and
The necessary and

sufficient

condition

vectors be

linearly independent is that they be coplanar.

The Unit Vectors i, j, k*


In
5 case (iii), we expressed any vector r
three non-coplanar vectors a, b and c as r**x
7.

in

terms of

21

Introduction

in this case the directions of the three non-coplanar

Here

vectors will be

and

mutually perpendicular say OX,

OT and 0%

unit vectors in these

denoted

are

directions

by i, j, k respectively.
Such a system of vectors
is

ortho-normal

called

Now we

system.

shall

express any vector r in


terms of the three unit
vectors

i,

Fig.No.15

and k.

Proceeding exactly as in

case

5,

(iii), if

OL~x,

*z, then

x,y and z are called the coordinates of the point

OP*OL +LP*
2

Also
of

OP on OX,

i.

as

OL

is

the orthogonal projection

^OLF^77/2

or

OP*=OL*+PR*+RL*=OL*+ON*+OM*

or

i. e.

square

coefficients

=sx 2 +j> a

of

the

P.

2
,

where r

is

module of vector

ofi, j and

tuhen

is

the module of OP.


r is

sum of

the squares

of the

expressed in terms ofi, j, k.

Direction cosines of OP.

Again
then cos
/.

if

OP makes an
OL x

angle a

with the direction of

i,

oc^^^y
*=r

cos

a.

Similarly y

=r

=r

cos ft

cos

y,

where

and y are the angles which OP makes with the directions


of unit vectors j and k respectively, cos oc cos ft cos y are
j8

22

Vector Analysis

called the directions cosines of


1,

OP and are

written as

m, n respectively.
*2

But

+y+

+J

-f

=r 2

2=1*

(cos

a+cos 2
2

cos a-fcos

:.

j8-f

cos 2
2

y).

0+cos

= l.

Direction ratios of OP.


Also cos

a=

-,

cos /?=

direction cosines of

the

coefficients

of

OP

i,

cos

y=

of

shoeing that the

are propoitional to #, jy, and z, i. e.


and
and the actual direction

cosines are obtained

module

by dividing their
OP; i.e. V(*2 -h? 2 + 2 ).

coefficients

are

*,.)>,

by

r,

the

called the

rectangular coordinates of P.

Note : In the case of unit vector, the module is unity


and hence if a unit vector be resolved in terms of i, j, k,
then their coefficients themselves are the direction cosines.
Distance between two points P l and
the direction cosines of the line joining them.
8.

Choose any point

as origin

P and P 2
l

terms of unit

where

(xll

yl

OP l and OP2
vectors i, j and k as
are

Zi)

and

(xa ,ya ,

coordinates of PI and

then the position vectors

of

P 2 and

Pa

We

can express them

in

zj are the rectangular Cartesian

i)

J+U,- *j)

k.

Introduction
If r

PiP2 and

be the
is

23

P-f^ then

distance

it

i,

k and

and

the module of

sum

equal to square root of the

of the coefficients of

is

of the squares

therefore equal to

is

Also the direction ratios of the line PjP2 are the the coefficients

The

#a~ *i ^a-.?! and

actual

direction

V[(*

or

and are therefore

-2

Putting

OP l

of

and

recpectively.

by dividing

and

P a P2

i.e. r

<1

l
m

OP2

the value of

obtained

In case

-*i)

<2

i,

are

by the module

these coefficients

Note

1\ of

22

cosines

are unit vectors, then

r,

we

find

the corresponding

direction cosines.

Ex.
and

7.

Three vectors of magnitudes

their directions

of a cube.

are

along the

diagonals of three

Determine their relultant

(Lucknow B.

Sc. 51,

2 i, 3a meet
f

a,

and

Utkal B.

in

adjacent faces

its direction cosines.

Sc.-

Hon's

53, B.

M.
Prove also that the

sum of

the three

from that

determined by the diagonal of the cube.

H. U.

Sc. 54)

vectors determined by

diagonals of three adjacent faces of a cube passing


corner, the vectors being directed

a point

the

through the same

corner is twice the vector

Vector Analysis

24

Let the edge of a cube


be taken as unity and the
vectors represented

OB,
ous
be

OR,

OC

of

edges
i,

by OA,

the three coterminunit

length

j and k respectively.
OS and OT are the

three diagonals of the

three

adjacent faces of the

cube

8
Fig.No.16

along which act the vectors

OL,

OM and

OJf of magnitudes

In order to find these vectors


in these directions

r\

2a and

a,

we

30 respectively.

shall first find unit vectors

and then multiply them by their

corres-

ponding given magnitudes.

and

its

module

unit vector along

/.

is

clearly

V (0+1+1)=

OR*= ~

V^ (j+k)
-

Now

the magnitude of vector

Exactly
tude 20

is

in a similar

vj~

manner

OL

is a.

the vector

OM

of

(k+i) and vector OJVof magnitude 80

magniis

(i+J)

25

Introduction

The magnitude

,.

of the resultant

is

40

30

50

direction cosmes.are

or

V2'

5V2*

e5V2'

Again OB+05+Or=2 (i+j+k)=s2.0P which proves


second past [on putting *=j=2=l in (1) of
7 P. 21],

Exercise No*
i.

a,

position vectors of A and B respectively, find


C in AC produced such that AC3AB, and that of
BA produced such that BD2BA.

'

are the

fiat of a point

a point

D in

be the origin, then

If

0^=a, OB=b. Now from

--

AC^SAB, we
>

get

>

>

OC- 0/1=3(05-0.4),

/.

OC3b-2a.

Similarly

OD=2a-b.
vertices

r^

/A^

vectors

of a triangle are

the points

Find

determined by the sides ?

the length

of

these vectors.

See

8.

Sides are
3.

If

(bi-aj i+(* 2 -<z 2 )


{(fc 1

its

the postiion

module.

+ (i 3 -s) k etc.

-fl 1 )H(*2-^)a +(fr3 -'fl3) 2 } 1/2 etc.


rectors

respectively.find

and

AB

of

and

and determine

^Donated by

arc

its

i+3j-7k

direction

cosines

26

Vector Analysis

Ans.
,

4.

module9V2.

4i-5j

-5

The position

11

vectors of the points P,

Q, R, S are

(1+ j +k), (21+5J), (3i +2J - 5k), (i - 6j -k).


Prove that the lines PQ and RS are parallel, and find the

ratio

of

these lengths.

In this question
lines

show

that the direstion

and RS are proportional.

PQ

ratios of the

Ratio uf their lengths

is*.

In

5. (a)

the ad-

joining figure if

OC-c,
find the vectors given

a+b+c,
cta-b,
and

by

b-fc-a,

a+b-c,

verify that the four vectors

independent.
in terms

along the four diagonals are linearly

Also find the vectors

along the

diagonals of the Jaces

0/a, b, c.

0/l-a+b-c..(3)

......... (4)
From

(1), (2), (3)

and

(4),

we

find that

27

Introduction

Hence they

Similarly

are linearly independent.

we

can find the other diagonals

the other

of

faces.

(b)
2,

1,

at the

particle

Ibs.-wt. respectively

cube which meet the particle.

0=1

Putting

6.

ABCD

is

acted on by forces

along

of the faces of the

Find

their resultant.

Ex.

solved

in

force represented

is

the diagonals

corner of a

by

a quadrilateral.

AC

act at a point.
8,

prove that

in

from

/.

act

2BA.

Add

a pentagon. Forces

AB+BA

i. e.

AB, AE, BC, DC,

Prove that their icsultant

is

ED and

SAC.

the mid-points

of the sides of

the vertices

and hence

the vectors determined by the medians of a

triangle

vectors

of

the vertices is zero.

being the mid point

oiBC.

fl=5.

----DA

(Agra 37)

get

BA, BC, CD,

terms of the position

sum of

we

Find the position vectors of

triangle

directed

Forces

that their resultant is

ABODE is

23,

V ^ (5i+4j+3k).

R=BA+BC+CD + DA.

7.

P.

--

Show

at a point.

cube

->-OD^OB+BD

28

Vector Analysis

c-b
- b+c
-

etc.

etc.

Prove that the following vectors are coplanar

9.

5a+6b+ 7c,
Find

10.

vectors, a,

the

7a-Sb-f9c, 3a+20b+5c.
between the following systems oj

linear relation

b, c being any three non-coplanar

vectors

a+3b+4c, a-2b+3c, a+5b-2c, 6a+14b+4c.


Ans.

n.
of

the

(ia+14b+4c(a-f 3b+4c)+2 (a-2b+3c)


+3 (a+5b-2c)
resultant of two forces is equal in

If the

components and perpendicular

to

it

in

to one

magnitude

direction, find the

othet component.

Let the force


zontal and Q, be
it

at

an angle

resultant

is

0,

be

hori-

inclined

so

to

that the

a vertical force

P. If the unit

horizontal

vectors along

and

vertical

be

denoted by i and j, then


force P is Pi and Q, is Q, cos 0i along the direction
()

sin 0j along

j.

Equating the

The

coefficients of

Q, sin

or

Q, cos

resultant

0=P,

0= - P

/.

and
tan

is

Pj.

and

P+Q,

0= -

j,

we

cos

get

0=0.

or

0135

of

and

29

Introduction

Find

12.
angle

of 60

the

horizontal force

to the vertical

and

the force inclined at

resultant is a

whose

vertical force

an

ofP

Ibs.-wt.

Ans.

2P.

V3P,

If two concurrent forces be represented by

13.

n.OP and

w.OQ,

prove

respectively,

that

their resultant

is

given by (m+n).

>

OR

where

divides

PQ, such

that

n.PR=m.RQ.

/.

n.OP=n.OR+n.RP,

.*.

m.OQ^m.OR+m.RQ.

resultant of

72.

OP

and

is

Now we

are given that

->

n.PR=sm.RQ.

->

-^
or

-n.RP*

or

.%

resultant

the ratio

is

(m-h)

0^2.

The

point

Cor.

/?

divides PQ, in

In case the forces be

1,OP and

l.OQ,, then their

80

Vector Analysis

OR

resultant will be (1+1)

the ratio 1

i. e.

i.e.

QOR where R

the middle point of

is

divides PQ, in

P&.

Prove that the system of concurrent forces acting at a


point

14.

--*
and

OC

OA, OB,

represented by

is

equivalent to the system

of forces

____
OD, OE,

represented by

are the middle points

OF acting at

ABC.

the triangle

(a)

Two*forces
-

ABCD

where

ofBC

middle point

is

Write similar other relations and add.

13).

1*5.

OA+OB+OC=*OD+OE+OF.

that

Now OB+OC=2OD
(by Q.

same point where D, E, F


and AB respectively of

_>____._

We have to prove
-

the

EC, CA

sides

of the

corner

of a quadrilateral

->

->

AB,

represented by

at the

act

AD

and two at

represented by

CB

and

->

CD.

Prove that their resultant

F are

the

middle points of

AC

is represented

and

BD

(Agra M.

where

E and

[Use cor.

P the point

of

Q,. 13]

intersection

middle points of opposite sides. Show that the

lines joining the

--*--

resultant of

4EF

respectively.

Sc. 58)

Proceed as in Q. 14.
ABCD is a quadrilateral and
(b)
of the

by

->

OD is

A, OB, OC,

equal

to

40P

where

is

any

point.

16.
transversal

Prove that

Forces

Q,

act

at

cuts

their

lines oj

A + OB =Q C
Q-

and have a resultant


action at A,

'

(Agra

and

R.,

If any

respectively,

^ ^ ^. Luck B
.

Sc. 49)

Introduction

P and

Let the forces


-

OL and

represented by

Q be

OM

so

that

the

diagonal

ON

repre-

sents the force R, so that

R ...... (1)

Let

CU=a

and

(2)

P is

same

in the

direction as

a and hence

it

can be

>

expressed as

- p

0-4.

->
Similarly

Q**n OB
.

and

let

R*=t

We

OC.

---

have to prove that


Q'd+ffB^n^

From

(1),

we

get

or

m .OA+n. OB=t .OC

...... (3)

or

b,

we

3=1

or

Comparing a and

A
Note.

get

=1-A; and "T"^-

m-j-n=f.

Hence proved.

Alternative method for this question will be

gevin in next chapter.

(Ex. 16 P. 70)

82

Vector Analysis

Ex. 17.

Pl

and

P2

Prove that the


2

>

^i +^ 2

is

magnitude of the resultant offorces


is the angle
+2P1 /J 2 cos 0) 1 ' 2, where

between the direction of the forces.


If

/!,

%,

fy

lines of action
i,

and

^2

/2 ,

n 2 be the direction cosines of the

forces with reference to unit vectors

of the

and k, then these forces can be expressed as


Pil l i+P l ml \+Pl nJk and P
If

The magnitude

Note.

a triangle

18.

is

(ii)

(ioi)

APis

is the

is

given by

is

number

given by

IfO

ABC,
(i)

where

of

If there be

the magnitude

Ex.

be their resultant, then

{2P1

of

forces

Plf P 2 P3 ,.
,

+2ZPf P, cos (Pr P

s )}

circum-centre

and 0'

-->-->

OA+OB+OC=00',
0'A+0'B+0'C~20'0,
AO'+0'B+0'C=*AP,
of the circum-cirde

Before doing the above


problem we should note that
in the adjoining figure

20DAO'

(by geometry).

OB+OC=*20D, where
the middle point of BC

(i)

is

(O. 13 Cor,).

then

the ortho-centre of

then prove that

the diameter

1/2 .

D r
Fig.No.22

Introduction

---

33

/.

OA+OB+OC=*OA+<20D=OA+AO'=00'.

(ii)

O'^+O'C^O'/)^ (0'0-fOZ))20'0+20Z)

->

(Note)

(iii)

from

(ii)

(the vector represented

through

by the radius

of the circum-circle)

= AP where AP is diameter.
19.

ABC

the resultant

is

and

triangle

->>
PB

of AP,

PC, show

and Q, therefore a fixed point.

P any point
that

ABQC

in

is

BC

If

PQ

is

a parallelogram

(Luck. B. Sc. 54)

AP+PB+PC
Now

CD
AB.

through

parallel

C draw

and equal to

Therefore

parallelogram,

ACDB
so

But the resultant


with Q.

is

that

is

given to be PQ,. Hence

Since with the change of position of

D
in

coincides

BC,

AB

Vector Analysis

34
is

CD

not affected or

not affected, there ore

is

CQ,

i. e.

Q,

is

fixed.

20*

man

wind seems

to

he finds that

it

8 miles per hour finds that

east at

travelling

blow directly from the north.


appears to come from

JV.

On

doubling his

Find

E.

the

the

speed

velocity

oj

the wind.

Let us
unit vectors

towards

m.

and

-E.

sented by
lling

at

is

as he

8 m.

repre-

the velocity of

xi+yj where
required to

Now

towards

Again suppose that


the wind

east.

w~

velo-

travel-

is

p. h.

.^

res-

JV.

man

the

h.

p.

pectively so that the


city of

/V

represent

i,

of

velocities

that

suppose

Fig. No 24
in the

and j

plane

is

given by

we

and j are scalars whose values

are

find.

the velocity of

wind

to the

relative

man

is

given

by
actual velocity of

wind + velocity

of

man

reversed.

We are given that the wind appears to blow front north


and hence

it is

given by

/J.

(*-l)i+.rf--/d; /.
Again when the man doubles
/.

2i then the relatjyej^locity -is

But

in

this

from N. E. and

is

pas'e'-* tyfe

(*i+jj) 2i

are given that

therefore given by

and

j>

Putting *==!

*-l0
his speed

we

-/2

sl,

becomes
i+jj.

appears to

blow

(i-f j).

-/,.

find that

it

(x - 2)

or
it

*L

or
i. e.

and hence

j-

35

Introduction
/.

module
8

wind

velocity of
is <\/2

in. p. h.

units

and clearly the direction


Show that if two

21.

the one are proportional to those

that

the

given by i-j
-(j-i). Its
h.
as
one
unit
p.
represents

is

8V'2 m.

e.

i.

three points

from N. \V.

vectors are parallel, the components

of
Hence or otherwise show

of the other.

B (5, 0, -2), C
which B divides AC.

-2, -8),

(1,

are collinear and find the ratio in

Ref. P. 13.

is

of

Any

two

the

Ans. 2

expressed as a multiple of the other.

The

22.

B(-4,2,-2),C
forces
lively.

are

quadrilateral

and

(2,

- 1,

Ibs-wt. act along

3).

A(l,2,l),
At

the point A,

AD

AB, AC,

resrec-

their resultant.

find

AB** *2. unit

in

Express

23.

unit vectors along

force of 2 Ibs. alog


1

2, 3,

Refer Ex. 7 P.
and then

- 5)

(4,1,

of magnitudes

Find

of a

vertices

can be

vectors

like

(11, 3, 7)

AB,

terms of unit vectors

AC

vector along
.

'' ''

vas

'

-y_
)'

-^
vUi8)'

AD

AB
.

magmtude

direction cosines of result are

_ _

and

then the

etc.

1S

//11B\

V UT>

CHAPTER

II

CENTROID, LINE AND PLANE


To find

i
the

join

a and

the

vector

position

of two given points

b in a given ratio say

and

the point

of

with divides

whose position vectors are

n.

\Ve are given that

Now

m+n

AB.

m-f

TZ

-^
or

(b-a);

-.

m-\-n

->

Alternative.

From

(1),

we

get

or

or

->

n.AP=*m.PB

OP~n.OA+m.OB

Centroid, Line and Plane

Middle point*

Putting

vector of the middle point of

m=n=l, we

AB

37

......... (3)

as

the position

get

JL

The

Note.

A and B

point

is

with associated

and divides

AB

internally

numbers n and
in the ratio m
n,

the point

respectively

Cartesian equivalence.
Let us suppose that in terms

centroid of the points

called the

of unit vectors

i,

is (x lt

*2*+.)'j+**k so that the point

is (* 2 J>a

P be

the point *i+j>j-|-dc, then from

Equating the

coefficients of

m+n
2 Centroid

_
V
7

i,

m+n

y l9 ^)

(2),

*}

we

get

and k we get
;

Z __^

~~"

*"

m+n

and centroid with associated numbers.


(Agra

If

and

is

given by
tfii+JiJ+^ik so that the point
and that of B is

If

there be n points whose position

be given by
any origin
whose position vector is
C/Crsss

M.

vectors

Sc. 53, 58)


to

relative

then the point

a, b, c, d, ...

----

(a+b+c+d+...)
n

is

called the

centroid or centre of

mean

position

of

the

given points.

Again if there be n scalars


whose position vector is

q, *, s,

then the point


.

Vector Analysis

38
is

the centroid of the given points with associated

called

numbers p,

Note

q, r,

cover up

s...

The words associated numbers

forms of centroid,

all

m^ m 2 m 3

the masses

. .

e.

parallel

will

.,

may

of

if

we

of gravity or centre of

place

masses

of

particles

whose

J3, C,
position vectors
then the position vector of their centre of

at points A,

are a, b, c, d.

mass G

Thus

forces.

tt*3-

used to
stand for

particles placed at
system
the centroid is called the centre

of a

given set of points; then


of mass, i.e. c.m.
Similarly centre

mi> f*hi

they

g.

are

. .

be

Equivalence of Cartesian Forms.


Let us express the vectors
unit vectors i, j and k, so that

A
B

the point
the point

i.e.
i.e.

a, b, c,

is (x l9

is

terms of

in

etc.

y lt

Zi\

(x^y^

2)

............................ and so on,

and

us suppose that the


the coordinates of the centre
let

from

(1),

we

of

so that

G=*i4:jj+k,

point

mass G

are (x,y,

then

z)i

get
-

2m

we

Equating the coefficients of i, j and k,


coordinates of the centre of mass as

get the

27

Ex.

Particles

z.

respectively

are placed

at the corners

27

nij

A, B, C,

of masses
at the corners

27

nil
1, 2,

3,

of a

of one face and

m^

4 9 5y 6,

unit cube,
the

last

7,

grams

the first

four

at

four
their

Centroid, Line and Plane

D'

projections A', B', G',


the coordinates

of

as

and

AD

let

AA',

on the opposite face.

respectively

their centre

39
Find,

of mass.

origin

AB

and

represent unit vec-

tors

and k respec-

i,

tively.

The
of

are

(the

vec-

position

the 8

tors

corners

masses of the

are
also
particles
written within brackets-

Fig.No.26

(1)

,4=0

(5)

(2)

J5=j

(0)

4=i+j

(3)

C=j+k

(7)

C'

(4)

Z)=k

(8)

D'

IT=i

position vector of the centre of

.*.

Tt

B'

mass say

is

.0+2 j+3(j-)-k)+Jk+5i+C(i+j)+7(i+j+k)+8(k+i;
.

1+2+3+4+5+6+7+8

magnitude

/.

of

^G

Also cooadinates of c.m. are


//

3.

position

q, r,.

<w<f

60

/A* centroid

vectors

G'

/Afl^

whose position vectors


P'

q'i r '*

^^

are

(Jfi

Ai

is)-

of a system of points A, B, C,.

a, b, c...

wiVA

associated

numbers

of another system of points A' 9 B', C'.


are a',

b',

c'

wtVA

associated

. .

numbers

centroid of all the points taken together is the

40
<

Vector Analysis

centroid

the

of

(P+4+r.

By

.)

if

the

fce

numbers

associated

and
.

G'
.

with

associated

numbers

.) respectively.

we have

definition,

Now

points

and (p'+q'+r'+

of the points

centroid

(/>+?-fr-f-..

and G' with

and (p'+q'-\-r'+

.)

.. .,)

respectively, then

0//=-

------

/a+?b-H-c-f/a'4-?'b'+r'c'4- .

or

is

To

that

prove

centroid

is

independent of the origin of

(Agra

vectors.

Let G be the centroid


1

tors

clearly the centroid of all the points taken together.

4.

points

whose

position
to

relative

be a, b, c.

numbers

. .

p,

of

M.

Sc* 53, 58)

Q'

vec-

an origin

with associated
g,

r.

respec-

tively.

Fiq.No.27

Now we have to prove

that

if

instead of

a)

we choose any

point 0* as origin, then the position of the centroid

will

Centroid, Line and Plane

remain unchanged.
the position vector of

Let the position vector of 0' be


A relative to 0' will be

Similarly position vectors of

the

new

= OG-00'

or

0'G'

or

0'G' = 0'G.

i;

then

0' as

relative to

etc.

C,

b-i, c-i. .etc. If C' be


C. G. relative to new origin 0', then
origin aie

41

position

of

[from

(1)]

Above relation shows that the point G' should coincide


Hence the position of the centroid remains un-

with G.

changed with the change

To prove

5.

of origin.

that the vector relation

/a-f qb+rc+...==0
will be independent of the origin if

and only if />+#4-?"+

=0,

where p,q,i.. -are scalars.

With

reference to the origin 0,

we have

/>a+0b+rc+...=*0

Now
to

be

i;

origin 0'

is

(^a+^b+rc+
If

then

and

(3)

possible only

. .

......... (2)

.)-i(p+q+r+

. .

.)

...... (3)

to be

independent of the origin


should represent the same relation which is

the relation

(1)

(I)

choose 0' as origin whose position vector relative


then as in last article the relation (L) w. r. t. new

p (a-i)+ j (b-i)+r (c-i)+


or

the relation

........

(1) is

if

..=Q

.........

(J)

Vector Analysis

42
Also

same.

p+q+r+.

if

.=0, then

We

have

already

Thus

satisfied.

that

shall

see that

property

is

a, b, c.

with associated numbers

if

be

p, q, f .

the
.

.,

. . .

. .

is

the above

centroid

of

then

- R (p+q+r+ .)=0.
the above relation the sum of the coefficients of
is (p+q+r+
.)-(p+q+r+ .) which is zero.

p*+qb+rc+

In

centroid

proved

and we

independent of the origin

vectors

represent the

(3)

Hence proved.

Note.

or

and

(1)

the

Exercise No.
I.

the

is

another tetrahedron

such that

Prove

bisected at G.

of tetrahedron

centroid

that

ABCD.

AA', BB', CC'


is

also

the

and

centroid

A'B'C'D'

is

DD'

are all

the

second

of

tetrahedron.

With

respect

a+b+c+d where
C and

B,

C' D' be
t

to

any

origin

a b c d

D respectively.
a', b', c',

Now G

is

'

point

the mid. point of AA', BB',:,CC', DD'.

= d_? b^5

d'^

is

of A', B',

b+c+d-a D=

vectors of A,

Let the position vectors


d' and we have to prove that

-g-

the

are tbe pos ; tion

..

say

Centroid, Line and Plane

43

~~

4
2,
so on

-4

ad

ttlaced at

from

AB

line

2,

bisected in

Ol9 O^B

Particles of masses m,

infinitum.

O ly

is

Show

j.

that the points C^,

If

"93
^-

Y"'

a
a-A
vir^/'J 2

c.

m. of the particles

"

^'-

w /a
(

'

^ a

4 be

^^

are

to

c.

taken as a so

a
then

"
G. P.

of

BG=

("Sum
T

...CO

Above shows
that of

A from

m.

B.

at these points,

2+02 +g 3

or

and

distance of their

V^y^^

m/a\

are

^L
be the

in

as origin, the point

2,

Am2~b2
Let us choose

-^ m, ^~

that the

2,

distance from

is equal to one-third of the

in

that the distance of

7^
)

from

a
__^
L

is

one-third

Vector Analysis

44
Find

3.
3j,

of 3n points

centroid

the

2i,

i,

3i...wi;

j, 2j,

"k.

.j; k, 2k, 3k,.

mats are placed at (n 2) of the corners


Find their c. m.
of a regular polygon of n sides.
b>3 the position vectors of the
Let a, b, c, d, e, f
Particles of equal

4.

polygon with respect

the

of

vertices

of the

centre

to

polygon. If equal masses were placed at all the corners,


then the c. m. will coincide with
and hence

Now
the

let

b b2 vacant anl G be

the vertices a,

the

c.

g. of

placed at the other vertices

remaining (-'2) particle,

so that

c -f d
4-f -(-...
-f-e--*T^
------ =
OG~
-

r
u>i
{-(afb;} from
/

/IN
(i)

-25
where P

is

the mid. point of the join of vacant vertices,

or

5.

or

w-2r

some of which attract anl some


as

the

distance.

The

by a number of centres offorces,

on

acted

is

particle

repel,

intensities

the force in each

case

Prove that the resultant passes through a fixed point for


tions

of

forces
. .

posi-

Sc. 40)

be the position of the particle and the various


be a, b, c. . w. r. t.
as origin.
L
3
2

centres of force

The

all

(Agra M.

the particle.

Let

varying

for different centres are different.

acting on the

where p l9

p. 2 , /i 3

particle

are

given by

are constants which

may

/t^a,

b,

/u 2

bs +ive

Centroids, Line and Plane


or

45

according as the centres attract or repel.

ive

resultant of these forces

is

The

given by

l^a+^b+^c-f ............ (1)

Now

if

numbers

But

be the centroid of

/;

^3

..,

a, b, c.

with associated

then

independent of the origin of vectors

is

I. e.

the particle.
Hence G is fixed. Thus the resultant passes
a
fixed
through
point for all positions of the particle.

IfG L

6.

RZJ

C* 2 ,

is

the

mean

centre

of

A l} Bls C l

A4+A+C C
1

Hint.

that of

A 2,

be the in-ccntre of the


triangle

ABC

and

a, b, c

be the

the sides, then prove that the


forces

of

We

->-->

a .iA, b. IB, c.iC are in equilibrium.


have to prove that

Now we know
centroid of

and

Gx

be the origin, then

If

//

7.

lengths

and

then show that

points

that
A,

the

incentre of a triangle

C, with associated numbers

is

the
a,

Also we know that centroid is indepenrespectively.


dent of the origin of vectors.
Therefore if we take i as
c

origin

are i4,

of

vectors,

ifl

incentre

then the position

vectors of the vertices

->

an

1C respectively and the position vectors of


w. r. t. \ as origin will be 0.

i itself

46

Vector Analysis

---

a.iA+b.iB+c.iC

0=

a.iA+b iJ5+c,iC=0.

or

Hence the
in

triangle

D,E,F

AB

g.

(4, 2, 3),

and 3

AD, BE, CF

vectors

vertices

3:2

sides

the

--->

in the ratio 1

The

divide

1:4,

ratio

the

sum of the

that the

divides

forces are in equilibrium.

The points

8.

a+b+c

of a

Show

respectively.

CK where K

is parallel to

3.

of a

ABC

triangle

Find

(6, 3, 4).

AB

BC, CA,

the

are

coordinates

1,

(2,

3),

P and

of points

>

which divide

BC

in the ratio

3:2

AQ has

Also show that vector

direction.cosines proportional to 8, 6, 9.

In terms of unit vectors

is

21- j-3k
10.

etc.

The position
are a, b, c and

j and k the
Ans. P (V,

i,

7 P. 20.
vectors

of the

n and m'

Q, which

ri

respectively.

is the intersection

The
forms w.

of

AB

and

BD

the ratios in

CD

Q, (10, 5,

6).

of a quadrilateral
The diagonals AC and
in the ratios

which

the point

divides these sides.

P written
position vector of point
are respectively
r. t. AC and

in

two

different

BD

wa-f me

/.

Find
and

),

vertices

d respectively.
ABCD
BD intersect at the point P which divides AC

position vector of

n (m'+n')

n'h+m'd
"~

(m+n) b+m' (m+n) d


bro'
(m+n) d-m (m'+n c. .(A)
(m+n)

a+m (ro'+n') c=*n'

or

n (;rc'+;z')Ja-tt'
Now n (m'+n')-

and

m' (m+ri)-m (m'+n')=*nm'- rim.


Hence dividing both sides of (A) by wn'-n'm,

n'

(m+n)=*nm'

n'm

n(m'+n') *-n' (m+n)b "~ m' (m+n)


nm' - rim

we

d-m (m'+n

nm' - rim

get

47

Centroid, Line and Plane

L. H.S. represents a point on AB dividing it in the


r
ratio -n (m+ri) n(m'+n') and R. H. S. represents a point on
DC dividing it in the ratio -wz (m'+n f ) m' (m+n).
:

Since the

CD

and

are

vectors of both these points on AB


it is their
point of intersection

position

same,

hence

&

i.e.

Vector equation of a straight

6.

To find

the

vector

equation

a and

through a given point

of a

line.

straight

is parallel to

line

that

passes

a given vector b.

Let P be any point on


the straight line through

a which

is

and

position

its

to

parallel

vector

AP

is

Now

r=OP.

be r so that

parallel

to

Fiq.No.2a

and
>

hence

where

it
t

must be some multiple


is

of b.

AP=tb
side of A

Therefore

a constant positive for


points on one
for points on the other side.

and negative

-->-

Now

OP=OA+AP.
r=a+/b

.\

P on

Every point

the line

(1)
is

obtained for some value

and for every value of / we get the position vector of


a point on the line and hence above represents the vector
of

equation of the required line.

Cor.

x.

To

find the vector equation of a straight line through

origin.

Putting

a=0

in

(1),

we

get the required line as r/b..(2)

Vector Analysis

48
Cor.

To fnd the vector equation of a line


A and B whose position vectors are a and b

2.

given points

(Agra

M.

through two
respectively.

Sc, 42, 45, 47, 52)

Now AB^OB-OA^b-a;
therefore the required line is
and passes
parallel to b-a
A.
through

Hence from
tion

is

(1) its

equa-

given by

Fig.No.29
or

r=(l-/
Cor.

of the

the cartesian equivalents


above.

of the vector equation

lines (1), (2), (3) found

Let
the

To fnd

3.

in

vectcr

[Refer author's book on Solid Geometry]


terms of unit vectors i, j, k through the origin 0,

r=*i+^j+k so

that

is

the point

j+0 3 k so that the point A is


b=i 1 i-)-fc2 j+4 3k so that the point B is (4 lf 4 2
Substituting in (1), i.e. r=a+/b, we get.
assflji-f

Equating the

Above

coefficients of

i,

and

(x,

}\ z)

and

a3)

and

2,

(fl 1?

4 3 ).

k we
;

get

corresponding cartesian equation of a


straight line through a given point (a l9 a 2 a z ) and whose
is

the

direction cosines are proportional to b l9 b 2 b 3


,

Again substituting

in

bi

which

2) i.e.

b2

r=/b we

get

b%

the cartesian equation of a straight line through


with
direction cosines proportional to b lt b 2 b 3
origin
is

Centroid, Line and Plane

Again substituting

we

ing as above,

which

in

r=a-H

(3), i.e.

49
*0

fa

and proceed-

get

the cartesian equation of a straight line through


the points whose coordinates are (a lt a z
3 ) and (b lt b& fr 3).
three
to
be collinear.
Condition
for
7*
points
is

(Agra M.
To prove
in

points

that the

necessary and

sufficient

three dimensional space to be

a linear relation connecting

Sc. 32, 37, 51)

condition for

three

collinear is that there exists

their position

vectors

sum of tlie coefficients in it is zero.


We have seen that the equation of a
through a and b is given by

such

that

the

algebraic

straight

line

which may be written as

(1-0 a-Hb-r=9.
Above

a relation between

is

the

position

vectors of

B and P which are collinear and we observe


algebraic sum of the coefficients of a, b and r is

three points A,

that

1 which is zero.
Hence the condition is necessary.
In order to prove that the condition is sufficient let us
suppose that any three vectors a, b and c be connected by

/a+wb+nc

the relation
n (n^O),

we

L a+

ing

/+m+=0

where

Dividing by

get

Putting
/.

0,

b+c0,

where -~ +

we
--=-*,
n

-a-

get

or

^ +1-0.

(Agra 45)

-n

c-(l-<) a+/b.

Above relation shows that c


a and b and hence a, b, c are

is

a point on the^ine joint

collinear.

50

Vector Analysis

Alternative Proof*

Let us suppose that the points a b, c are collinear.


Let c divide the line joining a and b in ratio m / so that
:

(l+m)=^n or /+m+n=0.
to
order
Again
prove that the condition is sufficient
us suppose that /a+mb+-c=0, where

/a+*ttb+wc=0, where

or

in

let

or

-nc=(/+w)

/a-}-mb

/a+mb =

or

l+m

in

c,

c.

Afcove relation shows that c divides the join of a and


Hence the three points a, b, c are
the ratio m I.
:

collinear.
8.

Bisectors of the angle between

two

straight

lines.

Let the equations of the lines AL and AM be given by


r a-Mb and r=a + *'c
so that their point of intersection
tion vector

b and

Let

the

points B,

from A.
then

we

posi-

modules

be

by corresponding
choose

C and

three

on AL,

LA

(produced)

unit

distance

and

at

whose

litters a, b) c.

Now

AM

the point

c.

denoted
italic

is

a and they are parallel respectively to vectors

is

If

4Pand

P and

AQ

F^ No 3

middle points of BC and CD,


are the required bisectors whose
equations
Q,

are the

are to find.

OB~.
.

Now

position vector of

is

Centroid, Line and Plane

AB

.*.

Similarly that of

Therefore

P and

is

a+

is

Now

a.

vectors are

position

AP and

lines

AQ, as

and

and that

a+J

using the formula

a straight

equation of

as such

of

Q, being the mid. points of

have their position vectors

and

b and

a unit vector along

is

51

line

a and

b,

r=a-fk.iT

r=a+k'.i

b
is

a--,-.

BC and CD

\ a+J

-f

r=a+* (b-a)
the points

through

we

it is -T-

~Ty
as the

whose

get the equation of the


-\

T-

] respectively

or

Note. In case the point of intersection of the two lines


origin, then the corresponding equations would be

were

If

b and

c are the unit vectors, then

A A

A
b=6b
A A

A
and c=cc.

r; (b+c) and r**t' (c-b).


Alternative method.

AL

Taking the point A as origin the equations


are r==pb
and

AM

and r~p'c. Take any


pt. P on the internal
bisector

and draw PJV

parallel

to the

tion of

AM.

fore

direc-

There-

of the lines

Vector Analysis

52

ANNP* Now

hence
directions

same

c and therefore they are the


the corresponding unit vector in

multiples of

scalar

24JVand JVPare in the

b and

vectors

of

that direction,

~*
i.e.

AN- tb =

if

t -r-.

--b

-*
A
c
then JVP=/c=/

AP' is obtained by
Similarly the external bisector
of
lines which are paralconsidering the internal bisectors
lel

b and

to

c and therefore

its

equation

is

A A
Ex.
and find

Prove

i.

the point

of

of concurrency.

ist

Method.

Let

a, b,

tion vectors

the

points

c be the
the

of

D,

52,

Agra

52, 55)

posi-

vertices

mid.

the

of

(Lucknow

so that the

triangle

co-ordinates

medians of a triangle are concurrent,

that the

and

of the

sides are

b+c c+a
'

and

respectively.

Now A

is

which divides

a and

AD

D
in

is

b+c
T-

and therefore the point

the ratio

trisection) is

a+b+c
"2+1

(i.e.

the point of

Centroid, Line and Plane

53

The symmetry of the result shows that the point G


on the medians BE and CF as well and divides them in
the .ratio 2 1. Therefore the three medians are concur-

lies

which

rent at the point

also the centroid of the

is

given triangle.

2nd Method.

We

know

the points
or

is

b,

the vector equation of a line joining

that

ind

is

r=a+' (b-a)
r=(l-/)a-f/b.
B

is

b and E

is

Equations to medians

AD

and

is

BE

are

......... (1)

rd -*) b+*

and

two

If the

find

some

identical

straight lines intersect,

and

suitable values of s

values

coefficients of

......... (2)

For

of r.

we

this

equal vectors in the

we

should be able to

which
shall

two values

should give

compare the
of r.

~~"
2 '~2

Solving
t

and

s in (1)

we
or

get

(2)

we

intersect at the point

Ex.

2*

Substituting the values of

*ss,y=f.

observe that the medians

AD

and

^
o

Prove that the internal bisectors of the

angles

of a

triangle are concurrent.

(Agra

BE

*c

47, 5*, 57;

Lucknow

53;

Dacca

54

Vector Analysis

Let

c be the posi-

a, b,

tion vectors of the

A B and C

respectively

and

0, y

a,

be the lengths

BC,

CA and

respectively.

Also

of the

AB

vertices

sides

C=c-band CM=a-c and ^=b-a.


Now by 8, the equation of bisector AD

.-_+.b+c

or

Similarly
bisectors

we can

BE and CF

is

P J

write

down

...... (1)

the equations of

the

as

_._-c--

a ......... (2)

and

we should be able to find some suitwhich make the values of r identical.

If (1), (2) intersect

able values of p,

For
and c

this

in (1)

we

and

q, s

will
(2).

compare the

coefficients of vectors a,

and Plane

Centroid, Line

55

On

putting the value of p in (1) or of q in (2), we get the


position vector of the point of intersection of (1) and (2) as

oca+/3b+yc

The symmetry
point

of

of the

intersection

result

shows that

three internal bisectors meet at the r


point
is

this is also the

other bisectors and hence the

of the

-c which

~r

the centroid of the points a, b, c with associated

a, 0, y,

the lengths of the opposite sides.

i.e.

(See alternative

The

(b)

numbers

method

internal

the external bisectors

of

in

Ex.

5).

of an angle of a
other two are concurrent.

bisector
the

triangle

and

Let C be the origin and the


position vectors of

respectively

A and B be

a and b.

Again

the lengths of the sides BC,


CA and AB be a, b, c respectlet

ively.

Equation
bisector of

LC

to

the

internal

57
Ftg.No. 34

is

A A
(1)

Equation to the external bisector of

bisector of
is

by

8,

angle between

CA

i.e.

internal

produced and

^Bixb-a

Vector Analysis

56

external

Equation to

bisector of angle between

by

bisector of

Z.S,

i.e.

internal

CB produced and BA

i.e.

a-b

is

8,

...... (3)

Now
paring,

and

(2)

(3)

intersect

at

L say and hence on com-

we have

or
'ft
J I

JL-I

2
c -{-bc-\-ac

or

abc

ab\

we

get the point

(ab

or

abc
s

(a-\-b-c)bc
be
S

=j+b=c'
Substituting the value of s in

or

1+
+

-.

---

fl+i-^

r~r~"^
r~r~"
a+b~

(2),

a + ~TT

This point

will lie

on

(1), i.e.

*
if

we choose

Hence the

three bisectors are concurrent.

as

57

Centroid, Line and Plane

Ex.

3.

Prove

that

internal

the

containing the angle.

in

(Pb. 60,

With reference

to

Ex.

the ratio of the sides

Lucknow

point of intersection of the internal bisectors

Thus

(i)

vuth associated numbers a and

/3-f y

corresponds to the point A\ therefore


vector of the

ancle

point I) \\here

A meets BC.

But

>-

and C with associated numbers

BC

in the ratio y

/2

or

AB

the
is

is

point a and

the centroid of the

is

B. Sc. 53)

position vector of the

the

2,

oj any angle of a

bisector

triangle divides the opposite side internally

respectively.
-

--

is

Now a

the position

internal bisector of the

the centroid of points

and y and therefore divides

/?

Hence proved.

AC.

Alternative.

^A (Origin)

3D

>

Fiy.No.tt

Taking A as
be

b and c
The

origin, let the

respectively and

let

position vectors of

AB*=y and AC=f}.

equation of the internal bisector

A A

AD

is

B and C

Vector Analysis

58

,. t

......... (1)

py

For

values of

all

By
*=T-> we

Choosing

we

/,

point on the
j8b4-yc

get a

on

g et the P oint

the centroid of the points b and c with


BC.
ft and y and is therefore a point on

Hence

BC

-^

in the ratio y

Note.

gives the point

or

By

D which

li

1S

numbers

clearly divides

AB AC.
:

and solving with (1), we can


D direct as found above.
2.

but

!t >

associated

Writing the equation of

1.

bisector.

BC

as

find the position vector of point

considering the external bisector,

and

its

can

now show

BC

intersection with

that external

side externally

at

is

the point

----

We

the opposite
containing the

bisector divides

the ratio of the

in

P-y

sides

angle.

Ex.

ABC

4.

is

a triangle in

the middle point

A meet
of DD'

ofEE' and FF'

respectively.

bisectors of angle

which the internal and

the opposite side

BC

in

external

A'

and D'.

is

Similarly B' and C' are the middle points

Show

that

the

points A', B' and C'

are collinear.

With

D_,

are

reference to Ex,

j8b4-yc
/,-.

P-ry
.

AA'

1S

2,

the position vectors of

and

,.
~ 5 *- respectively.
and Bb-yc
,

p-y

^b

4-yc

+
,

-*

j8b

- yc\

//3

b - y ac\

--" a

say

Centroid, Line and Plane

59

Similarly the position vectors of B' and C' are

-'

Now we know

i. e.

2
(/3

(^

Hence
Ex.

say.

a', b', c' will

form

-y 2

la'+Afb'+JVc'O such

-a2 b'+(a 2 -/32


b- y ? c+y 2 c~ a 2 a-

a'-f (y

La! +Aflb'+JVc'-i

and also

7 P. 49 that the three points whose


be collinear if there exists

by

position vectors are


a relation of the

Now

,,/- jWb\
sayandi--c

.,

(j8

2
2
-y 2)+(y*-a )+(a2 ~ e )=0
j

i.

that

c'

e.

the three points A', B' and C' are collinear.

5.

Prove that internal bisectors of the angles of a triangle

are concurrent.

We

have already proved that the internal bisector


divides the opposite side in the ratio of the sides containing
the angle. Thus if the internal bisector of A meets BC in
0, then

AD

Z)-^25

an d

is a.

Therefore the position vector of a point which divides


/3+y a is

in the ratio

]8b+yC
or

The symmetry
the

of the result

other bisectors.

Hence the

shows that
internal

it

also lies on

bisectors

are

concurrent.

Ex*
ctntroids

6. (i)

The

lines joining the vertices

>/

tetrahedron to the

of area of opposite faces are concurrent.

(Agra 43,

535

Rajputana

56)

The joins of the mid. points of the opposite edges of a


(ii)
tetrahedron intersect and bisect each other.
(Agra 34, Utkal 53)

Vector Analysis

60

Let the position vectors


of the

be

a,

b,

c,

respectively
.centroid of

and Gj be the
B,

C,

that

so

b+c+d

and AA

Now

A, B, C,

points

is

a.

is

Fig. No*

the position vector

of the point

AG l

which divides

b + c+d
'"*

in the ratio 3

be on B<7 2

CG 3
-.

Alternative

The

result

shows that the point

Hence these

which

point

of the

and Z)G 4

1 is

+ 1 -* a+b+c+d
~~

8+1
The symmetry

36

is

will

concur at the

lines

the centroid of the tetrahedron.

vector equations of the lines

AG

and

BG 2

are

+ c+d\

r=U-/)a-Kl

:-

/1N
(1)

they intersect, then for some suitable values of / and


the corresponding values of r should be identical. ComparIf

ing,

we

Putting in

l-/-, 1-J-,

get

(1)

or

(2),

we

--,-.
S

get the point

of intersection as

and the symmetry shows that

it

lies

on other

lines also.

(ii)

The

mid. point of

DA

is

--y- and that of'#C

is

Centroids, Lines and Plane

and the mid. point

and symmetry

of these mid. points

61

is

We

can
proves the theorem.
also prove the same by considering the vector equation.
Ex,
of

of the result

The four diagonals of a parallelepiped and

7.

mid. points of opposite

the

are

edges

concurrent

the

at a

pins
common

point of bisection.

Taking

as origin,

let the position

B and C

of A,

vectors

be a,

and c respectively so
that those of I), E and

are

a+b, b+c, c+a

respectively
of

is

If

and

that

a+b+c.

M!

be the mid. point of diagonal OG, then

t is

a+b+c
If

Af2 be the mid. point of diagonal

a+b+c

also the point

whose position vector


of

BD

is

M of

CF

is

Again mid. point

and

mid. point

/.

mid. point

of

LM is

2 is

{ e

DC

point of other diagonals

Similarly mid.

AE and

is

and

BF

(b+a+b)=s* ~

i
~~
>

a+b+c

is

62

Vector Analysis

which

is

same as the mid. point

we can show

In a similar manner
of the join of mid.

of diagonals.

points of opposite

that the mid. point


is also given by

sides

a+b+c
*

2
Ex*

Hence proved.

IfM,N are the mid. points of the sides AB, CD of


ABC D, prove that DM and BN cut the diagonal

8.

a parallelogram

AC

at its points

of DM and J5JV

Taking A

of trisection which are

also the points

of trisection

(Agra 48)

respectively.

as

position vectors

let the

origin

C and

of B,

be b, c and d respectively so
and
that position vctors of

and

JVare

Now

equation to

AC

is

r=/c=J (b+d).

AC=AB+BC
Again equation

of

AB

is

Equation of

DM

is

...a)

i.e.

v**(l-s)

d+*.~

or

(2)

(3)
'2

V
(1)

find

and

(2) intersect at

suitable

values of

r;

values of

is

--

and

comparing the

point

c-b+d.
therefore
s

we

should be able to

which should give

coefficients,

or

we

get

AE=$AC.

identical

Centroid, Line and Plane

63

b-2d

Again

=IDM.

=f (AM-

or

Similarly

we can prove

A BCD

Ex. 9.

is

AB

points of the sides

Ex.
produced

toD,E,F

BJf.

are the mid-

DM and

Prove that

respectively.

(Agra 48, Lucknow


straight lines OA, OB OC

AC.

Three concurrent

10.

51)

respectively.

Show

are

that the points of inter*

DE, BC and EF, CA and

of AB and

section

BC

BF=f

and

M and N

a farallelogram.

and

DN trisect the diagonal

AF=\ AC

that

FD are collinear
(Agra 45)

A,

Let us choose the point


as origin and the points
as a, b, c so that the points D, E and F may be

B C
9

taken

c respectively.
Equation to AB is r =(!/>) a+/>b
/ja,

/ 2b, /
8

Equation ty
If

DE is

to

find values of

which

r=(l-g) tp+q.tjb

(2)

they intersect say


we should be able

at

l9

(1)

give

values for

r.

us

p and

identical

Hence comFig. No.

paring,

39

and />=
or

1-qttti-qti
or

Substituting the value of p in


position vector of the point

(1)

or q in

which say

is

(2),

rx

t".

we
e.

get the

OXV

64

Vector Analysis

Similarly if ra r3 be the position vectors of the points


of the intersection of other pair of lines, then proceeding as
,

above,

Now we know
are rx , r2 r3 will

and

(5)

whose position vectors


there exists a relation of
Adding

-/!)(!-/!)

(!-/) (l""

^^+^2+^3=0 where

i.e.

which

is

(3),

7 P. 49),

ll
(1

if

f ^r3 =0 where tf+^+^0.

the form xi^-fj>ra


(4)

that three points

be collinear

zero,

clearly

Hence the

points

whose

position

vectors are r 1? ra r a are collinear,


,

Ex. XL
points of

Prove

intersection

Ihe

are

converse
real

and

of the

last

exercise,

collinear, then

i. e,

DA, EB

if the

and

FG

are concurrent.

DA and EB. Join


F
OC. Taking
on
that
also
lies
as
prove
c
let
C
us
choose
are
and
that
and
a, b,
-4, B,
origin,
F lies on OC, then position
are ^a and f 2b. In case
Let

to

be the point of intersection of

C and we must

vector, of

vector of

F should come out to


F be
*a+jb-f sc.

BCis

BF
BC

and

/8 c.

Let the position

r(lr-(l

is

be

On

comparing the coefficients,


1 -/>=(! -0) fa +ft? and J)S and 0*=0.
When gxsO, we get either #s=0 or A?=0. But if ?=0 then
clearly p is also zero and we get from 1st that f2 =sl which
intersect.

65

Centroid, Line and Plane

B and

shows that

#=0.

but

q^Q

intersection of

tion vector of

or

FC

Using

mid. points

the

of

Establish

collinear.

vector

Hence

the

also

of a straight

equation

on

OC

show

tliat

lies

line,

of a complete quadrilateral are

diagonals

harmonic property ofthefgure

the

diagonal of a complete quadrilateral

each

points of

the

get.y=0. Therefore the posi-

->
c=,cOC.

given by

is

impossible and hence

considering

Similarly

CA and DF, we

is

also passes through 0.

12.
the

coincide which

i. e.

cut harmonically by the

is

other two.
(a)

any
two

Let

ABCD

be

quadrilateral,
of

its

diagonals

being AC and ED.


Let BA and CD meet
at E and AD and BC

meet

plete

and

then

F;

at

ABCDEF

a com-

is

Ftg.No.4O

quadrilateral

EF

is

its

third diagonal.

middle points of the diagonals BD,

and we have to prove that

Chooss
vectors of

P, Q,,

as origin of

A and C

Let

P,

AC

and

and

be the

EF

respectively
are collinear.

coordinates and let the position

be a and c respectively, so that

if

B and

D may
BC

be taken as/>a and #c respectively, the equation to


joining pa and c is

The

equation to

Now

and

r(l-0.*a+fc
AD joining a and
r=(! -j)a+,y

(1)

(2)

intersect

at

(1)

qc

is

0c

(2)

F and

hence on comparing

66

Vector Analysis

we have

l-t)p=l-s ............ (3)


t=sg ............... (4)
or s(l-pq)=\-p.

!-*

Substituting the values of t in (1)


the position vector of the point F as

or of s in

(2),

we

get

.........
the mid. points of AC, BD and EF
respectively so that their position vectors are say r 1? r 2 r3

Now

P,

Cl,

are

Now

if

rlf r2 r3 be
,

and

Now

x,

collinear then

j, 2 are

or

(1 -/>?)

where

/>(/

AC and
B, T,

Z>,

[from (A)]

Ta0

collinear.

(by

Now let the diagonal BD be cut


EF in points T and 5 respectively
5

will

TJT>
H. P. or

Now

form a harmonic pencil


1

i
,

equation to

c]

- 1+1 -pq=*Q.

Hence r1$ r 2 r3 are


(b)

scalars.

r3 ==ra -^rt

/><?*!- r 2 -Ml -/?0)

or

in

any three

^^+^2+^3=0 where

r,=4 [(/>-^) a+(q-pq)

(1-jty)

(a+c) and r 2 =4

*i

^
and

AC

are
joining

if

by the diagonals
;

then the points

BT BD
9

-AOA. P.
m

t. *.

a and c

7 P. 49)

and

-65 are

112
+^

is

(1)

67

Centroid, Line and Plane

BD

Equation to

joining pa and gc

is

......... (2)

.qc

Equation to
of part

EF

AC and BD
and

(2),

.*.

to point

given by

(5)

and hence on comparing

(1)

origin

joining

(a) is

we

intersect at

get

(l-^)~(l--0/> or

Substituting in

(2),

we

rl*^ and

-s

=*ipl-q

get the position vector of

as

~
/.

length

BT

is

the

module

j?

Sr^i-^
Again
ing (2) and

fiZ)

and

(3),

we

'

BT.

__ I
"|/-jc|

F intersect

get

of

at

'

...... ^
w

5 and hence on compar-

68

Vector Analysis

position vector of S

/.

888

\pq
*>-

/i

-1"

rp(i

ES

i. e.

q)

f_TT~

Q (Is-P)

*~i _

T
r

_1-p-q

BD-ED-EB=qc-pa=-(pa,-qc).

Again

'2

Now

from
1
,

(4)

and

(5),

we

_L

BT^BS

get

R^-L"

r/w-^c| Li~^

r
?

Hence proved.

Theorem of Pappus
Ex. 13.

Bl9

(a) If there

B%i B$; then prove

that

the

the pair
points of intersection of

of

A^B^ A 2 B^ A^B^

lines

A^B2 A^B ls
,

Let

A^
be

intersection of

B^B^

z are collinear.

the

A l3 A, A&

be two sets of collinear points

^g^g^ and

^2/^y^i^^
7

which may be taken

A,A

/ ^>-^^'

point of

as origin.
Again suppose
that A 19 AK A z are

'\^

,
'

^^^^^

^^^^^

Fig No.41

^^^^B^

Centroid, Line and Plane

P&L and

B,,

to

53

2,

are ftb, ? 2b,

intersection of the

points of

prove that

Clf C 2 C 3
,

Equation to

A 2 B%

g 3b.

69

Clf C2 C8 are
lines, we have

If

three pairs of

are collinear.

joining

/> 2

a and

qjb

is

...... (D

r~(l-OAa+'ftb
Equation to ^ 3 Z? 2 joining />3 a and
Since they

#a

...... (2)
r=(l- s) A*+*ftb
intersect at C l9 we have on comparing

or

A-

Ifrjbe the position vector of the point C 1? then on


substituting for * and (1 *) in (1), we get
(Prtt-pzq^PiPs

*i

(ft -ft)

a+ftft

(P2

-A)

c.

Multiply both sides by /^ft,


or

r^ft Mt-Ms)^!^ A-ft (ft- ft)


(

+?iftft-A ( A-#s)

r2 and r 3 be the position vectors of other


Similarly
points of intersection, we have
if

't-Mt^sft-M^iPtA-ft
'3-

(ft -ft)

a+?iftft-A

(&-A) b

Ma (Aift-Aft)-^t A-ft (?i-ft) a+Jrfrft-A (A -A) b


27 rjL.ftft

.'.

(Aft-A?s)-O

i.e.

Irx+Af^+JVra-O

i+A4+JV is also zero. Hence


Aft (Aft Aft)
the three points whose position vectors are r ls ra rs are
where

27

-^

collinear.

Theorem of Desargues.
Ex.
lines

13. (b)

If ABC,

AA 1} BB l9 CC X

of pairs of sides BC,

are

A&Ci be two

concurrent, then

triangles such that the

the points of intersection

BjC^ CA, G^Atf AB, AJB^

are collinear.

70

Vector Analysis

Mr. N.
M.Sc. (Maths)

U*

Fiq No 4?

Note

For Ceva and Menelaus Theorems see Q. 9 P.

and Q. 10 P. 95.
Ex. 14* Through

ABCD,

parallelogram

and

CD produced

Take A

B and b

BP

is

of the side

AD of a
AC

at

is bisected at

drawn

cutting

Prove that

Q.

as origin and

b and d

as

pectively.

at

middle point

the

the straight line

98,

Now

res-

proceed

exactly as in Ex. 12, 2nd


A

part.

Ex* 154
and

The

BE is produced

median

AD of a

meet the side

to

triangle

AC

in

FigNo.43

ABC

F.

Prove that

and.

Ex*

16.

Forces

P9

act at

Q.

and have a resultant R.

any transversal cuts their lines of action in A,

P
then prove that

and

R
(Agra 49, Lucknow 42)

Let the forces Pand

Q,

be represented by OL and
-

OM

ON

so that the diagonal


represents the force R so that
/Z

If

respectively,

...(1)

Fig No- 44

Centroids, Lines and Planes

-*
P=*m>OA and Q*=*n.OB and

that

Again suppose
-

71

let

R^t.OC.

-^,-^and/-^r

.\

Now

--

substituting for P, Q, and

in

(1),

(2)

we

get

m.OA+n.OB=t.OC

(3)

or

But A,

B,

are collinear

position vectors

cients of their

them should be

and as such sum

of the coeffi-

a relation connecting

in

zero.
/.

/=0

m-fw

or

m-{-n=t.

Putting the values of m, n and

from

(2),

we

get the

required result.

Ex. 17. Prove that the straight lines joining the mid.
points of two non-parallel sides of a trapezium is parallel
to the parallel sides and half of their sum.

We
PQ

is

have to prove that


AB and
parallel to

equal to 4 (AB+DC).

Taking

PL

as origin,

the position vectors of

be

b and d

is

||

to

B andD

J:

^
Fig.No*45

respectively.

Now DC

.\

let

_
*

AB]

position vector of

.\

is

DC**t.AB**t.b,

AC**AD+DC**d+fo.

where

72

Vector Analysis
being the mid. point

AD

and Q, being the mid. point

of

has

BC

its

has

position vector d/2


its

vector

position

Since PQ,
parallel to

some

is

AB

scalar multiple of

AB, hence

PQ

is

mid .points

oj

and hence to DC.

A,so

PQ.-1 (AB+DQ.

/.

Ex,

P;0#e //m/

1 8.

of

/Ae?

of a trapezium

the diagonals

straight line joining the


is

parallel

sides

to parallel

and half

their difference.

Ex,

Prove that in any triangle the line joining the mid.

19,

two sides

points of any

parallel to the third side

is

Ex*

and half of its

(Agra 56, Rajputana B. Sc. 60)

length.

Prove that

220.

the

diagonals

of a parallelogram bisect

each other and, conversely, if the diagonals of a quadrilateral bisect

each other,

it is

Show

Ex. 21.

(Agra 36, Lucknow B. Sc. 54)

a parallelogram.

that the figure

formed by joining

of the sides of a quadrilateral taken in order

is

the

mid. points

a parallelogram.

(Lucknow
Ex. 22.
triangle,

the

// a straight
line

joining

line is

the

drawn parallel

vertex

diagonals of the trapezium so formed

to

the

to the base

intersection

bisects the base

48)

of a

of the

of the triangle.

(Agra 59)
Taking
be

b and c

Now

as origin let the position vectors of

respectively.

JfM

is

parallel to

BC.

B and C

Centroid, Lire and Plane

AM

AJf

"

AjrAu=*^

Therefore

vectors of

position

and

are

*b and xc respectively.

BM

to

Equation

0b+*.xc

r=(l

is
(1)
?

CJV

to

Equation

Fig.

No -46

(l-s)c+s.xb
sx
and !-$=*#
1-*

1-;

Comparing

Z>

is

or

Hence

the point

Equation

to

AO

Again equation

given by

is

is

.-

(b-f c).

r=

to -4Z)

where

Z)

is

mid.

point of

r=i(b+c)
Relations

(8)

and

(4)

show that

BC

is
(4)

/) lies

on

AO

produced*

If through any point within a triangle, lines be drawn


sum of the ratios of these lines to the corres-

(b)

parallel to the sides, the

ponding sides

(Agra 51, 61)

is 2.

Take the vertex C


origin let the
tors of

as

position vec-

A and B be a and b

respectively

and

that

within the

point

of

be

c.

Stoce
therefore
of

P be

if

/a,

PQ

is

to

\\AB

position vectors
then then that of

Vector Analysis

74

will

P
AD

be /band also Q=*l .................. (1)


let

Again

LA=(\-m)a..
is drawn parallel

:.

Now

in

&ABC,
.

LM
LM

RS^RBn

x- i

RS

to

CA

be ma.

BC.

on

CB

be nb

fl=(l-n)b.

.'.

Now

on

LA

Similarly let the position vector of

Its

vector of

the position

LT~

""""

-i

yj)

-*-

PQ, passes through /a and /b.

equation

is

r=(L-/)

passes through

i.e.

/a-(-//b

nb and

............
is

parallel to

CA

(4)

a.

i.e.

r^=nb+ja ............ (5)


iW passes through L t. e. ma and is parallel to CB i.e. b.
Its equation is
r*=wa+ttb ............ (0)

Its

equation

Since
therefore

is

the three lines

all

(4),

we have on comparing
(1

(5)

and

Now sum

- /) l=s, tl^n

of the ratios

from

0,

and

(5)]

the coefficients,

[from

H=M, s=m

and

at

intersect

(0)

[from
(1), (2)

and

(4)
(5)

and

(6)].

(3) is

Hence proved.
Ex. 23.

Prove that the sides about the equal

angles

of equi-

angular triangles are proportional .

Let us consider two equiangular triangles ABC and


have to prove that

ADE having a common vertex at A. We


AB^AC^BC
AD AE DE'

__>_

Let

AB**c, BC**a, CA**b.

Centroid, Lire and Plane

AD

EA

Kg. No 47

Again

- *a (a+b)+*
1 m+ t t b=

or

[from

(1)]

or
(2)

Now we know
a

exists

relation

from
of

non-collinear vectors,

5 page 16
chapter I
the form ^a+^b-^0

*0, j=0.

then

that

if

there

between two
Hence from (2),

\ve get

or

AB
Ex. 24.

=^ CA
BC

/A a parallelogram

side

AD,

AC

in the point

such that
.

n.AP^AD.
Prove that

ABCD
The

(n+ 1)

a point

tine

BP cuts

b,

c,

vectors

as origin let
be the position

of

B,

C and D
and

respectively

--*>

since

we have

the diagonal

AC

AQ^AC.

(Lucknow
Taking A

Hence proved*
P is taken on the

B. Sc. Supp. 48)

Vector Analysis

76

b+d-c ............. (1)


"*

AP=

AD=*

.d,

AQ**x.AC

Above

is

(I),

say.

fz

-*

Hence from

"*

we

get

J
_.

-*

AQ=Q.

between the position vectors

relation

three collinear points;

AB+nAP-

of

hence sum of the coefficients should

be zero.

or

Ex. 25.
of the same.

D,

E F
y

ABC is any
AB BO, CO

triangle

meet

respectively.

Ex. 26.

and

the

any point

sides

CA

BC,

in

and

the plane

AB

in

Prove that

(a) Prove that the points

a-2b+3c, 2a+3b-4c, -7b + J0c

are collinear.

Let the above points be A, B, C respectively relative


Now if we can find three scalars L, M, JV
to any origin 0.

---

such that L.
then A, B,

we

OA+M. OB+N. OC^O

find that 2 (a

where

21

where

Choosing L=2,
- 2b-f 3c)~ 1 (2a+3b - 4c)-

are collinear.

1=0.

Hence

collinear.

Alternative.

2a-10b+14c.

I+Af+JV=0,
- 1, JV= 1,

- 7b+ L0c)=0,

77

Centroid, Line and Plane

->

We observe
(b)

3a-2b

i-2j+k

Ans.

B (8, -1,

are collinear.

the vector

54,

Agra

55, 58)

line joining the

of the

equation

fc,

3k- 2j.

r(i-2j+k)+/(i-2k),

Stoie;

(b)

and

whose position vectors are a,

(Delhi 50, Banaras

Find

(a)

B, A,

.*.

the points

that

are collinear.

Ex. 27.
points

BC=24C.

that

Prove

#za*

Zine

the points
--

joining

the line whose distance from


(14, 1, 5)

Ex.28*
4c and

dwd

Jj

is

14.

and (-10, -7, -7).

Prove that the line joining

the line

-3, -

(2,

-^=1^ =$--. Find two points on

2) has equations

Ans.

joining

-a

2b

6a-4b+^c,

the points

3c,

a+2b

5c

intersect at

-4c.
Ex. 29.

In the triangle

AB
sides CA and
AR=2RB. BQ and CZ

'the

respectively
intersect

Also find the ratio in which

Let a, b, c be
vectors

position

divides

such

0.

and

taken

CQ^QA

that

Prove

are

that

in

and

CO=30R.

BC.

the

of

points
pectively so that

AO

at

the

B and C

A,

ABC, points

res-

under

given conditions the posivectors

tion

C
are

J&

of

and

and

Q,

-^
o

c4-a

Equations to BQ,

Equation to

Now
we

get

(1)

and

CR

(2)

is

is

r(l-*)b-M -^- ......... (1)


2b+a
- -~r-(I-,s)

intersect at

c+*

..

...

.(2)

and hence on comparing,

Vector Analysis

The

position

vector of

therefore g

is

+}(c+ a)

=|(a+2b+c).

Now

should be

vector of
is

divides

if

C# in the
q 2b+a

3:1, then the

ratio

- 8"""
g -*

or

position

(a+2b+c) which

true.

Again equation to AO is r=(l-Oa+*.J(a+2b+ c).


Since they intersect at P we have on comparing,

LH-^O,
From

first

1-1-., i-..

two we get /=f .,y=J and these

satisfy

the

third relation also.


/.

P divides BC

If

of

position vector of

Comparing,

in the ratio

-- =i

divides

Ex, 30.

-4

BC

and

then

1,

-=f

the ratio 1

in

linejrom a vertex of a

It intersects a

side.

is

position

vector

^-nr

is

/.

similar

line

// a, b, c

whose equations are T**(-

be

m=^.

2.

triangle- bisects the opposite

issuing

Prove that these lines intersect in the ratio 3

Ex* 31.

/.

from
:

the

oilier

vertex.

non-coplanar,

show that the

lines

J0a+b-cj-H

are coplanar> and find their point of intersection.

coplanar, then they must interwhich we should get identical values of r. Comparing a, b and c in the two equations we will have three
relations in s and /.
Solving the two we find that the
If

sect for

the above lines are

Centroid, Line and Plane


third

is

also satisfied

of intersection

Ex. 32.
which

is the

and hence they intersect and the point


7c.

Prove by vectorial methods the equation

equation of a straight

on the axes of reference.

6a+5b

obtained as

is

79

line

in

terms

oj

--(-:=./

its

intercepts

The axes may be rectangular or oblique.

Let the position vectors of


and B be a and b respectively

and their modules be a and

b,

a=0.a and b=i,b.


Let P be any point (x, y)

so that

on the

parallel to

and

From P draw PM

line.

OM =x

OB, so that

MP=y.
.a and

Now P being

any point on AB,

Also
a

Comparing a and b

Eliminating

t,

we

in (t)

get

and

(2),

we

get

+ ^- = 1 as the

required equa-

tion.

The straight line through

33

coplanar edges of a
point,

P such
hedron.

to the

that

the

mid. points

of

tetrahedron, each parallel to the line joining

three

ajixed

mid. point of the opposite edge are concurrent at a point

OP

is bisected

by the centroid

(of volume)

of the tetra-

(Agra 46)

Vector Analysis

80

Take
the

and

as

be a, b,

points

c,

respectively.

Let P 1 and

let

A B C

vectors of

position

and

origin

be the mid.

opposite edges, so that

of

their position vectors are

b+c

a+d

and

Now

the equation of a line

through

P l and

Similarly

through Sj

i.e.

parallel to

OT

we can

write

c+dand

pirallel to 0/? a

The equation

00

of the line through

where

is

Now
we have

(1)

and

down

(2)

a+c
-- x

Again

intersect at

is

a+b
is

"t

e.

as

.(2)

'

b+d and

para-

as $ (a+b-j-c-f-d).

satisfies (8) also.

lines are concurrent at P.

OP is

the cetitroid of volume


is

i.

#x

hence on comparing

and we get the point


we choose />=*!, the point

mid-point of

Hence OP

fcj

P and

HO+Ha+b+c+d)]~
which

where

b+d

Hence the above three

Now

equation of line

is

/=5 = 1
if

the

2V 2;/"

lei to

is

bisected at G.

a+b + c+d

of the tetrahedron*

Centroid, Line and Plane

If a*)* are two

4
points

/.a-f^ab,

L-ftf'ib,

non-collinea)
/s

81

vectors,

a-rff*3b=0

are

show

collincar

that the

and

if

only if

wg

rij

rn$

(Agra
If the

given vectois are collinear, then

where

we must have

x+y+z^Q

(1)

18

or

(xl

+j>/2 -f c/3 )

Since a and
there

/=0,

exists

m*0

two

are

5 chapter

of
1,

x, y,

non-collinear

the

if

t] ien

-0

hetwen

type

vectors and

/a+tfzb=0,

P. 1GJ

*/i+jtf|+-rf 3

Eliminating

P. 48]

a + (*m i +J^2 + ^2) b = 0,

a relation

or

7.

(2)

(2).

and

(U)

(1),

we

get

the

required condition.

Vector equation of a plane.

$ 9.

To find
the origin

tin vector equation

and

ts

parallel to

as origin

Choosing
the vectors

OA

and

OB

of a

two given

in

which passes

a and b.

let

be a and

P be "any point on the


From P draw PM and
plane.
PJf parallel to OB and OA
resOB
and
OA
pectively meeting
b.

plane
vectors

Let

M and JV respectively.

OM bemg collinear with

Fig.No.52

through

82

Vector Analysis

ON being collinear with


If

then

r be the

OB^tb.

position vector of

P any

point on the

plane,

r=OP
Hence the vector equation

given

by

where

and

of

the

required

plane

is

P moves on

are scalar* which vary as the point

the plane.

Note.
Cor.

5 P.

To find

i.

c and

given point

of

See

Iti

I.

Chapter

the

vector

parallel to

a and

equation

of a plane through a

b.

[Agra 40]

Let c be the position vector


any point C on the plane and

P be and

point on

it,

Now

vector

GP

b and

as such CP=ja-|-/b.

is

coplanar

with

(5
If

the

a and

chapter

r be the position vector

fiy.Nc.53

P. 15).

of P, then

Hence the required vector equation of the plane is


r **c -Ma *Hb wh^re s and t are scalars which vary as the
the plane,
point P moves on
Cor.

a.

To find

through three points

the vector equition *of a

whose position

plane

that

and

c.

of

the

vectors are a,

passes

(Agra 39)
Leta,

S and C

b,

cb2

the

position

vectors

respectively on the plane, so that

points A,

^4Bb

and

83

Centroid, Line and Plane

Now
one

the

the

through

AB

parallel to

a and

required

or

is

and

through
and c-a

i.e.

b-a

to

and hence by Cor.

point

and AC,

parallel

plane

1, its

equation

fiq.No>54

is

r*(l-$-J) a+sb+te..

Cor. 3. To find the vector


points a and b and parallel to c.
If

a and b be the

..

....... (3)

equation of a plane

position vectors

of

through

A and B

two

respec-

tively,

^B=b-a. Hence

then

passes through a and


Cor,

1, its

equation

is

the required plane

parallel to

b-a

is

one that

and c and hence by

is

r=(l-j)a+jb+fc ........... (4)

or

Cor.

4.

To find

the

equations of planes found

Cartesian

equivalents

of the vector

above.

(Refer author's Solid Geometry.]

Case
given

i.

Plane through origin and

parallel to the

line.

Let

in

terms of unit vectors

i,

and

through origin

G, the vector
so that

is

(x,y, z).

so that direction cosines of vector

and

baa^i+taj-f *k

are proportional to <Z], a a , a 9 ,


so that direction cosines of vector b
are proportional to ov b tt b+

(a)

Now

vector

equation of a plane through origin

84

and

'

Vector Analysis

parallel to vectors

a and b

is

or

Equating

coefficients of

Eliminating

-5 and

-/,

ai

bl

i,

and k, we get

we

get

=0.

or

Above

is

the

cartesian equation

coi responding
arid

plane through origin

to

parallel

cosines are proportional to a^ a 2

lines

and

whose

b l9 b 2 , b 3

of

direction

Case a* Plane through a given point and paratwo given lines.

(b)
llel

to

Vector equation of a plane through a point and parallel


to

two

straight lines

is

r=sc-f Ja+'b
or

[by Cor.

Ij

(r-c)=jfl+/b.
Putting in terms

cients of

i,

unit

of

vectors

and k, we get

Eliminating

^,

/,

we

get

=0

aa

ai;d

equating

coeffi-

Centroid, Line and Plane

or
*2

*2

*3

Above

is

*3

corresponding cartesian equation of a

the

plane that passes through the point (c^ c2 c 2 ) and is parallel


to two lines whose direction cosines are proportional to
,

a v a * a s an d b l9 b 2 b z .
(c)

Case

Plane passing

3.

through three given

points.
Vector equation of a plane through three given points

r=(l-,y-/)a+,yb+Jc

r=a + J (b-a)+f

or

is

[Cor. a]

(c-a).

Putting in terms of unit vectors and equating coefficient?


of

i,

and k, we get

Eliminating

we

/between the above equations,

and

get
=0.

#3

The

bz

a$

r3

above determinant can be written as a fourth order

determinant as following

Vector Analysis

Adding fourth column to

first,

second and third column,

we get
x

bl

al

cl

or

1111
or

Above

the

is

a plane -through
(fj, C 2 ,

10*

corresponding cartesian equation of


three points (a l9 a 2 a 3\ (b^ b^ fc a) and
,

Condition for four points to be coplanar*


that the

necessary

and

sufficient

to be

three-dimensional space

points in

exists

=0.

C3\

To prove
four

coplanar is

a linear relation connecting their position

the algebraic

sum of the

coefficients in

it is

condition for any

vectors

that

such

there
that

zero.

(Pb. 60;

Agra

37, 54)

We have

seen that the vector equation of a plane


through the points whose position vectors are a, b and c is

The above
Above

(l_,y_f)

relation

may

a+jb+'c0.

[Cor. a]

be written as

a relation between

the positron vectors of


) which
are coplanar and we
points A, B, C and
observe that algebraic sum of the coefficients of a, b, c and
r is 1 J /+$+* 1 which is zero. Hence the condition
is

four

is

necessary.

Centroid, Line and Plane

87

In order to prove that the condition is sufficient let us


suppose that any four vectors a, b, c, d be connected by
the relation Ja-f;wb-f-;ic-|-/>d0 where

we

Dividing by p (p^Q),

m
+-r +

where

Above
a,

/>

~s

relation

b and

c;

~-i.
,

and

=/,

'++

shows that d
hence

'-;-;

through

Putting

get

_L a _.. b _

d==

/+m-f n+p=*Q.

a, b, c,

is

a point on the
are coplanar.

plane

Alternative Proof.
Let us suppose that the points A,
tion vectors

are a, b, c,

CD

at

not parallel and

if

intersect

are

ratio

then

its

q and

CD

position

written from

and

coplanar

whose
let

posi-

AB and

being assumed that AB and CD are


they are, then we will choose any other

P divides AB
:

(it

pair of non-parallel lines formed


If

B, C,

in

by given

points).

^o

in the

'

n,

vector

AB and CD

A~

"

wi^^-^^P

is

Fig.No.5S

m+n

m+n

or

where
Hence the condition

is

necessary.

88

Vector Analysis

From

Note.
point

d on

here

we

find

the position vector of any

the plane through a, b, c as

Lm+Afb+Afe

Lm+Afb+JVc

Converse*
Again

we

will

Now
is

let /a-f

show

of the

not zero for

/.

Also

wb + flc+/>d=0 where

that the points A, 5, C,

if

m+fl+/>=0 and

n-p=Q or
/+w+n+>=0 but
;H

w=0,

;=0=m = w=:j&

Let us suppose that

From

/-f

are coplanar.

three scalars l+m, /+, /+/>, one at least


all of them were zero then

or
/.

which

l+m

the given relation,


/a

/?=*;

+ mb

is

/.

^==.

impossible.

not zero and therefore

is

we

get

"""

/+^i

is centroid of points A and B with associated


and TW and hence is on AB, and R. H. S. is centroid of C and D with associated numbers n and/*, hence is
on CD. Hence we find a point on AB is same as a point on
CD showing that they intersect and hence A 3 B, C, D are

L. H. S.

numbers

coplanar.

AB and CD

Also the point of intersection of

Ja+mb
l+m'

nc+pd
n+p

'

is

Centroid, Line and Plane

89

Exercise
Ex.

//

i*

within a

any point

and AO, BO, CO,


faces in P, Q,, R> $, show that
to the vertices

DO

tetrahedron

OPOQ,OR OS_
i

(Agra 33,
as

Taking

is

joined

35, 39, 43, 46, 49, 58, 61)

the

let

origin

position vectors of A, B, C,

ABCD

are produced to cut the opposite

be

a,

fi

respectively and we know


that any four vectors are linearly

b,

c,

there

i.e.

dependent
tion of the

exists a rela-

form
Fig No 56

...(1)

r=s

Equation of AO is r= a and hence equation to OP is


ksi where k is a scalar
positive for points on AO pro-

duced and

ive for those

But from

(1),

on OA.

we have

mb+nc+pd
^

Putting for a from above the

equation to OP\

is

given

(mb+nc+pd)
Again equation to plane BCD is
r=(l-,y-f) b+5c+/d [Cor. 2]

Now

(2)

and

(3)

intersect

at

JP

*?

...

...(3)

and hence we have on

comparing,
^m

.(2)

*A_

90

Vector Analysis

Adding,

Hence
A

y(m+n+^)l

vector of
position
1

k^-

or

4P

...... (iv)
---

is

a.

m+n+p

>

Aga.n

OP
AP

----;/

Proceeding as above,

we have

l-i-

n
m
OR_
BQJ*l+~m + n+jr ~CR^l + m + n+p'

0(1
'

OS
and

T^c-=7
,~
;
D6
l+m + nrp
,

Adding, we get the required result*


Find the equation of the plane through the origin and
Find also the point in which this plane
points 4j and 2i-fk.
2.

the

is cut

by the

2j+k

joining the points i

line

and

5k

(Agra
Equation of plane
Equation

of line

For point
find

that

of

f|,

is

is

r=j 4j-H

r=*(l~/>)

2j.

42, 45, 56)

(2i+k).

(i-3j+k)+/> (Sk-2j).

comparing and solving, we


and />=~5', an 1 hence the point is

intersection,

j-i

K6i-10j+3k).
3.

Prove

3&+4b-2c
ist

that

and

four

a,6b+6c

Method.

respectively.
intersects the

the

We

points

2a+3b-c,

a-2b-f-3c,

are coplanar.

Let the given points be A, B, C,


will

prove that the lines joining any two


the other twxo provided they are

line joining

not parallel and hence the four points are coplanar.

91

Centroid, Line and Plane

We

find that

/.

let
.

AB= -2CD

AB is parallel
AC and BD say.

i. e.

us consider the lines

to

CD.

Equation to 4C is r=2a f 3b-c+/ (a+b-c).


Equation to BD is r=a-2b+3c+.y (-4b+Hc).
If they intersect, we have on comparing,
Solving the first two, we find t=* -1 and s= 1 and
these values satisfy the 3rd equation as well. Hence the

two

lines intersect showing thereby that the four points


are coplanar and the point of intersection is a-f-2b.

and Method.

AB=* - a- 5b+

Ic,

and
the given points are coplanar, then the three coter-* ~
should be coplanar and as such
minous vectors 4S, ^!C,
If

AD

they should be linearly dependent, t. *., there must exist a


relation between them.
Let us suppose that

or

/(-a-5b+4c)+w(a-hb-c)=-a-9b<+-7c.
Comparing; we get

-/+:= -1, -

92

Vector Analysis

we

m=l

and these
-* the third relation 4?-ro=7, and hence AB, AC

Solving the
values satisfy

first

two,

get /=2,

AD are coplanar.

and

coplanar.
Prove that
4.

- 4c and

Prove

5.

D are

6* +2b - c, 2a - b+3c, -

a+ 2b

the points

- b - 3c

72a

Therefore the points A, B, C,

that

are coplanar.

the

Find the point

6. (a)

is

The

equation to the

Find

(b)

(4,

are coplanar.

in

which the plane

cut by the line through the point 2a-f-3b and parallel to c.

of intersection

~ a -Hb-3c, 3a4 2b-5c,

points

3a-f #b - 5c, - 3a+2b+c

1)

and

-1,
7.

line is

For point

r=2a+3b+jbc.
Ans.

2a+3b

4c

the intersection oj the line joining the points (1,

compare

(2, 3, 1)

etc.

with the plane through the points

Prove that the

(2, /,

Ans*

2) and (3, 0, 1).


six

planes

containing

one

-5J,

(f, |, J)

edge

and

bisecting the opposite edge oj a tetrahedron bisect each other.

Plane

containing

and bisecting

BC

i.e.

OA
plane

OADis
IS 9]

Plane

OCF

is

Plane

PBC

is

r-(l-Js-W J-+*sb+/c
4
Compare
intersection.

the coefficients of

9 Cor.
a,

1].

b and c

for

point of

Centroid, Line and Plane

.i.!- V-'s

"2

Hence

'

'i_

the point of intersection

Similarly

we can show

_*

'

7"""2

'i

8'

a+b+c

is

for other planes.

Prove that the middle points of the six edges of a cube


7. (b)
which do not meet a particular diagonal are coplanar.

A be

Let

and AB 9

AD

taken as origin

and AA' as axes

along which the unit vectors


be i, j and k
respectively.

Let the edge


Q/,

are the

the edges on

marked.

The

cube be

the

of

of unit length. P, Q,,

R and

P',

mid. points of

which they are


position

vector

of these mid. points are

R^

pj.af
P'

(i+i+kWj+k)_
"
2

Similarly, Q,'

is

'

i+j+k,
i

",

2 tj

i+4 +k

and

Now the

equation to the plane

PQR

is

...(D

The

,A

3
point P' -^--f-j+k will

the coefficients,;

lie

on

it,

if

on comparing

94

Vector Analysis

]-,-/!,

,+*-!,

Solving the last two, we get


values satisfy the first also.
Similarly,

we can show

^-1.

/=*2, 5s=s-2,

that for

some

and these
values

suitable

and /the other two mid. points Q/ and


will also lie on
the same plane. Hence P,
&, #, />', Q/ j?' are coplanar.
8.
If a, b, c be any three non-coplanar vectors, then
prove that the points ^a-f T^b+WjC, / 2 a+m 2b+w 2 c, / 3 a+w 3 b
of s

/4

a+m4b+w 4 c
;

are

*s

/i

coplanar if and only if

/,

/4

ll 11
the given vectors are coplanar, then there
a relation of the form
If

+w
where

(/ 4

must

a + w4 b +

exist

4 c)

x+y+z+w=*Q.

= 0,

...(1)

b
Since a, b, c are three non-coplanar vectors and if
/a+^b+c=sO, then
5 Chapter 1, P. 15]
/w=/z=0,
L

there exists a relation of the form

Q.

0.

Eliminating

we

x, y,

^ and

M;

.(2)

. .

.(4)

between

(2),

(3),

(4)

and

(1),

get the required result.

Therom
9-

Vp

Q,/

Rw*

respectively of a triangle

three

ABC,

of Ceva
points

on

the sides

such that the lines

BC, CA,

AP,

BQ^

AB

i>il

95

Centroids, Lihes and Planes

BP
-

CR

are concurrent, then

Let

AR

Cd
~~

^-

-1

<wd conversely.

be the point

BQ

of intersection of AP,

and the position vectors


of A, B, C,

be

a, b, c,

Since these

respectively.

four points are

coplanar,

where

xa-

- (& + wd)

xa

and

point on AB and R. H. S.
and both being ame, either gives the point
and CD and hence R.

The
on
to

CD
AB

L. H.

Thus
troid

thus

S. is

position vector of

which

is

is

a point

common

is

the cen-

x+y

A and B with associated numbers


divides AB in the ratio y x.
AR _. y r\r AR
y
of

x and

and

_,

RB~

c-

-i

BP

Similarly

'

x
z

-~^.
r
ux

AR
BR

z.

i
and

BP
CP

'

BRCQ *-r^

^*><,

Cd^_
^(^^

x __

~lc'~y'~z~~~
Hence proved.

Converse.

We

are given that

AR
BR

'

BP
CP

'

and we are to prove that AP, BQ, and C/2 are concurrent.

98

Vector Analysis

Let us assume that 7^5*=


t
/n 4jR
from(l),

and

-- and hence
z

A\^

j3#=- ^
From above, we

get

Suppose I) is a point on -4P dividing


x and therefore its position vector is

it

in

the ratio

or

x+y+z
The symmetry

also

lie

of the

on BQ, dividing

dividing

in the ratio

it

Hence the three


10.

AB

//

P,

it

x+y

be

this point will

:y and on

CQ,
~?Z

sides

BC,

BQ

and

the position
of A, B, C,

as a, b,

c,

AR

"?<,

,
and
"&=**
DJK,

the

CR and
vectors

D be taken
d

respec-

CA and
R are

such that the points P, Q,,

point of intersection
of

CR

z.

ABC,

BP
Lti

z+x

lines are concurrent.

collmtar, then prove that -~~

Let

shows that

ratio

Theorem of Menelaus
Q, R are three points in the

of a triangle

respectively

result

in the

Ftg,No.59

tively.

Since these four points are coplanar,

x*+yb+zc+wd**Q where

we have

x+y+z+weQ.

97

Centroid, Line and Plane

the position vectors of Q, and


respectively can be taken as

Just as in Q.

and

AB

9,

"

'

x+z

R on AC

(1)

(2)

Subtracting,

we

get

(*

Dividing both sides by c

q- (x+y)
we

j,

get

L. H. S. gives a point on the line joining points whose


position vectors are q and r z. e. on RQ, and R. H. S. gives
a point on the line joining b and c, /. e. BC. Since they are
equal therefore L. H. S. or R. II. S. gives the position vector

BC and RQ.

of the point of intersection of

Now

from

(1), (2)

_
Q_A~

x
'

and

we

(8),

AR_y am
RB~"x

of P.

get

B?
PC~

,
'

,BP_z AR_

i. e.

CP~ y' BR~~ x' AQ_


BP CQ AR
"
.

CP

Converse.

We

'

'

AQ_

are given that

AR

CQ,

CP

Ad

'

BR~
CO

np
Let us suppose that

AR

Bit"' x'

M~

^SBLI r

and -7^= ---- and hence


*

98

Vector Analysis

BP_

Hence

if

Now

p, q,

AR

x
'

r are position vectors of


respectively, then from above ratios,

if

relation of the

where

p, q,

P, Q,

r are ollinear then there must

and

exist a

form Lp4-Mq-|-JVr=0, such that

- fe -y) +(x +^) -

(x +y>)

=0.

Hence

collinear.

CHAPTER

III

MULTIPLICATION OF VECTORS
There are two

i.

different

ways by which

vector

quantities are multiplied : one is called scalar or dot


product and the other is called vector or cross product*

The former

is

a mere

whereas the

tion

number and does not involve any

later

is

direc-

ass3ciated with a definite direction

and as such

is a vector quantity.
However, in each case
the product is proportional to the products of the lengths of
the two vectors and they also follow the distributive law

dot

of

product

two vectors a and b

(Agra
The

Def.

tlte

i,

e.

axb

i.

e.

b.

Scalar product.

a.

and

a b

the vector or

written as

is

X between a and

scalar or

written as

is

b whereas

by placing a dot . between a and


cross product of the vectors a and b
by placing a cross

The

numbers.

just as in the product of ordinary

scalar product of two vectors

respectively

directions of

is

and

Again a

b=0

b cos

to

equal

(b

is

ab cos

a and b

where 6

is the

of moduli a
angle between

Thus a . b~ab co

Now

32, 33, 40, 44, 51, 57, 58)

cos

.
|

cos

0.

0).

of

length

the resolved part OL of OB i. e.


module of vector b in the direction

or

of

a whose length

we can

write

a
where a cos
direction of

it

b=/>
is

is

as
(a

cos

Pig No<6o

0)

the length of the resolved part of

b whose

length

is b.

in

the

Vector Analysis

100

Hence

of two vectors

dot product

equal to product of the

is

length of one of them with resolved part of the other in

direction

the

of the former.

Again if we take that b represents a force in magnitude and direction whereas a represents a vector drawn in
an assigned direction, then
the force
the

work done by the

cation from

to

force

Thus

in

its

moving

OA and work

along

of

point

appli-

doe? not involve

hence dot product

and

the idea of a direction

the resolved part of


ab cos
represents

b cos 9 is

in the direction of a.

is

a scalar

quantity.

Properties of Dot product of Vectors


!

From

above

we

find that

b=b a=a&

cos 6.

Jrhnce scalar product is commutative.

cos 6=*

or

ab
2.

The

a
a

dot product of two vectors will be -\-ive, zero or

according as cos 6

is -\-ivr,

zero or

ive

ive

which means that according

as B is acute or a right angle or obtuse.

Again a
3.

Jtow

\>ab cos 0.
a b will be
.

zero

only

if either a or b or cos 9

is

zero.

Hence dot product of two


be

a zero

vector

or

vectors is zero if either

if they are non-z*ro vectors

tions should be perpendicular.

of the vectors
then their direc-

[Remember],

be the angle between the directions of a and


Again
be the angle between the directions of a
ir-Q
will
b, then
and b or of -a and b
if

Thus a
or

(w-0)(-a) (b)
a (-b)=(-a) b ~ab cos 0-(a
.

(-b)

aft

4., fn

dot pfo'duct

cos
,

b).

of two vectors if either of the factors

is

Multiplication of Vectors
by minus sign,

multiplied

minus

then the total

101

product

is

multiplied by

sign.

Again if be the angle between the directions of a and


then
since vertically opposite angles are equal,
is also
b,
the angle between the directions of
a and b. Thi\s

(a) (-b)=0
of vector

cos

(a

Again

b).

a then vector ma

be

if

a vector

is

the

module

the direction of

in

but of length ma.

Thus

(wa).(ttb)=(w0)

(nb)

cos Q=*mn(ab cos

=mn (a b)
=0 (mnb) cos 0=a

8)

or

or

=(rzfl) (mb)

Thus

5.
If
i.

e.

the scalar product of

the vectors

be

their

the vectors be

written as a2 =fl

T&MJ

the

vectors

0=180,

i.

is

e.

equal to the

cos

0= -1.

a.b=-0*.

/.

equal then a

a=fl

cos 0=fl 2 *and

2
.

square of any vector

is

equal to

square

of

its

(Remember)

In case

there be

are each unity.


8.

is 0,

b=0fe.

In case they be unlike then

module.

vectors in associative.

of two like

the product
moduli.

77*1/5

product of

7.

wb,

then angle between them

like,

/.

If

= 1.

cos

6.

two

(mrcb)

cos 0=fla

Thus

two
.

the product of two

then their moduli

vectors

unit

A A
Thus a b = l

cos

0=cos

0.

unit vectors is equal to the cosine

of the angle between their directions.

Orthonorinal vector triads

We know
unit vectors,

that

i, j,

i, j,

k.

are three mutually perpendicular

3)on<*td by

102

Vector Analysis
i

/.

and

=j*k

jj

Distributive

9*

kk

Law

l (Properties 7)
.

i=*0 (Property 3).

i. e.

(b+c)=a . b+a

c.

(Punjab 60)
as origin let

Taking
->

-*

and

04,

BC

the vectors a,

represent

b and

res-

->

OCb+c.

pectively so that

BM
OM

and CJVbe perpendiculars from B and C


Again if
on OA, then
and OJV are the projections of OB and OC
on CM and hence
is the
projection of BC on 0-4.

MN

Also

OM+MN=*ON.

Again we know that dot product of two vectors is


equal to the product of the module of one with the resolved
part

e.

i.

.'.

projection of the other on the former.

(b+c)=a . (OC)fl
=(2

OAW (OM+AfJV

OM+a MjV= a
.

b-f a

c.

(b-c)=a [b-h(-c)]*a b+a (-c)


=a b +[-( c)]=a b-a c.
By repeated application of above, we obtain
(a+b) . (a-b)=a . a-fb a-a . b -b .b
= aa -b 2 V a.b=b.a.
Similarly

For geometrical interpretation of above see Q. 5 next


exercise.

(a-f b)*=:(a4-b)

In general (a

==a

.(a-hb)a

+b+c + ...)
p+a q+a

af b a+a b+b . b
.

(p-fq+r +-.,.)
r

-f-b

p+b q+b*

-fc.p-fc

q-f

c.

rf ...

103

Multiplication of Vectors

Again

a and b be expressed

if

terms

in

vectors

of unit

as

and

b^i+iaJ+baJk,

then a

*V(V+V+V)

.'.

b^a^+a^+a
ab cos

or

V
COS

Or

i'j*~k':l

and

j-j

k-k i0.
.

(Prop. 8}

fla

n
Note.

(al9

a& a z ) and

A and B

the points

if

l9
Again
a and b, then

of

/i

10,

n*

respectively.

m ^ and

l9

55

are the coordinates of

b& b^

(4 lf

m!^=

Z2 ,

m2

nz

be the direction cosines

^=5-^ and k

etc.

cos tf/1 /2+m1 in2 +n1 n2.

.%

Components of a vector

[From

any vector r

is

inclined at

(1)]

(Imp.).

(Delhi 50,
If

an angle B

Lucknow

52)

to the direction

A
of

a and a be a

unit vector in this direction then the resolved

A
part of r in the direction of

is r

^
_._..*__
ar cos

(r

a)

cos

<l>

a)
(r
m--.a,
mt
.

V aaa.
(Remember)

Again

a and

of

r be resolved into two components in the plane


one parallel to a and the other perpendicular to

if

r,

then these components are

-V-

a and

r- -A-

a.

You can

write

a2

for a*

'!

Vector Analysis

104

Similarly if a vector r be resolved into components


2
since i
j a=sk2s=sl
parallel to unit vectors i, j and k, then

these components are


.

(r

i) i,

(r

j) j, (r

We

Alternative.

(Remember)

that

any vector r

k)

know

expressed in terms of three non-coplanar vectors


let us suppose that r=xi+jj+* ...... (1)

k and

and

kk
/.

i=x, r

.j>,

Putting the values of x,y and z in

and

5 P.

1ft]

noting that

Mulplying successively by i, j
2
. i=0 and i - j*-k*--l.
i . j *j .

can be

i, j,

(Property

8)

k=<:.

(1),

we

get

r be expressed in terms of any three


there exists a relation
non-coplanar vectors a, b and c, then
iflj,

if

Again

between them

i. e.

r=*a+jb+sc

............ (1)

Multiplying the above relation scalarly by a,

we

successively,

get

b and

5 P. 15)

a=*a a+?b a+^c a ......... (2)


r .b xa b+jb c+zc b ......... (8)
...
...('!)
r c=#a c+jvb c+^c c ...
r

Now a

a . b and c b etc. are scalars. Hence


z between (1), (2), (3) and (4), we get
.

a,

minating x,y,

eli-

0.

Examples
Ex,

i.

Prove

that

in

a tetrahedron if two pairs of opposite

105

Multiplication of Vectors

show that sum of the squares on two

(Lucknow

each pair.

Taking
the

vectors

B and C

A,

same for

opposite edges is the

49,

Agra

Utkal 52)

389 53,

as origin, let

position

points

Also

third pair are also perpendicular.

edges are perpendicular, the

of

be a,

and c respectively.

We
AC

is

are

perpendicular

and we are

to

Fig.

Ho

DB and AB is perpendicular to C;
DA is perpendicular to BC.
b (c-a^O cr b cb a...(l)
/.

to

prove that

AC 1 toZ);

that

given

dot product of

two perpendicular vectors

is

zero.

JBltoDC;
Now a b=b
.

/.
.

(b-a)=0or c. b=*c

a; therefore from

(1)

and

(2),

a.

we

...(2)

get

a.b = b.c=c.a .......... (3)


From ahove we deduce

DA

is

perpendicular to

that

showing that

(b-c)

Hence proved,

CB.

Again

square of a vector

*/

is

module.

=a2 +ba -fca -2a b


b=b c=c a from
.

Ex.

which

equal to square of its


(Property 7)
is

symmetrical

because

(3).

2,

Prove that

sum of

the

squares

on the edges of any

tetrahedron
(i)

is

equal to four times the

of the mid-points of opposite


(ii)

is

equal

sum of

the

squares on the join*

(Agra 51)

edges.

to four times the

joining the vertices to the centroids

sum of

the squares

of opposite faces.

of the

lines

106

Vector Analysis

Ex.

Prove

3.

AB* + AC***BC*>

in

that

right-angled

Taking A as origin and the position vectors of


as b and c, we have b . c=0,
V Z. 4=ir/2 etc.
Ex.

Prove

4.

ABC,

triangle

being a right angle,

that

the

points

2i - j

+k,

and

- 3j - 5k,

are the vertices of a right-angled triangle.

3i~4j-4k

(Lucknow
Find

52)

also the other two angles of the triangle.

Either calculate modules of each and prove that sum


on the two is equal to square on third, or if

of the squares

-*

CA

the given points be A, B and C, then find A3, BC,


prove that dot product of any two is zero.

Again a

b=a6

and

1
cos 6 etc. cos*" 1 VrA')> cos- V(if).

(Use Prop. 8)
(b)
vectort

7/r=a+/b
that

prove

r=ss {a

(a

the

be the equation of a sf. line,


line

through

origin perpendicular

b) b} and length of perpendicular

to

it is

is

2
2
V{a -(a.b) }.

(Agra 4 1)

through Pa and

The

being a unit

parallel to
given line passes
unit vector b.
are to find the equation to the line
is

OM

We

which passes through origin and is perpendicular to it.


Note : In the figure take OM perpendicular to MP.
AfP= projection of OP a in
fr

the

direction

ba cos
/.

unit

p^

vector

8.

But a

module a

of

b0

vector
.

MP in

cos

6.

the direction of

b.

b)

unit vector

is

of

307

Multiplication of Vectors

-^

.->

MP=OP- OM

Now

OM0?-MP=a~ (a

or

Hence

r^s

(a -(a. bib}.

OM=V{*

/V000 //wf

5.

squires on the

the

the other upon it;

sides

is

to

equal

of
of

contained

rectangle

either

the difference

projection of the other upon

BC=*AC

AB*=sc

module

sum

the

of the

sauares on two

the

diagonals

these sides

the squares on

is four

and projection
two adjacent

diagonal and the

by either

>

diagonal

--

(b)

}.

it.

Let 4#=b, AC~c, so that

of

sum of

of squares on

difference

and

-(a.b)

twice the

is

diagonals

sides;

any parallelogram the

in

times the rectangle contained by

of

is

;.

adjacent

a line through origin

0.

i.

OM*=OP -MP 2

Also

Ex.

OM

the equation to

b) b.

Remember

b.

multiplied

Prove that a

that

^4Z)=b + c and

c=bc cos

by the projection c on

parallelogram

0,

i t e.

b, etc. etc,

whose diagonals are equal

is

rectangle.

Refer Q.

5.

AD**BC\

:.

AD*~BC*

4b

or

Ex*

6.

c~0,

If a straight

straight lines, prove that

Taking
coplanar lines

AB

i.e

line

it is

(b+c^c-b)2

-*

AC and
is equally

hence a rectangle.

inclined to three coplanat

perpendicular to their plane.

--as origin,

or

let

OA,

OB

04=a, OB=b, OC=c,

and

OC

be the three

103

Vector Analysis

Again

OZ)=d and we

let

are given that

OD

is

equally

inclined to all the above lines.


~

COS v
cos 6

b d

~f~j

ss

d
;~

cd

cos

cos

bd

-^,
^ ^

But

Hence the above

0=90 which means


lines

sss

aa

OC which

OA, OB,

hold good only

will

that

OD

is

in other

ABC,pwve

AB*+AC*=2

BC

0=0

i.e.

the three
it is

per-

of a triangle

(AP*+BP*).

Choose P as

all

words means that

pendicular to their plane.


Ex. 7. IfPbs the middle point of the side
that

cos

if

perpendicular to

origin and

JM = a and P/?=b,

so

that

Square of a vector=square of its module. (Property 7)


Ex. 8. Prove that in any triangle ABC,
3

(AB*+BC*+CA*)=

-9
where P,

Q,,

respectively

of

are the middle points of the sides

the triangle

and

BC,

CA

and

AB

is the centroid.

Use AB***AB*.
Ex.

where

9.

and Q, are

respectively.

(Property

In a qurdrilateral

the

ABCD, prove

middle points of

the

7)

that

diagonals

AC

and

BD

Multiplication of Vectors

Ex. 10.

Prove that the

109

middle point of the hypotenuse of a

right-angled triangle is equidistant

from

its vertices.

(Pb. 60)

Choose P the middle point of BC


as origin and let the position vectors of
A,

be a and b; so that of

PB=PC. We have

/.

-b.

is

to prove

PA=PB=PC

that

or

2b2 +2a 2 =4b2

or

Ex. n.
from

or

Prove that

(a)

(b-a) +(-b-.a) =(-2b)

or

b2

a2

in

any

or

PB a ^P^ 2 PC 2

triangle the perpendiculars

the vertices upon the opposite sides are concurrent.

(Agra 42,
Also prove that

the right bisectors

Let the point

of inter-

47;

Utkal

53;

Luck. 54)

of the sides are concurrent.

(Luck. 49)
section of altitudes BQ.

CR meet

at

and

and taking

this point as origin let the

position vectors of vertices

A,

and

AO
that AP

Let

be

a,

b, c.

produced meet
is perp, to BC.

4C*c-aand

BC

at P.

Then we should prove

Vector Analysis

110

BQ is

perp. to AC]

CR

perp. to

is

AP

is

;.

-vc

(c-a)=0;

(b-a)=0;

or

b and

middle points

Now OD

is

c,

----,

c,

.'.

BC and CA meet

be true

if

OE is

^-

~-

perp. to CA;

ib-a)=0,

Prove that the median

perpendicular

b2 -c2

OF
i.e.

is

c2

a2

also perp. to

b a =a 2 which

to the base

Prove that

the

of an

is

AB

which

true.

isosceles triangle

diagonals of a rhombus intersect

(Luck. 50)

at right angles.

;.

to the base.

Ex. 12. (a)

Taking

at 0.

perpendicular to BC.

Noyy we have to prove that

as origin, let the

of
position vectors

b and d,
we have b*=*d2

be

b=*b

the position vectors of the


so that the position vectors of the

F are

E,

>,

Similarly

is

b.c=c.a.

let

*+5.(c-b)-a

(b)

-Aa.(c-b)-0,

right bisector of sides

this point as origin

vertices be a,

will

.'.

perp. to BC.

Let the
Taking

-fib

a.b=b.c=c.a

.'.

i.e.

AB\

:.

and since
.

D
ABAD,

Also

and

Multiplication of Vectors

sd a -b 2 which

Hence AC
(b)
angled

// a

zero as

ba =d2

perpendicular to BD.

is

'point

triangle,

is

111

its

is

join

the vertex of a rightmid. point of the hypotenuse is

equidistant from
the

to

perpendicular to the plane of the triangle.

Let

a, b,

the

52, 54)

c^

c be the

vectors

of

right-angled

tri-

position

(Agra

angle.

or

(b-a) =(c-a) 2

a . b=a . c-fb . c-c*


(1)
Let p be the position vector of any point P which is
equidistant from the vertices.
or

/.

2p

.'.

(p-a) =(p-b) =(p-c)

(a-b)=a -b 2p . (b-c)=b2 -c2


2

2
2p.(c-a)=c -aa

and

(2)

or

DP is

or

We
for

perpendicular to BA.

should also prove that

which we must have

Now

p.(c-a)-J
C

or

DP

(a.

-^----!

(c

DP is

perpendicular to

L (c-a) equal to

^
^

zero.

c+b c-a2 -b.a)

-a2)=0

from

(1)

and

is

perpendicular to AB as well as
perpendicular to the plane ABC.

an

the diagonals is equal to

is

Ex* 13*

AC

Prove that in a parallelepiped, the

sum of the

(2).

AC

and hence

it

sum of (he squares

squares on the edges.

112

are

and

Vector Analysis

The

four diagonals

OP,

CD, AE, BF,

clearly,

(a+b+c) 2 +(a+b-c) 2

+(b+c-a)a +(c+a-b)2

= 4 (a +b +c
a

a2

Ex.

14.

a cube

a
)

04 2 =04 2

line

prove that

makes angles

cos

that the angle between

(Property

OL+COS* fi+cos y
two diagonals of a cube

let

cos 2
is

cos" 1 J.

->

Refer figure Q. 13 and

with the diagonals of


8==|. Also prove

a, ft y, S

OA*=i,

7)

OB^j

and

OC=k,

so

that the diagonals are

Let OL be any

line

OP. 01= OP

OLcosa,

V (*

(i+j+k)
cos

or

Similarly cos

cos

cos 8=
/.

cos2

a+cos /3+cos y+cos

a.

+J>

T*

COS a

(Property 8)

113

Multiplication of Vectors

Ex. 15.

Prove that

of the squares of

angular points of a

its

distances from

sum of the squares of half the


Ex.

1 6.

be the

If

OH in the ratio

divides

exceeds the

triangle

the mid.

sum

points of its sides by

(Agra 48)

sides.

circum centre,

ortho-centre of a triangle, prove

squares of the distances of

the

from

any point

sum of the

that 0, G,

the centroid

H are

and

collinear

H the

and

that

1:2.
"

Take

the circum-centre of

the triangle

ABC

are a,

O^^OB^OC

b and
,

a ^=b

Again position vector of centroid

OA^GB^OC,
=c2 ...... (1)

Also

c.

Fig. 67

as the origin so

that the points A, B,

or

/J

is

---.

e~)

From

(1),

a2

(a+b)
or

b2

b 2 =c 2

or

(c-a)=0

(a-b)-O, [(a+b + c)-a]

[b^c]0

[(a+b+c)-b]

[c-al0.

[(a+b+c)-c]

if

(a-b)~ b+c) . (b-c)(c+a)


.

and

Now

c2 =a 2

or

whose position vector


show that

be the point

then above relations

is

CM B40, AM CB=0, BM AC=Q


CM J. BA, AM J, CB and BM AC.

i. e.

Hence
H.

is

ortho-centre of the triangle

to be

0,

G and

H are collinear.

__>~
QH-QG**WG
divides

OH

_*
or

C?H20C.

in the ratio 1

2.

which

is

given

Vector Analysis

114
Ex. 17.
in

AB

side

the

ABC,

7/P

be any point

of a

triangle

such that

AP~p

PB,

then prove that

A.
..(1)

..(2)

Multiply
A

(1)

by A and

2
<M+f* CB

or

p, .

PB

or

and add

/z

CP 2 +A

.
/LL

BP*

PB-A

PB+A PA^Q.

Ex. 18. Prove by

Hence

etc.

vectors that in

ABC,

any triangle

a^b
and

(A+ft)

Now

by

(2)

cos

C+c

cos

a*~b*+c*-2bccos A.

(Agra 42)
Let
modules

C,
a,

C^ and

--6

and

4jB

have

respectively.
*

Also

Squaring,

115

Multiplication of Vectors

we

Squaring,

get

cos

-A)

(IT

cos A.

Again

J5C= -

J5C

BC*=*-CA .BC-AB.BC

or
or

<2

a cos (T

a=6

or

Ex.

cos

a cos

c)

B)

(IT

C+c cos B

Prove by vector method the following formula

19.

plane trigonometry

of

cos (<x~ft)^cos

Let there be

two

cos

ft+sin a sin

ft.

unit

along OX and
OT, two perpendicular lines
vectors i and j

the

in
If

plane of the

paper.

OP and OQ, be any two lines

in

same plane

the

making

angles a and ft with OX res~


..
.,
,
n
pectively, then Z.POQ,= a~ ft.

&

"O

Fig
y No. 70

-*.

represent unit vectors along OP


and OQ, respectively so that their dot product is cosine of
the angle between their directions,

Again

i.e.

let

OA. OB
But OA

OA and OB

l.l.cos(a-0)

similarly
/.

(1)

inclined at an angle a to the direction of

be expressed as 0-4=cos a

and

cos(a-0)

OB=cos

OA OB=[cos
.

ft .

i+sin a
i+sin

i+sin a

can

ft . j.

j]

cos a cos /3+sin a sin

[cos
ft

ft .

i+sin

j]
(2)

Vector Analysis

116

i~jl

V
Hence from

and

(1)

i.jj.i0.

or

(2),

we

get

cos (a-/3)s=cos a cos /3+sin a sin j9.


Ex. 20. Particles of masses ml} wa, w3 ...... are placed at
Prove that
points A,B,C.. .respectively and G is their C. M.
,

the

for any point P,

+(2? mj)
It

be convenient

will

if

we choose

the centre of

G as origin and the position vectors of A, B, C.


as a, b, c.
.respectively and that of P be p.

PG 2

mass

be taken

. . .

. .

AP=GP-GA=*p-*, BF^p-b

.".

Also since the

m.

c.

is

at the origin,

etc.

we have 2
...(1)

2
-2p a+a

Also

aswit (p

- 2p . a+a 2)+wa

(p

- 2p

+(m1a
=(2:

Ex. 21.

pectively

If there be four non-coplanar

cos (a, b) stands for the angle

are parallel to

+m b
2

+....)

2
2
m^ PG 2 +m1 ^G +m2 BG + ....
V 2?m1a0by(2)

straight lines

vectors parallel to their directions be denoted by

and

b+b 2)+ ....

a and b

b, c and

between the

then prove that

cos

a.,

(a, b)

cos

(a, c)

cos

(a, d)

cos

(b, c)

cos

(i,

d)

cos

(c,

f)

cos

(b,

a)

cos

(c,

a)

cos

(c,

cos

(d, a)

cos

(d, b)

b}

cos

(rf,

c)

and unit

lines

res-

which

117

Multiplication of Vectors

We know

that there exists a linear relation

any four non-coplanar vectors and

......... (1)

xa-kyb+zc+wd^Q
Multiplying

and

b=l
x

We

.
.

1
1

(1)

cos

+j>

(a, b),

cos

by

scalarly

a.al

that

noting

in succession

a unit

is

and

vector

we

get on multiplying (1) scalarly by a


.(2)
b)+z cos (a, c)+w cos (fl, d)=sO
. .

(a,

can write down similar

multiplication of (1) scalarly


ting x, y, z and w between the

we

c and

a, b,

PS

between

be

let it

by

relations

b,

as above

On

c and d.

on

elimina-

four relations thus obtained,

get the required result.

The position vectors of the foci of an ellipse are


b, and the length of the major axis is 2a. Prove that the

Ex. 22.

and

equation of the ellipse

We

know

is

that

major

Hence

axis.

if

case of ellipse the sum, of the


it is
equal to the length of

the

in

focal distances of any point

on

r be any point on

or

is

+2r b+b =4a -4a (r-b)


|

square of
-fr

its

module]

+b -2r .b
2

r b]=a (r b
Square again.
2
b+(r . b) 2 a2 (r2 -2r . b+b
a 4 -a 2 (r2 +b2)+(r . b) 2 =0.
2

or

[a

a 4 -2a*r

or

Ex* 23.
and

then

[Square of a vector
or

it,

r+b + r-b =2a


2
2
(r+b)
[2a- r-b
|

b2

).

Prove that the hyperbola whose foci

and whose transverse axis


|

Ex. 24*

r _ bl

is

_
|

2a

is

r _ ba

Prove that

b
ab

given by
|

-2a.

we

points

bx

118

Vector Analysis

Vector Product*

3.

Definition.
51, 57, 585

Pb. 60; Raj. 57)

two

a and b

(Agra 40,
The
a

vector (or cross) product of

and b

is

respectively

being the

between the

angle

whose direction

a vector

is

directions
to

perpendicular

being regarded positive, if the rotation


clockwise.

In case the rotation

and hence

clockwise direction

Thus
where n

axbfli
is

perpendicular

and

sin

unit
to

vectors

whose

is

of

module
vectors

of moduli
ab sin Q,

is

and

and

a and b, this direction


from a /o b appears counterboth

from

to a,

then

it

will be in

negative.

.n

vector

both

b.

Fig.No.il
f

Vector product is not commutative.

1.

We

have proved that dot product of two vectors is


commutative, i. e. a . b=b a, but this does not hold good
in the case of cross product of two vectors a and b, i. e.
axb is not equal to bxa because the rotation which
carries

to b,

i. e.

counter-clockwise is opposite to that

which carries b to a, i. e. clockwise. Of course the magnitudes of the two are same, but their sence is opposite.

Hence a x b
Therefore
five,

i.

e.

we

- (b x a).

conclude that

the factors in

the

and only if the sign of the product


2.

vector

product

is not

commuta*

a vector product can only be


interchanged tf

Vector Product

is reversed.

is associative*

Just as dot product of two vectors is associative, their


cross product is also associative, i, e. if either factor a or
b, iiithe cross product
is multiplied by a scalar m, then
their product

is

al$b multiplied

by that

scalar,

119

Multiplication of Vectors
(ma)

i.e.

X b=a X (mb)=ro

(a

X b)=m

Thus the vector product of two vectors

(ab sin

n)

is associative.

Cross product of two parallel vectors.


We know that aXb=0i sin . n.
In case a and b are parallel, then angle between them
6 should be either
or 180 and in either case sin 0=0.
3.

i.

e.

axb=0.

.'.

Thus we

From here

it

also follows that

vector product

conclude that

aXa=0.

of two parallel or equal

vectors is zero.

Conversely, if axb=0, /. ab sin 6 n=0, then either


or i=0, or sin 0=0, i.e. either of the vectors is a zero
or null vector, and in case neither of the vectors is a zero
vector, then sin 6 being zero shows that they are parallel.

0=0

Thus

of two vectors neither of which

if cross product

is

a zero

vector vanishes, then these vectors are parallel.

Cross product of two perpendicular vectors.

4.

In case the vectors are perpendicular,


sin

5=1;

i.e.

0=90, then

aXb=fl.n.

.'.

of two perpendicular vectors is a vector


whose module is equal to the product of the moduli of the given
vectors and whose direction is such that a, b and nform a right-

Thus

the cross product

handed system of mutually perpendicular

vectors.

Cross product of unit vectors.


5.
In case a and b are unit vectors i.e. their moduli are
each unity, then aXb=sin 6 . n.
Thus
module

is

the

of two unit

cross product

equal to

the sine

of

the

angle

vectors

is

between the

vector

whose

directions

of

the given vectors.

6.

Unit vectors

From above

it is

i, j,

k.

(Very important relations)

easy to deduce that

iXi=jXjkxk=0
whereas
and

i*sjk2 l
ixjk~jxi,

120

Vector Analysis

kxi=j-ixk,
whereas
7.
If

j=j . i~0, j k=k . j=0, k . i=i


We know that aXb=ai sin 0.n.
i

b' be

b so

that

of

remains the same.

aXb'=fl6

/.

Thus we
remains

k=0.

perpendicular to a then
in the same plane as a and

the

component
Also a and b' are

b'4 sin 0.

conclude that

sin 9

the

unchanged if one J actor

n=axb.

of two vectors
replaced by its normal compo-

vector product

is

nent to the other.

The component of a vector r perpendicular to a


(Delhi 59, Lucknow 52)

8.

given vector a.
Let a unit vector

in the

direction

a be denoted by

of

a=0i and in a direction perpendicular to it be j.


Let the unit vector k be perpendicular to the plane i and j.
Now if r be any vector in the i j plane inclined at an angle

so that

B to
a,

a then

i.e.

its

is

components in the direction perpendicular to


...... (1)
rsinfl.j

Now

aXr=0r

sin 8

where k is a unit vector perpendicular


both a and r lie. Now k x i=j.
:.

ar sin 6

j**ar sin

c
sin
.

component

Note

We

component

of

of

have

in

will

found above.

show

"i
or

- ax(axr)

in

2'11

P. 103 that

a direction perpendicular to a

in the

(2).

'

proved

We

(aXr)Xa

r JL to a

which

to i j plane in

kXi=(aXr)xi by
(axr)xi
(aXr)Xfli
~
sa
.

or

...... (2)

.k

a8

the

is

a
a*

following pages that

it is

same as

Multiplication of Vectors

121

Distributive Law.

9,

aX(b-fc)=aXb-faXc.
See

5 Cor. 5 P. 138 for proof.

Thus

in general

(a+b+c-t-

. .

.)

X (p+q+ r +

. . .

.)

sraXp+aXq+aXr+....

+bxp+bxq+bxr+....

+cxp+cxq+cxr+ .....
10.

Expression of vector product in terms of

unit vectors.
If

and

a and b be expressed

in

terms of unit vectors as

b=i1i+
.'.

a X b=(a1i+a 2 j+

sk)

x fti+M+W-

+(A-A)k
V ixi=jxj=kxk=0
The above may

axb=

(1)

andixj=-k=--jxietc.

be expressed in determinant form as

bl

b%

bs

[Remember]

and remembering that square


of a vector is the dot product of a vector by itself and also
2
2
l and i .
.
. i0,
that ia *j *k
Squaring both sides of

(I)

kk

jj

sin 0*

or

Again if /lf mx
of a and b, then

and

/a ,

^,

be the direction cosines

122

Vector Analysis

Tv+v5 u< T *1

^-

and

" i= *''

'

sin 2 6

.'.

In case the two vectors

hence from determinant


*.P,,

-as

bl

a*

and

identical,

as

aXb=0

we must have two rows

(2),

ai

are parallel, then

-y-

b3

b%

Again
sin 2 ^

= l-cos2

and

known Lagrange^s

11.

Incase

Now

if

2=ss

from 10 of

and putting the value of cos


the well

+V+i

2
1

l='2
2

+^2 2 +

P. 104,

it

(1)

does not

and

mean

then

12.

(axb)

axb=0&

...... (-2)

Squaring both

(axb)

=a b
V
13*

fl

where

sides,

==a and

2
fe

we

is

a unit vector.

get

0,

n2

(1-cos 0)=a

-(a.b)

=b

sin (oc-/3)*=sin

sin

then

from

a as b=c+A'a.

=<2 2 6 2 sin 2

axb=axc,
b may

parallel to

n,

2 2

if

but that

is

that

b=c only

=a2 b2 -(a.b)2

sin

we conclude

(2),

that

c by a vector which

get

...... (1)

axb=ax(c+a)=*axc
V axa=0,
From

we

identity

aXb=axc

b=c+#a,

2
2

-02* 2

cos 2 $

a
,

and a

a cos

2 a

ft

b=ai

-cos a

cos

sin

(a+j8)sin a cos /?+cos a sin

/3,

jS.

0.

differ

Multiplication of Vectors

Proceeding as in

19

Q.

123

P. 115,

04=cos

j,

OQ' makes an angle

If

\\ith

a i-fsin a

the

direction

of

IT

in

opposite direction, then

FigNo.72

[sin

j,

0#xO^=(cos
or

where n

is

sin(a

jS)

i+sin

j8

n==(cos

jS

]8

a unit vector

J. to

Now ixi=jXj=0
sin (a

.*.

or

]8)

and

Similarly

and proceeding

j)X(cos a i+sin a j)
sin a ix j+sin ft cos a jxi),

ij plane.

ixj=n

n*=(sin a cos

sin (a-/3)=(sin

sin

/?)=

]8

and

jxi=-n

cos a sin

a cos j8-cos a sin

]8)

0).

OB'xO^=(cos ]8 i-sin
as above, we have

sin (a+/3)=sin a cos

)8

-f cos

j)

(cos

a sin

ai+sin a j)

/?.

Exercise
Ex.

i.

Two

vectors as follows

vectors

a and b

a=3i+j+2k, b=2i - 2j+4k.

unit vector perpendicular to each

of the

vectors.

sine of the angle between the given vectors ?

aXb

in terms of unit

are expressed

312
2-24

Also

W%o/

determine the

(Lucknow

=8tf-j-k)(fromlOP.

121).

48)

Vector Analysis

124

Nowaxb

represents a vector perpendicular to both


unit vector in this direction is obtained

a and b and hence a

by dividing 8 (i-j-k) by
or

module,

its

i.

?.

V(8

+82 +8 a

8V 3.
Hence

the required unit vector

is

(i-j-k) ~
z

Again (a X b)*=8 (i-j -k)*=8


(ab sin

n)

=88 .

. 3.

a= V(9+l+4)= V(14)

but

and *=V(4+4+16)=V(24).
.'.

a*b* sin 2 6

1=8*

"*-]j7i-*

Prove that the

2.

'

8in

Ex,

unit

sin *

=
77'

vector perpendicular

" 3l
2i-j-hk am* 3i+4j-k

the vectors

and

i*

of

the

(155\

sine of the angle between them is

to each

{ufij(Utkal 53)
Taking a and b from Ex. 1 or 2, prove that
is perpendicular to both a and b.
represents a vector which
Hint. You should show that dot product of both a and
Ex*

with

aXb

Ex.
point

axb

3.

4.

and

is

zero.

Find

the

equation

equally inclined to

tine

of

the

straight

vectors a, b,

c in

line

the

through the

form

(Imp.)

<=>+

Let the equation


parallel to

unit vector

of th

A
k

line

through the point

so that

its

d be

equation

is

125

Multiplication of Vectors

AAA

Since the required line

equally inclined to vectors a,

is

b,

therefore they are equally inclined to k.

c,

.'.

k=0

1 cos

a
/.

6,

A
k

b k=i
b A

1 cos

Above shows
tions of a, b and c are

A
k can

then

k^c.

1 cos 6.

k=cos

A A A A A A
a,k:=:b.kc.k=:cos0
A
that the resolved parts of k

or

k=s=

i-

0,

(2)

in

the direc-

In case their module be

equal.

A,

be written as

A A, A

Substituting the value of

,/a

.,.
. .

A
k from

(3)

in

(1),

we

get

,(o)

the

required equation,

as

=b+/A+-+.

Second form.
Again from

(2),

we

Now we know

AAAk=0,
AAA
(b-c).k0.

get (a-b)

is

parallel to

(a-b)x(b-c) because

pendicular to both

a and

b.

is

zero
that

A A
and therefore

perpendicular to both (a-b) and (b-c)

A A
is

two vectors

Hence we conclude

A A

A
k

that dot product of

provided they are perpendicular.

Replae/Abyj.

axb

is

a vector per-

126

Vector Analysis

AAAAAAAA

A
k=f (axb-axc -bXb+bXc).
A A
A A A A

.'.

Put

bxb=0

-aXccXa

and

.\

A A A A A A
k=f (axb+bxc+cxa)
/axb bxc cXa
+ be + 'ca
V~S~
(bxc)+*(cxa)+c(axb

J
(b)

Prove that in a regular tetrahedron the perpendiculars from

the vertices to the

opposite faces meet at their centroids.

Let the vector


sectors of

the

be chosen as origin and the position


other vertices A, B and C be a, b and c.

Since the tetrahedron

on the face

9C

i. e.

vectors

is

regular, therefore perpendicular from


inclined to OA, OB and

ABC will be equally


as such
a, b and c

But since the tetrahedron


.'.

its

is

or

a**b=*c.

......... (1)

(a+b+c)

r=^

Equation to the plane

r c

regular,

OA=OB=OC
:.

equation will be

ABC

is

......... (2)

r(l-*-Oa + fb+fc
For the intersection
:oefficients,

of

(1)

and

on comparing the

(2),

we have
A
a

A
*

Hence the

s=l
a

,
ssl-j-/

--A
a

a
A-

A
=BJ
a
or

A
a

position vector of the

=*t

=$=*.

point

where

the

127

Multiplication of Vectors

perpendicular from

meet

ABC

is

a+b+c
3

which

vector

method

and angles of a

triangle,

By

5.

ABC.

the centroid of face

is

establish

the

sin
sides

AB be

Let BC, CA,


a,

b and

relation

sin

between

the

sin

e.

i.

vectors

c respectively*

a-fb+c=o.

.'.

Now we know

that cross

product of two vectors one of


which is zero and cross product of two

equal vectors

fig No 73,

is

zero.

axb + axc=0,
aXb=cXa V -aXc=cXa.
or bxa+bxc=0
Similarly, bx(a+b+c)
bxc=axb.
axb = bXc = cXa

or

or

.'.

ah sin

or

or

ax(a-fb-fc)=0

.*.

(TT

or

C)=fo

ah sin

sin

C=fo

Dividing throughout by
sin

(TT

A)^ca

sin

(TT

B)

sin A*=ca sin B.


abc,

we

get

^ sinb B "" sinc C


->

(b)

//

variable vector
that locus

ofB

the

vector product

of a constant vector

OB

in a fixed plane

AOB

is

a straight

be a constant

line parallel to

with

vector,

show

OA.

(Luck. B. Sc f 46, 49, 55)

Vector Analysis

128

OAssti say and OB=*r.

Now aXr=constant

given.

=axb

where

OP=b

ax(r-b)=0.

:.

Since the cross product


llel

say.

zero, therefore

is

r-b

is

para-

to a.
.'.

b=/a

Above represents a
Hence the locus of B is as
Ex.

Find

6.

OA~a 0B=b
3

the vector area

the

r=b+/a

or

line

through

parallel

to

OA.

given.
area of a

vector

and they are

at

inclined

of a triangle whose

vertices

triangle

OAB

where

and hence find


are the points a, b and c.

an angle

(Utkal 50)

We know that axb=06


Now

area of
.'.

sin

&OAB=$OA OB
.

axb=0ft

Hence vector area

of

sin

n.
sin

6=\ab

sin

B.

ns=2A n

A0-4J3=|axb.

Also the vector area of the parallelogram two adjacent


sides of

which are a and

Now
c,

if

is

a X b.

the position vectors of A,

then fiC=sc-b and

B and C

be

a,

b and

&4=a-b.

Therefore vector area of

&ABG

is

Uc-b)X(a-b)

=J (cXa-cXb-bXa+bxb)
= i (a X b+ b X c+ c X a)
[Remember]
V bxb0and -bxaaxb.

129

Multiplication of Vectors

Note.

In case the

three points

aXb-f bXc+cXa=0.
In a triangle ABC
7.
BC, CA and AB respectively

are

collinear,

then

clearly

Ex.
tides

D,

points

E F
9

such that

BD DC^CE EA^AF FB=n


:

A DEF- -*~-? t

prove that

(n+2)

Taking
the

let

position

B and C

of

respectively.

the

F,

position

and

as

are taken on th*

/,

ABC.

origin

vectors

b and c

be

Therefore
vectors

of

E are respectively

--

n+l

Vector area of &ABC*=1> (bxc).


>

Vector area of

-*

[by ^.

G.J

&EFD*=\ (EFxED)

(n^lf

(b-.c)x[nc+(l-w) b]

bxb=cxc0

130

Vector Analysis

Note
1,

we

Ex.

Z), E, F are the mid.


&EFD=&DEF=*l&ABC.

In case

get

The

8.

a, b, c being the lengths

We

tig.

know

Ex.

D, E, F; show

CA

of BC,

and

that

AB

respectively.

7.

that internal bisector

of

an angle divides the

opposite side in the ratio of the arms of the


divides AB in the ratio CA CB, i.e. b a etc.
:

/.

b+a

'

angle

i.e.

position vectors of F,

^ib+a.O

E are

and

respectively

cc+bb ^a.O+tc
a+c
c+b
'

^b
Tt.
Tt,
EF=AF-AE=r~
JTt,

.\

ABC

infernal bisectors of the angles of a triangle

meet the opposite sides in

Refer

points, then putting

cc

(a4~c)

bb

'

(b-\-a)

cc

c+b
1

"
Now

+ a)

V+*)
1st

multiply

and 2nd by

(c+a]

']

bracket above and below by

(fl-f i).

X{c (a-*) (a+i)

Now

keeping

-cxb=bxc, we

in

get

view

that

c-f-A (c+a)

(a+A)

bxb=cxc=0

b}].

and

Multiplication of Vectors

131

(b+c)

(c+a) (a+b)]

"-*+"+*

(b+c) (c+a]

Ex.

Prove that (a - bj

9.

X (a + bj

(Agra

2a X b

Lucknow

47,

0/2 rf

interpret

Andhra

56,

it.

36}

We know that
aXa=bxb=0
and

-bxa=aXb.
(a-b)x(a + b)

/.

=2aXb.
Interpretation

Ftg.No.75
:

In the parallelogram ^BCZ)

0^=-b and

/.
.*.

AB=a-b

and

OD=b.

and

b)X(a+b) represents the area of parallelogram


being 2axb, i. e. twice the area of the parallelogram

gram.
Ex. 10.
by the vectors

Ex.

of the

>

<40a

(a

and it
whose adjacent

vectors

hence

let

n.

in the

sides are semi-diagonals of the

Prove that the

a=i+2j+3k
Given

the

direction

area of the

and

vector

parallelogram

b= -3i-2j-Hk is
r=0i-J-j where

parallelo-

determined

6^/5.
i,

are

unit

of the axes, find an expression for the vector

same length perpendicular

to

of the given

through

the

(Lucknow

46)

the given vector

origin.

The module

first

vector

is

V( a2 +*

2
)

an <*

it

plane and therefore ixj represents a vector


perpendicular to i and j plane and hence perpendicular

lies

in

i,

132

Vector Analysis

to the given vector

which

lies in this

Since

plane.

its

length

be the same as that of the given vector, hence


should be V(02 +* a )iXj.
is

to

Ex. 12.

Prove that the area of the triangle formed by joining

the middle point of one


the extremities

it

of

non-parallel sides of a trapezium to

the

of the opposite side

half that of the trapezium.

is

(Agra

45, 57)

Let P the middle point of the


oblique side EC be joined to the
extremities of the other side OA,

then

we have

to prove that

Let OA be a and AB be b
so that position vector of

Now OC
The

i.

Fig.

e.

position

vectors of

/.

B and C

therefore the position vector of


J

OJ5=a+b.

AB

being parallel to

No IB

OC=kAB=kb.
are

a-fb and kb

the middle

point of

BC

is

(a+b+ftb).

(a-J-b+fcb)
...(1)

trapezium OABC=* &OAB+


2 A0^4Ptrapezium

&OBC**

O^BC. from

(1).

axb.

133

Multiplication of Vectors

Ex.

AB

of a

and meets

AC

Q, an d -R are the middle points of the sides

13.

ABC

triangle

BQ

respectively.

produced in P.

Prove that

and

parallel to

AB

&RQP &RCQ

and

is

drawn

&ABC.

each equal to one-fourth of

Taking

CP

as

origin let the points

B and C

be

b and

respectively so that
R and Q, are Ab and

Jc repectively.

A/

/?

*\BCxBA**\ (c-b)X(-b)

Equation to

CP

to

BP

i. e.

is

r^c+Jb

is

r~b+$

as

it i?

parallel to

AB.

->

Q,

i.

Equation
e. c/2 and

These

we

lines

(c/2-b) as

it

passes through

b.
at

intersect

P and hence on comparing,

get

1^=5/2 and

t=]

/.

^=2and/s

Therefore the position vector of

1.

P is c-b.

(Jc-ib)x(c-b-Jb)

etc.

Similarly

Ex. 14.

AC

BD

and

Prove that

its

area is

are

two diagonals of a quadrilateral.

134

Vector Analysis

If

be the intersection of

the diagonals, then quadrilateral


is

&PAB+ AP#C+ APCD-f &PDA


A
FigNo.78

~
+ PDXP.4)
>

__>__ ___.--

\ {(

- PB x PA + PD X PA) + (PB X PC - PD X PC )}

C=^ ACxBD.
^4

15.

ABC
Cft

AC

meeting
be

//^

drawn

is

drawn

A3

EC

of a triangle

E and F respectively. If BR and


AC, AB respectively to meet EF in R and

and

parallel to

in

&ARB= A^C(9.

that
respectively, then prove

4.

parallel to base

(Agra 59)

Product of three vectors.

Scalar triple product and vector triple product.


(Agra 37, 40, Raj. 56, 57)
We have already seen that the dot product of two

vectors a and
of
i.

two vectors

is

a mere sclaar vvhereas the cross

a and b

e.

is

a vector

where n

is

itself,

\>=ab cos 6

axb=a

and

sin 9

product

(scalar)
.

n, (vector)

a unit vector perpendicular to the plane of

given vectors.

Now

since

axb

is

a vector,

we can

multiply

it

both

135

Multiplication of Vectors

The

?calarly

and vectorially by another vector say

c.

former

will be

and the

called scalar triple product

latter vector triple product.


or c . (a x b)
.*.
(a x b) . c

is

b and

the vector a,

of

scalar

is

vector triple product of the vectors a, b and c.


\Vhat do (a
Naturally the questions arises
:

c X (a

b) stand for

product

triple

cX (axb)= -(aXb)Xc

c whereas

b)

c and

b is a scalar and as such (a b) c may be


b c, it simply represents a vector in the direcThus the
tion of c whose module is a b tunes that of c.
of
dot product
two vectors can occur as a coefficient of
Since a

written as a

a third vector.
Again as above a
therefore

cX(a

b)

is

being a scalar and not a vector,

meaningless.

Geometrical interpretation of scalar triple


product (axb) c.
(Agra 59)
5.

In

the

llelopiped

adjoining

three

the

OB

minous edges OA,

OC

represent

in

and direction
a,

b and c

vectors

respectively.

axb

a vector

reprc-

n whose magnitude

OADB

parallelogram
the plane of the face
the directions of
i. e.

arid

magnitude

the

Therefore
sents

paracoter-

OADB. Now

axb

n c=area OADB
.

e.

i.

n and

product

is

+ive when

right-handed system

of vectors.

area

of

we know
value

acute

i,

the

perpendicular to

between

that of c then (axb)

The
is

is

B be the angle

c^s 6 which

the volume of the parallelepiped.


triple

if

the

is

whose direction

e.

represents

of the scalar
a, b,

c form a

136

Vector Analysis

manner we can show

In a similar

(cxa)

also represents the

that (bxc) a and


volume of the above parallele-

piped.

Also

we know

that

b=*b

(axb).

a and as such

c=c (axb)
.

(bXc).a=a .(bxc)
(cxa).b=b .(cxa)

......... (1)
........ (2)
......... (3)

and each equal to volume.

Now we

We

find that

find that

and cross

we

(axb)

c=a

(bxc) from

(1)

and

(2).

have* interchanged the position of dot


triple product, but the cyclic

above scalar

in

order of the factors

maintained.

is

Similarly,

(cxa).b=c*(axb) and (bxc) a=b (cXa).


Thus we conclude tbat in the scalar triple product
the position of dot and cross can be interchanged at
.

'

pleasure provided
vectors a, b and c,

we maintain

the cyclic order of the


(Pb. 60,

Agra

59)

Effect of changing the cyclic

order

Now we know

that

(aXb)--(bxa).

F~(aXb) c=-(bXa).c
=B-c. (bxa),

/.

(bxc)

,a~(cXb).

-a.(cXb),

(cXa).b-(aXc) b
.

fig Mo.

80

Thus we observe
of

the vector a,

a, c,

or

changed.

c, b,

that by changing the cyclic order


c
or b, c, a or c, a, b to b, a, c or
b,

a the

sign

of

the

scalar triple product

is

137

Multiplication of Vectors
Also

we

the position

- fa X c)

see that

of dot and

maintain the

cyclic

order of a,

-a

X b) and

so on,

i.e.

changed at pleasure whether

you

b*=*

cross can be

(c

and c or not;

if,

however , you

change the cyclic order, the sign should be changed.

Notation
is

The

scalar triple

product of three vectors

Thus

generally written as [abc].

[abc]=[bca]=[cab]= -[acb]= - [cba]= - [bac]


In each of the above different forms

of

scalar triple

product the position of dot and cross can be changed.

[ijk] = i.(jXk)=i.i = ], V jXk-i.


Cor. i. Condition for three vectors to be coplanar.
[abc]=a . (bxc) where a, b, c are three coplanar
.'.

Now bxc

vectors.

represents a vector v\hich

is

perpendi-

the plane containing b and c in which also lies


the vector a and hence bxc is perpendicular to a.
There.
fore a (bxc)=0 being the dot product of two perpendicular

to

cular vectors

a and b X

c.

Thus

[abc] =0

when

the three

vectors are coplanar.

Converse

bxc is

If

[abc]=0,

perpendicular to a.

b and

the plane containing


in the

b and

plane of

Cor.

2.

i.e.

(bXc)=0 showing

that

But bxc is perpendicular to


c and h^nce a should also he

c, i.e. a, b,

c should be coplanar.

Scalar triple product

when two of

the

vectors are equal.

Now aXc

perpendicular to the
=
plane containing a and c and therefore a.(axc) 0or
otherwise also [aac]=a (axc)^(axa) . c because the posi-

[aac]=a

tion of dot

(aXc).

and cross can be changed.


/.

Cor*

is

3.

[aac>=(axa).

c0,

Scalar triple product

(aXa)s=0.
the

when two of

vectors are parallel.


Let a and b be parallel so that b=*A;a wheie k
scalar.

is

Vector Analysis

138

[abc]a (bXc)=a

/.

.'.

Hence from Cor.

3,

The

product

scalar

triple

they are coplanar or

Cor.

We

4.

(A;aXc)=A:

2 and

that

[by Cor.

[uc]-0.
3,

we have

the following

oj three vectors

two of t htm are equal or

know

will be zero

2].

when

parallel.

any vector can be expressed

in

terms of three non-coplanar vectors as

a =0jl +

c^l

bXc

<7

c2

-f- fl 3

n,

m + c^n

-f b 3

mX n+

(btf

-b

nX1

/.

[abc]=a

(bxc)

+( Vi

*i f 3)

n x * +-(*i^2 "" Vi) 1 X m]


n *l)
*i c a)
- 2 c,)n (1 X m).

fc

Also other terms in the above product vanish as the


scalar triple

is

zero

when two

of the vectors are equal.

Again we know that


1

(mxn)=m

(nxl)=n . (lxm)=[lmn].

[abc] =

[Imn].

(Agra 38)
In case a, b, c be expressed in terms of unit vectors
k, then [ijk]l and hence

i,

j,

139

Multiplication of Vectors
Mi

[abc]=

Above

bi

bz

bz

cl

c2

f3

the well

is

known expression

for the

(Agra 40)
volume of a

parallelepiped whose one vertex is at the origin and the


other three at (%, 2 a 3 ) (b l9 6 2 6 3 ) and (c lt c 2 c3 ) [rectangular
,

coordinates],

Note

In

case

then the two rows


cal

and as such

it

any two vectors are equal or parallel


above determinant will be identibe zero and hence if two of the vec-

in the

will

tors a, b, c be either equal or parallel, then

Cor.

To

5.

deduce the

two vectors by the help of scalar

[abc]=0.

law of cross product of

distributive
triple product,

c.

i.

aX(b+c)=aXb+aXc.
(Agra 51, Raj. 57)
Let r be any
butive,

vector; then since scalar product

is

distri-

we have
r

[aX(b-f c)-axb-axc]

=r

[aX(b + c>]-r

Again we know

that in

(axb)-r

(aXc)

(1)

scalar .triple product the posi-

dot and cross can be changed without altering the


value of the product. Hence we can w rite R. H. S. of (1),

tion

of

as

(rXa)

(b+c)-(rxa)

b-(rxa)

which can be written as


(rxa) .[b+c-b-c]=(rxa).0=0
because scalar product is distributive.
Hence we have for all values of r,
r

[aX(b+c)-aXb-aXc]=U.

)-aXb-aXc=0
or

The

other possibility of either r being zero

or

it

being

Vector Analysis

140

perpendicular to ax(b-fc) axb aXc


is
any vector whatsoever.

ruled out by the

is

fact that r

Vector Triple Product.

6.

aX(bXc)=a
M. Sc.

A/-~-^

Note
vector

b and

b-a .b

c.

1960, Pb. 60, Agra 32, 35, 59)


a scalar and occurs as a coefficient of

(Alld.

is

similarly,

as

being scalar occurs

coefficient of c.

Method.

xst

Consider the vector

two of the vectors are equal, i> e.


aX(aXb) = a . b a-a
\Vc have done that

b . a
a
a a

is

=a

when

b.

there be a vector b,

if

component along a given vector a


or

product

triple

then

its

(Page 103)

and

in

a direction perpendicular to a

ax(axb)
-

component

aX(aXb)
-

or

a&

its

is

(Page 120)

a
Multiplying by

scalar

a .a
a and transposing, we get
a . a b=a b a a . a b,

aX(aXb)=b a a
V b a=a
.

the

the

First

take

the dot product

bracket with the extreme vector inside


coefficient

of the

vector

bracket

and

same rule

is

Again
or

we

b.

Rule

of

the

remaining one.

outside the
it

true

becomes

when

bracket

the

Then we
with

coefficient

the

of

all the vectors are

the

of

the bracket

the

vector outside

and

it

becomes

take the dot product

nearer one inside the

remaining one.

unequal.

(axb)Xa- -aX(aXb)-(a . b a-a .a


(aXb)Xa=a. a b-a ba

which obey the rule written above.

The

b)

Multiplication of Vectors

141

All the vectors being unequal*

Let

P aX(bxc)=aXn

where

n=bxc

which repreb andc.


P aX(bXc) aXn is perpendicular to both a and
being perpendicular to the plane containing b and c

sents a vector perpendicular to the plane containing


/.

n but n

and therefore
of

b and

perpendicular to a and

is

Hence P

c.

Let

is

P a=0

/.

from

/.

lies in the

expressible in terms of

P=xb+yc
P a=xa b-f^a
already stated that P

But we have

it

(2),

Putting the values of x and

(1)

is

(2)

perpendicular to a.

(1),

we

to find the value of

say-

get

have

P=aX(bXc)=(a cb-a b
Now we

plane
c.

^ ^~if=
in

b and

c)

(3)

k.

P.b=k(a.cb b-a.bb.c)
.

:.

(4)

Now P b=aX(bxc) b
.

=a
as the
if

position of

the cyclic order

=a

=a

[b

.[(bxc)xb]

dot and cross can be changed at pleasure


is

maintained

b c-b
.

b-a

c b] by the rule written before when


two vectors are equal
bb.c
,
(5)

Equating the values of

P b

from

(4)

and

(5),

we

get

t-L
Hence from

(3)

by putting

fc==l,

we

get

P=aX(bXc)=a .cb-a b
.

(bxc)Xa=-aX(bXc)

or

The above obeys


rulel.

e. first

we

of nearer ones

the

=a . b c a . c b.
same law as written before

in the

write the dot product of extremes and then


the factor outside the bracket is included

in both the dot products.

Vector Analysis

142

is

Note : Just as a . (bXc) b . (cXa) etc. but aX(bXc)


not equal to bX(cXa) as the former is coplanar with
expressible in terms of b and c whereas the
coplanar with c and a and is expressible in terms of

b and c and
later

is

c and a,

bx(cXa)=b

i. e.

c-b

c a.

2nd Method.
Let

P=ax(bxc).
j, k and let b be along j.
k be perpendicular to b and

Consider unit vectors, i,


b=6 2 j sa Y an d let
.".

b and

the plane of
/.
i,

j and

c= 2 j+sk
k

and

be flji+flj-f

the third vector

let

X k=

2c3 j

fc

(a

2 c 3i.

<?

^k X

(a

terms of

V jxj=0and jxk=i,
j + 3 k) x (Va)

a X ib X c)= (^1+

/.

Again

in

3 k.

b X c=6 2 j X (f 2 j+ 3 k)=

/.

in

c.

......... (1)
V jxi=-iXj=-kandkXi=j.
(c j + cjt)] b j
c) ^^[(a^+a^ + a^L)
.

(a

+ flak)

b) c^Kflii+flJ

Hence from

c)

b- (a

(1)

and

(2),

b)

c=0 3

we

a X (b X c)=(a

-r

b, j]

3& 2 j

U 2 j+c 3k)

- aj>tfjk ...... (2)

get
c)

b - (a

b) c,

Similarly,

bX(cXa)=(b a)c-(b c) a.
cx(axb)-(c b) a-(c a) b. Hence proved.
Adding, we get aX(bXc) + bx(cXa)+cX(aXb)=0
because (a b) c=(b a) c etc.
(Agra 42, 53, Annamalai 38, Andhra 36)
.

Multiplication ot Vectors

Note

The component

.1
to be

shown

r
2

a2

r-r

143

r perpendicular

of

.a

*>

,ar-r

Note

8 P. 120)

Or

s=r

aX(aXr)
----4

Verify the above formula

aX(bXc)=(a c) b-(a b) c
a=i-2j+k, b=2i+j+k, c=i+2j-k.
.

by taking
7.

Scalar product of four vectors

(axb) .(cxd) =

a
Let us suppose that

(Agra

37, 51)

cxd=n.

(axb) .(cXd)=(aXb)

.'.

.a a
(See

a2

a was

(See P. 103)

a a

ax(rxa)

to

n=a

(bxn)

as the position of dot and cross can be changed,

.[bx(cXd)] = a

=a

.da. c-b

ca

dc-b c d]
b
d= a c

[b
.

a
8.

'

Vector product of four vectors (a X b) X (c X d)

Let c X d = n.
/.

(Agra 38, 42)

(axb)x(cxd)=(axb)xn=a nb-b na
= a (cxd)b-b.(cxd)a ......... (1)
.

Again
/.

let

=[acd]b-[bcd]a.
(axb)=sm.

us put

(axb)X(cXd)=mX(cXd)=m d c-na
,

=(axb) d c-(aXb) . c d
.

Equating

(1)

and

[acd]

[abd]c-[abc]d
(2), we get

......... (2)

b-[bcd] a=[abd] c-[abc] d

Vector Analysis

144
or

[bed] a-[acd]

Now

replacing

d by

b-[abd] c-[abc] d=0.


r, we get

[abcj r=lbcr] a-[acr] b+[abr] c

...... (3)

Now

a scalar triple product remains unchanged if the


cyclic order of the factors is maintained but tor every
change of cyclic order, there is a change of minus sign.

[bcr ]=[rbc], [acr]=[rac]=

/.

- [rcaj

[abr] = [rabj.

and

Hence from

(3),

we have

"
r = [rbc]a+[rca]b+[rab]c

...... (4)

'

[abcj

(Agra 35, 48,

Above

57, 60)

expressess a vector r in terms of any


other three vectors a, b, c provided they are not coplanar
t>.,

relation

[abcj^O.

Rule.

The

last three

arrangement of a,

and

cab

bca,

i.e.,

numerator are the

in the

vectors

c,

abc and

and

being r.

cyclic

the first

-^^

9,

The

Reciprocal system of vectors.


three vectors

b', c' defined

___

(Agra 59, 60)


by the equations

~[abc]'

~[afacj'

are called reciprocal system to the vectors a, b, c which are

non-coplanar

i.e.

i.

Property
system

[abc]^0.
If

a, b,

of vectors, then

XT

Now a

=a

a', b', c'

b'=c

b *c
ti~i

ass

r~ir~i

SSB

l'

b b'=c . c'l.
. a'+b
b'+c
.

Similarly,

/.

a ,b

1
-I-

c'3

and c

be

c'=l.

[abcj

[abc]

or

c and

a'^=b

reciprocal

145

Multiplication of Vectors

because of the above property that the two sys-

It is

tems of vectors are

Property

called reciprocal systems.

The product of any

a.

vector

of one system with

a vector of the other system which does not correspond


i.e. 9

ero,

b'=0.

,,
.

b'=a

FcXai
L

U facai]=()
'-

[abc]
as the numerator

two

to it is

[abc]

the scalar triple product of three vectors

is

which are equal and hence it is zero. (Cor. 2 P. 137)


= b . c' c . a'=0 etc. etc.
Similarly a c'

of

Thus we

Cor.

conclude

a', b', c' be

reciprocal

cal system to

a', b', c'.

Property

3.

non-coplanar vectors

system

The
is

from

to a,

scalar

the

triple

reciprocal

two properties that if


c is a recipro-

b, c, then a, b,

to

product [abc] of any three

the corresponding scalar triple

product formed out of the reciprocal systtm of vectors

a', b', c'.

(Agra

47, 51, 59)

''

...... (1)

[a'b'c']=a'.0>'xc')

Now
a,

b anc
.

..

substitute

the

values of

a',

b' and c' in terms of

c.

(bxc).f(cxa)x{axb)i
---------------

.,
[abc]

label 8

Now (cXa)x(axb)=(cXa)Xm say=m


.

...

......
c

a-m

_.

ac

ca-(axb).ac=[abc]a.

V (aXb).a=0.

[abc]

,--

ttom(-)

LabcJS

_
~

__ _

[abc] [bca]_[abc] [abc]


Iabc]
fabc]
~[abc]'
"

.'.

[abc] [a'b'c']=l.

(Agra 47)

Vector Analysis

146

From

Cor.

above,

we

conclude that

a X b, b X
\

c X a]=[abc] 2
(Agra 36, 41, 51, 53* 57> 6 5 Pb - 6 ; Andhra 38;
c,

Benaras
If

a, b, c be expressed

tfii-f

j+0 3k

etc

^ien we

56;

Raj pu tana 56)

terms of unit vectors,


have already done that

in

i. e.

[abc]=
*3

(Cor. 4 P. 138)

C*

Again

ax b=(a 1i
i*2- fl A)

(P. 121)

and similarly we can write

for

bxc

and

ex a.

[aXb, bxc, cxa]

/.

(Cor. 4, P. 138)

(where capital

"

-"2

-"

GI

C2

C3

letters

denote the co-factors

of the corresponding small


letters)

which

is

equal to
*, i. e.

[abc]

2
.

(Refer Author's Algebra]

147

Multiplication of Vectors

Note.

//

[aXb, bXc, cXa]


product

c are non-coplanar

a, b,

are

2
[abc] which

is

We

Cor.
expressed

in

also

scalar triple

have done before that any vector r can be

jMi

(b

_^

a', b', c'

X c)

,--i

a, b, c.

aa

[abc]

[sJbcJ

/.

their

terms of three non-coplanar vectors


b] c
[rbc] a _ r

c;

non-coplanar

then

[abcj^O,

e.

as

not zero.

is

r JrfElj.

where

i.

form a reciprocal system

r=r a'a+r
.

b'b-f r

of vectors to a, b,

c'c.

(Agra

38)

Also the two systems of vectors [abc], [a'bV], each is


and as such any vector r can
reciprocal of the other
also be written as

r=r

Again

i-j

a a'+r
j

b b'+r

^k k = l
.

and

[i j

the system of vectors i, j, k


Hence in terms ot unit vectors i,

r=r.ii+r

c'.

k]=l.

is its
j,

k,

own reciprocal.
we have

.jj + r.kk.

[See P. 104]

Exercise
Ex.

i.

Prove

[a+b, b+c, c+a]=2 [abcl,


(Dacca

that

being three vectors.

Ex.

2.

27, 29)

Prove that

l.a

[Imn] [abcjsa

and give

a, b,

l.b

l.c

n c
.

its cartesian equivalent.

(Agra

38, 40, 47, 49, 51, 54;

Lucknow

52, 5

Pb. 60; Benaras 52; Annamalai 47)


Let

a', b', c'

be a system of vectors leciprocal to

and hence [a'bV ] [abcj-L

a, b,

Vector Analysis

148

we know

Also

that any vector r can be expressed in

terms of any three non-coplanar vectors as


r=r . a a'+r . b b'+r . c c'.

We

express the vectors

shall

m, n

1,

in

terms

of

a', b', c'.

1=1 . a a'+l .
a a'+m
n=n . a a' + n
1 . a
l.a
[lmn]=

m=m

/.

b b'+l
.

n*a

c'

b b'+m c
.

c'.

b b'-fn c c'.
1 . b
l.c
l.b
.

n b

[a'b'c'l

c
.

[Cor. 4 P. 138]

Multiply both sides by [abc] and since [abc] [a'b'c']=l,

we

get the required result.


'

Cartesian Equivalent.

l=y+/ 2 j+/3ketc.

Let

etc.

Also

l.a

[lmn][abc]

.'.

b l.c

a=

...(1)

=j

=k2 =l.
...(2)

m.c

n a n b n

i2

I*

Wl

7^2

149

Multiplication of Vectors

Note

In case

1,

[abc] [abc]=

m, n

same as a, b,
a. c

are

a.b

b.a

b.b

then

c,
-

[abc]

we

get

c . a
c .b
c c
=[axb, bxc, cXa] as proved in Cor. P. 146.
Note. 2. In case 1, m, n are reciprocal to a, b, c
.

or

a=m b=n

[Imn] [abc]=l and 1 .


. a=*m . c=0, n . a=n

b=0.

then

c=l but 1 b=l c=0,


.

[Prop.

P. 145],

2.

(Benares 55)

we

/.

get 1=

001
Ex.

3.

Prove independently that

[axb, bxc, cxa]=[abc]

a.b a.c

a. a

b.a b.b b
c

The

first

part

Now we know

is

that (aXb) 2 =0 26 2 sin 2 B=a*b*


2

square

oi

a vector

is

-(a.b)

square

stands for the dot product of a vector by


and also a b=a6 cos 6.
2

itself

2 2

Again [(aXb)Xc]=(c
2
.%
[(axb)xc] [(c .
a)

b2 +(c

b)

a)

a3 - 2

module and

c)

a2

=a2 =a

it

b-(c
a
b-(c .b)a]
.

(c

6)

a)

-cos 2

i.e.

(1

=(aXb) c -[(aXb).c]
a
a
8
[ab'-(a.b) ]c -[abc]
a
2 2 a
2 a
=a b c -(a . b) c -[abc]

of its

m c - (m

Now [(a X b) x c]2 =[m X c]2

= (c

proved on page 145.

=a b
V

a) (c

b) a,

b) (b

a)

(1)

Vector Analysis

150

Equating the values of [(aXb)Xc]


2
a
2 aa
(a . b) c - [abc]
get a*b c

b2 +(c .
a 2 a a
(a
[abc] =a b c
a)

(c

/.

b)

from

a2 - 2 (c

2
2

b) c

- (b

a) (c
c)

and

(1)

b) (b

a 2 - (c

+ 2(a.b)(b
Now

if

we expand

a)

we

a).

b2

c)(c.a).

3.,

we

get

Hence proved.

the same result.

Ex. 4.

the determinant on L. H.

(2),

m, n be three
l.a
l.b

Prove that if I,

[lmn](aXb)=

n a

non-coplanar vectors

n b
.

(Agra 49, Dacca 40)


'Expressing

1,

m, n and

a,

terms of unit vectors

in

and
etc.

and

[lmn]=

ml

.'.

<*i

^3

<*a

bt

bo,

bs.

and (axb)=

[Imn](aXb)

TO.

l.a
m,

n a
.

l.b

n b

l.a

n a
.

l.b

n b

151

Multiplication of Vectors

Ex.5,

//a,

the following

[b X

1.

\*3

are

three

a X b]=[abc]2

c X a,

c,

36, 41, 515 53 9 57> 6

(Agra

Andhra

Xb

2.

b X c,

3.

Express a, b, c in terms

4.

c X a, a

Benaras

38,

56,

Rajputana

ofhxc, c X a, a X b.

axb in

terms o/a, b, c.

two parts we have already proved

first

56)

are also non-coplanar.

bxc, cxa,

Express

The

vectors, then prove

non-coplanar

in Cor.

P. 146 and Note P. 147.

Let

3.

...... (1)

a=/(bxc)+m(cxa)+(axb)

Multiplying both sides scalarly by a,3

a=Z a (bXc)+m a . (cXa)-fn a (aXb)


a a=6 [abc].
.

or

scalar triple

product

is

when two

zero

vectors are

equal.

a
" /_

'

[abc]'

Similarly multiplying both sides of

and

c,

we

(1)

scalarly

by b

get

a.b

m== r~il~~i an u n=: F~T~-i.


[abcj

(b

X c)=b

[abc]'
(c

X a)=c

Substituting the values of

Similarly
4.

we can write

(b
..

(a

m and

x c)

(b x c)=rJa

t
as u
before

(2)

scalarly

(b x

c)+mb

(bxc).
^---J.
i

X b)==[abc],

n in

the value of

Let(bXc)=/a+/Hb+Hc

Multiply both sides of

Z,

(bxc)

(1),

b and

we

get

c.

...... (2)
by (bxc).
.
(b X c)+ nc

(b X c).

Vector Analysis

152

Similarly multiply both sides of

(cxa) and (axb) and

find

by

scalarly

(2)

m-^^J^-*

_(bxc).(axb)
T~
t
;

[abc]

Substituting the values of

required result.

terms of

a,

Similarly

b and

Ex. 5.

we can

express

we

(2),

cxa

and

get the

axb

in

c.

Express a vector

a and another

m, n in

I,

as a linear combination of a vector

a and

vector perpendicular to

r and a.

coplanar with

We

have already stated that the vector aX(aXr) is a


vector perpendicular to a and coplanar with a and r and as
dot product of two

dot product with a is xero,


perpendicular vectors is zero.

such

its

...... (1)

r=/a+wiax(axr)

Let

Multiplying both sides scalarly by a,

a*=/

a . a,+m a [aX(axr)]=s=Z a
.

Again multiply both sides of


.\

v\e get

(1)

vectorially

a.

by

rXa=/aXa+[aX(aXr)]Xa
s=0+m

rxa=m[a.

[a

a-a. ar]Xa.

r (aXa)-a. a (rXa)]=
"~

Substituting the value of

P== ?J-? a _

a .a

a
/

-m (a

.a)

(rXa).

.a'

and

in (1),

J_

a .a aX(aXr)

Above shows that the component

we

get

<

of a vector

r along

353

Multiplication of Vectors

a given direction a
to

is

Ex.

- ao<jaxr)
-a

is

a and

in

a direction perpendicular

See

g p^

Prove that

6.

[aXb, cxd, exf]


[abdj[cef]-fabc][defj
[abe][fcd]-[abf][ecd]

*[cda] [bef J - [cdb] [aef ].

(Agra

36, 61)

\Ve know that any scalar triple product [pqr] is equal


to p . (qxr)=q . (rXp)=r . (pxq).
Writing the given
scalar triple product in the above three ways maintaining
the cyclic order

we

shall get the three results as given.

show only one


students themselves.

shall

and the

\Ve

can be done by the

rest

faxb, cxd, exf]


=(aXb) [lcXd)X(eXf)]
=(aXb) [nX(eXf)]
e(aXb) [n. f e-n e f ]
.

(n

f)

Hcxd)

[axb) c]-(n . e) [(axb)


f [abe]-(cxd) e [abf]
.

7. Prove that [a

f]

[abe] [fed]

Ex.

[abf] [ecd].

X p, b X q, c X r]+[a X q, b X r, c X p]
( Agra 34, 48, 59; Luck. 55)

+ [a X r, b X p, c X q]= 0.
and

first

C (Ax B)

unchanged if cyclic order is maintained and


changed for every change of cyclic order.

scalar triple
its

sign

Ex.

is

A. (BxC), second as B . (Cx A)


and then add keeping in view that

bracket as

Expand
third as

8.

is

If a, b, c and

vectors, then prove the following

(7j

a', b', c'


:

a.a'+b.b'+c.c'^J*.

form

reciprocal system of

Vector Analysis

154

axa'+bxb/+cxc'=0.
_b'xc' K _ c'Xa' _a'xb'

(2)
/Si
(

'

[a'b'c'j'

Ex

Find

9.

~[a'b'c']'

[a'b'c']'

the value

of

P=i X (a+i) + j X (a X j) + k X (a X k)
P-(i

a-(i

a)i+(j . j) a-(j . a) j+(k . k) a-(k


-(a+a+a)-[(a . i) i+(a . j) J+(a . k) k]
i)

= 3a-a=2a.
Ex. 10*

a)

2'11 P. 10G]

Prove the relation

ax[bx(cxd)]=b daxc-b . c aXd


=[acd]b-a .b cxd
.

and hence we prove

that

ax[bx{cx(dxe)}]
=[a d c . e-c. d a
.

e]

b-f(a

b) [c

(Agra

L.H.S.=ax[b . d c-b

=a

or

(cxd)

c d]

d e-c.

e d]

Delhi 51)

37, 42, 55;

etc.

b-a b cxd.
.

n. Prove that
(bxc) .(aXd)+(cXa) . (bxdj+(axb)

Ex.

(cxd) =

and dednce that


sin

(A+B)

(A-B)=sin

sin

=J
and

c os
*

(A

+ B)

379 4 6 >

5>

cos

A- sin2 B

(cos

(A - B)

2B-cos 2 A) = cos* B-cos 2 A

= cos 2 A - sin 2 B.

Benaras

539 60;

Luck.

b a b d

55,

53; Delhi,

Allahabad

d)+(c

b) (a

=(b

a) (c

Now

d)-(c

a) (b

using the fact that

60

a.d
b.c
.

d)-(a

b, d
7 P. 143

Sc.

b) (c

+(a.c)(b.d)-(b.c)(a.d)
b a etc., we get
t b
.

155

Multiplication of Vectors

L. H. S.=0.

Let

and
Let
:.

LCOD

:.

LAOD

Fig No 81

n be a unit vector perpendicular to


d which are assumed to be coplanar.

Let
a, b, c,

Now we know

(aXb)~

that

EOC) n

(ad sin

-(ac sin AOC)

abed sin

(A- B)

sin (A

+ B)abcd

n
n

(cxd)=0

(cd sin

sin

BOD) n
COD) n=0
A sin A
sin B . sin -6=0

(bd sin

.
.

}-abcd

V
.'.

sin

(4-B)

Again (aXb)

sin

OH^sin

(cxd)

AOD) n

+(ab sin AOB)


or

the

(cxd)0

(bXc). (aXd)-(axc).(bXd) + (aXb)


(be sin

is

treasured from

Again we have proved that


(bXc) . (aXd) + (cXa) . (bxd)-f(axb)

or

towards b.

in anti-chockwise direction

or

plane of

n where

sin

angle between the directions of a and

the

-4

-sin 2

nl.

d)

d
d)

c)(b.d)-(b. c)(a

Vector Analysis

156
or

(ab sin

B)

(cd sin

Cancel abed and put


or

sin 2

or

cos

B - cos a
(A+B)

n=(ac cos A) (bd cos A)


-{be cos (A-B)} {ad cos

B)

n n=l as n is a unit vector,


A - cos (A - B) cos (4 +)
-sin 2

=cos B

Ex. 12.

-4

cos (^l-JB)=co& 2

sin

-4.

Prove that

(aXb)

axc) + (axb)

(aXc)=(a.a)

(b

c).

(Annamalai 39)
Ex. 13.

Prove that

2(cXd)X(axb)=

~d
where

(cXd)X(aXb)

(cXd)Xm=(m

c)

d-(m

d) c

[(axb).c]d-[(axb).djc
[abc] d-[abd] c
d-| 0, c

ft

a9

...(1)

[Cor. 4 P. 138]

Aagain putting

(cXd)=n and

proceeding as above,

(cXd)X(aXb)=

rf

...(2)

Multiplication of Vectors

157

we get
2(cxd)x(axb)=

Adding,

If

we expand

four determinants of

E%. 14.

(1)

Ci

C%

di

d%

c/

the above determinant,

and

we

get the

(2).

Prove that

(aXb)X(cXd)-f(aXc)x(dXb)4-(aXd)x(bXc)=:2[bdc]a.
(Andhra 38)
Expand ] st in terms of c and d, 2nd terms of a and c
and 3rd in terms of a and d etc.
Ex. 15. Show that the perpendicular distance of a point C
b X c+c X a+a X b
from the straight line through A and B is
b a where a, b, c are the position vectors of A, B and C.
-7The vector area of a triangle ABC is
|

I
Its

module

is

[Ex. 6 P. 128]

(aXb+bXc+cXa).

J base

ABx perpendicular

from C on

aXb+bXc+cXa ~ b-a
|

AB

CHAPTER IV
THE PLANE AND SPHERE
Vector equation of a plane.
Let there be a unit vector

(Agra 31, 39)

1.

A
n normal

to

the plane.

If

be the length of the perpendicular from origin on the plane,

Let
plane
f

But projection
and

it

P on

be the position vector of any point


then the projection of OP on OJV is p.
r

OP on OJV is r
top, we have

of

being equal

cos

0=r

cos

the required

the

A
0=r n
.

equation

.................. (1)
n=/>
the
for
standing
length of the perpendicular from the
p
of the plane as r

origin.

Cartesian form

Let the coordinates


and k through

If

/,

be

of

#, jy,

referred to

so that

m, n be the direction cosines of normal, then

A
A
;.

or

and

unit vectors

Ix+ny+ik-p,
i.

2
j

=.k=k.i*

i, j

The
which

is

Plane and Sphere

159
Coordinate

the standard equation of the plane in

Geometry.
Cor.

In case

be

there

any vector

parallel to unit vector

A
and of module

n, then

n=rc

n.

Multiplying both sides of

(1)

by

we

get

A
w(r.n)=n/>

Thus

or

nnp=q,

say,

where

= qln.

n=<7 represents a plane ; the length of the


perpendicular from the origin is obtained by dividing the
R. H. S. by the module of n,
r

R.H.S.
g_
n"
n
Module

i.e.

of

Cartesian to vector.

then

the

A-

vector

corresponding

(*i-b>j+*k)

(2i-f-3j4-4k)
-

from
r r
perpendicular

The

Cor. 2.

the plane be 2*-f 3?-h4e

If

equation

= 10 i.e.t.n^q
T-T

is

module

10,

evidently

and the length of


10

origin
b

is

r~n

ot

KM2 +3022 +4
r^a;
2

V(2

equation of a plane that passes through

a given point.

Let the position vector of a point A be a through which


the plane passes and whose normal is n. If r be the posiof any point in the plane, then
and
as
such n is perpendicular to AP.
plane

tion vector

AP

lies in this

->

Therefore

(r-a).n=0

or

The

^P.n0
or

n=sa

q say.

length of the perpendicular from origin on the

Vector Analysis

160

plane

is

a.n

a.n

|n|

._

^^

|*|

1 cos

s=*fl

Converse

n *j

0, i.e.

To prove

IV

a.n

the projection of 0-4 along


the normal.

that the equation r

n=q

repre-

sents a plane.

...... (1)

*.n=tf
Let a and b be the position vectors
A and B which satisfy (1).

and
*

Multiplying

(2)

by

and

(/wa+wb)

vector

is

nb
/wa-f
-

also satisfies

> T

19

it

...... (4)
whose

position

(1).

if

an y point on the

the ratio n

in

point

/1V

jsow

and divides

n== *

shows that the

(4)

...... (3)
and adding, we get

n=(m+)

i^r.

by n

(3)

ma+nb
Equation

...... (2)

a.n=0
b.n=#

/.

any two points

of

..

line

joining a

Thus we observe

m.

that

every point on the line joining A and B satisfies equation


which therefore should represent a plane.

(1)

Two

sides of a plane : The points whose position


are a and b lie on the same or opposite sides of a plane

Cor.
vectors

and b

t.n~q

3.

according

asa.n

and

q are of the same or

opposite signs.

Let the line joining A and B intersect the plane at P


which divides AB in the ratio n m and therefore its
:

The
position vector

Plane and Sphere

-----

is

161

Since this point

lies

on the plane

n=?
m

or

n - q)~ - n

(a

or

(b

n - q)

n q
b .n-g

=_a

is +ive, i.e. a . n-# and b . n-g are of opposite


P divides internally the join of A and B, i e.
then
signs,
A and B are on the opposite sides of the plane.

If

n\m

If

is

n\m

i.e.

ive,

a.n

</

and b

# are of

same

sign then P divides the join of A and B externally, i.e. A


and B are on the same side of the plane.
Hence Proved.

Ex.

Find

1.

2i-{-3]~ k

the

equation

and perpendicular

the

of

to the

plane

vector

the point

through

Deter-

3i+4j-f7k.

mine the perpendicular distance of this plane from origin.

Here n=s.Si+4j + 7k and

a2i+8j

Now

equation of the plane through a point a


(r

a)

n=0

or

Also

/>=*^~.

In cartesian form its equation


2.

f
i.e.

n=a

r.(3i+4j+7k) = ll.

or

We
n=
t

k.

ft.n=6 + 12-7=41 and/zV(3 2 +4 2 +7 2 )==8...(l)

.'.

is

n.

is

[Cor. 2]

[from(l)]

[from

(1)]

3^+4y+7-c=ll.

Equation of plane satisfying the given conditions.


have

already seen that the equation of a plane is


and a plane through a given point is (r-a) . n=0,

n=a

Here we

n.
shall

deduce the equation of plane satisfying


by the help of vector and scalar

other given conditions


triple product.

162

Vector Analysis

Equation of a plane through three given points.

I.

(Agra 51)
Let the position vectors of any three points A, B and
C through which the plane passes be a, b and c respectivebe the position vector of any point Pon the plane.
the points P, A, B, C all lie on the same plane, i.e.,

Let

ly.

Now

AB and BC

b-a

and

c-b

are coplathree vectors are coplanar, their


scalar triple product vanishes.
[Cor. 1 P. 132]

vectors PA,

Now we know

nar.

(r-a)

or

/.

[b

V bxb=0

xc-axc+axb]=0,

(bxc)+a
.

(cxa)+a

(axb).

[bxc+cxa+axb]=fabcj

-axc=cxa

V
V

if

.[bxc+cxai-axb]
a

that

(r-a).{(b-a)X(c-b)}=0

/.

or

r-a,

i.e.

and a

scalar triple product

(cxa)=0*=a

zero

is

(1)

(axb),

when two

vectors are

[Cor. 2 P. 132]

equal.

The equation

(1) is

of the

form

n=<? and

is

therefore

the required equation of the plane.

The

plane

is

clearly perpendicular to

n=sbxc+cxa+axb
which

is

equal to

-">

whereas

Note.

2&ABC

N=p

(Q. 6 P. 128). If p be the length


on this plane, then

ON from origin

of perpendicular

n=/>

Equation

fabcl
L

n9

^(bxc+cxa-J-axb)

(1)

called

is

equation of a plane through

we have

already

found

[from (2)]
non-parametric vector

three points, a,

the

corresponding

b and

c and

parametric

The Plane and Sphere


equation of the plane in Chapter II

163

Cor. 2 P. 82 as

9,

r=a+5(b-a)+/(c-a).
Multiply both sides of above scalarly by

bxc-fcxa+axb.
r

(bxc+cxa+axb)=a (bxc+cxa + axb)


+5 (b- a), (bxc+cxa + axb)
.

+t (c-a) (bxc+cxa+axb).
scalar triple product is zero when
,

Now we know
two

that

[Cor. 2 P. 132]

of the vectors are equal.


coefficient of s is

/.

b (cxa)-a
.

(bxc)=[abc]-[abc]

0.

zero and hence

we

Similarly coefficient of
r , (b

which

is

same

/ is

X c+c X a+a x b)=a

as equation

(1)

(b

get

x c)=[abc]

found above.

Corresponding Cartesian form.


Let

in

terms of unit vectors


a^sflji+^j + ^ak,

then

(r- *)=(*-*!)

i, j,

and

r=#i+^j+k;
l+(j-aj }+(z-aj k,
5-<2 3) k,
fa

Since r-a,

b-a,

c-b

are coplanar,

^3)

k.

we have
=0

as the required equation of plane.


II.

Plane through a given point and parallel to two

given lines.

lines

Let the plane pass through the point a and


which are parallel to b and c.

parallel to

164

Vector Analysis

parallel to b and c is therefore


the required plane is one
hence
bxc,
perpendicular
a
and
to
bxc and its equation
through
perpendicular

Since the plane

is

to

therefore

(r-a)

is

or

(bxc)=(X

(bxc)=a (bxc)=[abc]
The corresponding parametric equation of the
plane is r=a+j b-f t c [Cor. 1 P. 81] and on multiplying
both sides scalarly by bxc, we get the form (2).
The plane containing a given line and parallel
III.
r

Note

('2)

to another line or perpendicular to a given plane r . c=</.


Let the plane contain the line r=a+/b and is parallel
to c which means perpendicular to plane r . c=^.

Thus the plane contains the point a and is parallel to


both b and c and therefore perpendicular to bxc and hence
its

equation

is

IV.

a)

(r

(bxc) = a

(bxc)=0

(bxc)=[abc].
The plane through two given points and para.

llel to a given line.

(Pb. 60)

Let
plane pass through two points a and b and
given line be parallel to c. Thus the plane is one through
the point a and parallel to b a and c and therefore perpendicular to (b a)xc. Hence its equation is
the

(r-a).{(b-a)xc)
or

{(b-a)xc}=a

{bxc-axc} = a

The plane containing

V.

a given

(bxc)=[abc],
line

and a given

point.

Let the
plane

and

is

be

through

parallel to a

Hence
or

line

its

r=a+'b and

a and

given point c so that the


c and parallel to b i. e. through a

c and b or perpendicular to

equation

is (t

a)

[(a

(a

c)

X b.

c)xb]0

,[(a-c)xb]=a .[axb-cxb]
= -a. (cxb)=a. (bxc)=[abc].

The

Plane and Sphere

165

Angle between two planes.

3.

Let the two planes be r n l ^q l and t . n 2 q2 and the


moduli of n t and n 2 be n t and n 2 respectively.
Now angle between two planes is equal to the angle
between the normals to the planes. If be the angle between n x and n 2 then n x . n 2
cos 0.
.

=i2

Angle between a line and a plane.


be parallel to b and the plane be r . n=#.
angle between a line and a plane is comple-

Let the

Now

line

and the normal

to the

be the an^le between the line and plane and

$ be

ment of the angle between the

line

plane.
If

the angle between normal and line, then


n b=ft cos $.
Now
;.

<= n

cos r

0.

nb

=rcos (90-0)=sin
v

~ Sm_
.

4.

0=90

0.

b
9

nb'

Intercepts on ax:es of coordinates (rectangular).

Let the equation of the plane be r . n=.


Let the unit vectors along the axes be denoted by

i, j

and k respectively. If x be the intercept made by the plane


on the axis of x, then the point xi lies on the plane.
/.

xi

n=

<?

or

x=r-*
i

Similarly the intercepts on

and

T7H
Ex.
intercepts
all

2.

on

respectively.

the^ and

-axis are

k !n'

Prove that sum of the reciprocals of the squares


rectangular

axes

made by a

of the

fixed plane is same for

systems of rectangular axes with a given origin.

The Plane and Sphere

166

Let the plane be r 11=5,


If x, y and * are the intercepts on the axes, then
.

-JL
the normal

If

then

cos

Also

and

-^i

makes angles 6l9

^+cos

+cos

= 1,

i.e.

n=l
n=l n cos X
k.n=l.KCos03
.

2>

(cos2

and

k,

^ +cos2

ei+cos2

constant because the plane

l*+m*+n*=l.

[from
i.e.

i,

...... (2)

'

-72+72+ ?- ?

with

n cos

8%

ea)=s

f'(1)

and

(2)],

is fixed.

Perpendicular distance of a point from a plane.

5.

(Agra 41)

Let the equation


plane be

of the

n=j, so that

per-

pendicular from the origin


on it, i.e. ON=p=qln.

We have

TT/

find the per-

to

pendicular distance of a point


A, i.e. a from the given plane.

N
Fig. No. 83

Now

consider a plane through

to the given plane


(r

whose equation
a)

n=0

or

the point

is

AI

a.n
--

Also

r.n=a.n.
from

parallel

is

ON'p'*= length of perpendicular


plane
r

A and

Tr ?-a,n n -^j-a.n
AM~*
=

n.

origin

on

this

The

Plane and Sphere

Whenever we have

167

to specify a vector,

we

multiply

its

magnitude with a unit vector in that direction

AM=*

or

The above

value of

~
2

n2

AM

positive for all those points

is

on the same side of the plane as the


ive.
points on the opposite side it will be
which

lie

For

origin.

Alternative Method.

Let
then

its

AM be

perpendicular from a

equation

is

r=a+/n.

. .

.(1)

and being perpendicular to plane

M
hence

the

is

we

intersection of

is

to the plane r

n=0

AM passes

through a
to
normal
n,
parallel
as

this line

and the plane and

should have

(a-fm)

n=

or

*=?~

or

2
[V n =

n-ffn =fl,

Vn

2
]

ra

Putting the value of

in (1),

we

get the point

M as

n-a

n*

HA|-'-i- .|n|-t^?-a. n
Ex.
(2,1,

Find

3.

-4)
the

Show by
are

vector method tliat the points (1, 1, 1) and


on opposite sides of the plane

perpendicular

3*+^+5=9.

distance

of the

former from the plane and

the vector through it perpendicular to the plane.

Vector Analysis

1G8

In terms of unit vectors

i.

k,

j,

A=*i+j+k, B=2i+j-4k,
and the given plane is (*i+j>j-f k) . (8i+4j + f)k)=9 so that
2
2
2
n=s3i+4j + 5k and * = V$ +4 +5 )=5V2.

Now

perpendicular distance of a point from a plane

#-a

is

n
/.

___

perpendicular distance from A,

i-f j

5V 2

is

~5V2

Perpendicular distance from 5,

+k

"

~~

5V2

i.e.

2i+j-4k
9~(G+4-20)
"
i.e.

is

19

"

^-a.n^=-3

Again since
are

and

opposite signs, therefore


opposite sides of the plane.
,

of

Also the vector through

(b)
equidistant

Show that
from

opposite sids

Cor.

of

x.^.

two points

the

[Cor.

on

P. 160]
is

and 3 (i+j k) are


7k) 9=0 and lie on

i-j+3k
(5i-f2j

they

are

perpendicular to plane

the points

the plane

5V2
^-b.n=19,

(Agra 59)
Distance of a point from a plane measured in
it.

a given direction.

Here we have

to find

the distance of the point A


from the plane but mea-

sured in a given direction

say of the unit vector b.

Fig H*l

A
Let a

line

through

parallel to unit vector

b meet the plane

The Plane and Sphere

in

M and
on

-*
let

d be the length

AM so

The
lies

109

position vector of Af,


the plane r . n=q.

f.

that

AM =*d

->--

e.

b.

OM=OA+AM~a+d

A
rf~q-a
b
which

the required distance.

is

Ex.
plane

Find

4.

x+y+z=*5

The
The

the distance of the point


measured parallel to the line

given point

(1, 2, 3) is

given plane

i+2j+3k=a

(^i+^j+^k)

is

(1, 2, 3)

from

the

say.

(i-J-j+k)=5 so that

n = i+j+k and q=5.

The

direction ratios

a vector in this direction

of the line
2i

is

are

Hence

the distance of

and hence

+ 3j-6k=b.

a unit vector in this direction

.*.

2, 3,

A from

is

the plane

is

g-a.n 5~(i+2j+3k).(i+j+k)
>(2i+3j-Gk) .(i+j + k)

A
b

6.

Planes bisecting the angles between the given

planes.

Let the equations


r

two planes be
and r . n a =sv

of the

njssr?!

Vector Analysis

170

Now

if r be
any point on the plane bisecting the angle
the
between
given planes, then the perpendicular* distance
of t from both the planes should be equal.

.M.

Above
7.

are the required equations of the planes.

Plane through the intersection of two given

planes*
,

Let the given planes be

Consider the equation


wheie A is any constant

Now

all

those

planes also satisfy

(r

n 1 ^q l and

nj-^-A

(r

t
.

n 2 =^2
n 2 ^q z)^0.

.(1)

points which satisfy both the given


the equation (1) for all values of A and

the points on the line of intergiven planes.


Rewriting (J) in the form
we
the
(nj-Analft-A^g,
get
required equation of

as such

it is

satisfied

by

all

section of the
t

plane through the line of intersection of the given


The value of A is found by an additional given
planes.
condition of the question.
For example, if the required
the

plane passes through a given point

a*

a,

then

n-

In particular, if the required plane passes theough


q^qt and hence the
origin, then putting 'a=aO, we get A

The Plane and Sphere


equation of the
r

plane

is

q 2 (f

n t - qj - q l

171
.

(t

n2 -

<? 2

)~0

or

.(jtih-ftii^B-oLine of intersection of
8.

two planes.
(Agra 51, 55, 61) [Read

Let

te

equations of the

t.n 1 =q l
The

line

two planes be
and r . n 2 =<72

intersection

of

common

being

carefully]

to

both the

planes is therefore perpendicular to both n a and n 2 the


normals of the two given planes. But n x xn 2 represents a
vector perpendicular to both b^ and n 2 and hence we

^Xfig- In order to
determine its equation completely, we must know a point
on it. Now if JV be the foot of the perpendicular from
conclude that the

on the

origin
of

then

and n a

is

parallal to

then clearly Off is parallel to the plane


we denote the position vector of JV by a,

line,

If

aas/^+^ng, where

/ x and / 2 are any constants.


on both the planes. .'. li^+lf^ should
both the planes and this will give us the values of

Now
satisfy
/t

line

and

JV lies

12 .

n i2 +/2 tti n 2 tfi


(!)
2
n
n
. n2
or
-(2)
l^ 2 +;2 2 =#2
tfiih+^ng)
=#
2
Multiplying (1) by n 2 and (2) by n x n 2 and subtracting,
we get h [n^n^-Cii! . n 2 2 ]=^n 8a - ^^ . n 2

ni

tfini-H 2 tt 2 )

".

=tfi

or

'

:==:

fi

Having found
JV on the

li

of point

hence

its

and

line

/2 ,

we know

which

the position vector

parallel

to

n x X n 2 and

is

equation

r=:/ 1 n 1 +/ a n 2

Note,

is

Above

line of intersection.

is

called

+m

x n2

paranretric

equation of

the

Vector Analysis

172

We

can however find the non-parametric equation of

the line as follows.


Equation of a

line through a given point

and parallel

to

a given

line.

Let

be any point on the line which passes through a


r
a and b are
.*.
given point a and which is parallel to b.
two parallels and we know that cross product of two parallel

vectors

is

zero.

(r-a)xb=0

/.
is

or

rxb=axb

the required non-parametric equation of the line.

In case the line passes through origin,


a=*0,

we

get

its

equation as

perpendicular to c and d, then


b by cxd
parallel to cxd and hence replacing

In case the line


clearly

it is

we

its

get

then putting

rxb^O.

is

equation as (r-a)x(cxd)=0.
the non-parametric vector equation of the line of

Hence

intersection of

two planes

is

(r-a)x(n 1 xn 2)

where

a/

It

1 n 1 +/2 n a and

may

Il9 1
2

(2)

have values found above.

be observed here

as in Note 102 that the non-

line can
parametric form of the equation of

be

easily

deduced from the corresponding parametric form

r=a+*

b.

6 P. 47]

Multiply both sides vectorially by b.


.'.

rxb=axb+/bxb=aXb.

/.

(r-a}xb=0

Ex.

5. (a)

planes r

The

Find

is

the required equation.

the equation

(3i-i+k)- J and r
line of intersection

of the

line

of intersection of the

(l+4}-Zk)2.

is clearly parallel to

(Agra 45)

na X n 2

The Plane and Sphere

173

(2-4) i+[l-(-6)l J+[12-(-l)] k

or

-2i+7j+13k.
we must know a

In order to find the line

point on

it.

Let a be the foot of the perpendicular from origin on it;


it is
expressible as a linear combination of n x and

then since

n 2)

let

a=J (3i-j+k)-K2

Now

and

(i-f-4j-2k).

on both the planes.

lies

(3i

[/,

Solving the above two,

.27

we

get

k-

and

H/,-8/,-1

.'.

/2

Having found the point and


the line

is

given by

r=a+*

=.
25

and

direction, the equation of

b,

05
f-- (3i-J+k)+^j(i+4J-2k)+/
(2i+7j+13k)
of,

i.e.

Above

is

Again

the parametric equation of the line.

in order

to find

the non-parametric equation of

the line,
(r

Ex.
jf,

7,

5.

(b)
1)

T
is

[Bottom P. 167]

rx(-2i+7j+13k)=5ix6j+4k.

or

- a) X (% X n 2)=0

Prove

that

and containing
.

the

the line

(i+3j-k)-0W

plane

through

of the planes

r.(i+2j-3k;0.

the

point

174

Vector Analysis

Perpendicular distance of a point from a given

9.

line -

(Agra 49)

Let the given

through

passing

the point

and

parallel to

We

unit vector b.

are

the perpendicular

tance of any point

because square

to
dis-

C whose

line i.e.CN. Clearly

vector

position

c from this

is

C^O^-OC=a-cand G',4 2 =*(a-c)

of vector is

square of

its

...(L)

module.

NA

projection of CA along unit vector


therefore equal to CA cos 0=*l .CA . cos 0.

Also

is

r=a+/b

find

be

line

is

A
CJ\T

b and

it

A
A
- c) a - [b

=CA z

(a

- c)] 2
from

(1)

and

(2),

Also

=(a-c)-[b,(a-c)]b

because JV-Assb .(a- c) and


A
.'.

In case

it

being in the

direction of b,

NA=[\>. (a-c)]b.

b be

not a unit

vector then

we

shall replace

A*

b by

b/A

Ex.

where
6.

find

is

the

module

of b.

the perpendicular distance

cube from a diagonal not passing through

Since the cube

is

a unit cube,

it.

let the

of a corner of a unit

(Agra

33, 41, 56)

vectors determined

The Plane and Sphere


by conterminous edges OA,

OB and OC

175

be

i,

and k respec-

lively so that the diagonal

through

OP

b=i-f-

is

j+k which

passes

origin.

Fig. No.

86

unit vector in this direction

If

CM be

perpendicular from

OM= project ion of OC

on

is

e.

k on OP, then

OP
-

s=sk
.

Also

10.

may be

unit vector along

i+j+k

OCk;
To

i. e.

OC

.-.

-1.

find the condition that any

coplanar

Let the given


and

OP

i.

e.

they

lines

may

two given

intersect.

be

passing through a l9 a 2

and parallel to bj and b2


respectively.

In

case

they intersect, then their


common plane should be

Fig.No.Q7

lines

Vector Analysis

176
parallel to a 1 ~a a bj
,

and b 2

i. e.

be coplanar, the condition for which

(a!-a 2)
or

aj

or
is

(b 1

xb 2)0

is

that

[Cor. 1 P. 137]

X b 2 )a a (b x x b 2
[ib 1 b]-[a I b 1 bJ
.

(bi

vectors should

these three

......... (1)

the required condition.

The

plane through the coplanar lines.

In case the given lines intersect, then condition (1)


holds good and in order to find the equation of the plane
through them, we write the equation of the plane through
the

and parallel to b x and b a


b t and b 2 it is perpendicular to

point

parallel to

perpendicular to

Since plane

is

biXb 2 which

is

r-a x any
,

dot product of

line in the plane.

two perpendicular vectors

is

zero,

or
or

......... (2)

[fbibjdOi^bjs]

the required equation of the plane and it


will pass through the point a 2 if [a a b 1 b 2 ]=L a it i^2] which
we know is true by virtue of (2). Hence (ty represents the

Above

is

required plane.

Note

considering the
equation as

By

also write its

plane

through a 2

we

could

rrihb.Ma.bjb.l.

Corresponding cartesian form.


[See author's Solid

Let
i, j,

in

Geometry P.

125]

terms of any three non-coplanar unit vectors

along the axes,

and
and
so that the two lines are r=sa1 +/b 1 and

r=a a -f /b a

The
the

Taking
rsss*i-fjj+k,

The
where

we

get their equations as

a 1 =-a 2 =(Ar 1

#a ) i+(^i

^2)

where

form

cartesian

corresponding

condition for coplanar lines

The above

177

Plane and Sphere

(Cor, 3 P. 48)

is

+ (i

2)

k-

means that

condition

[Cor. 4 P. 138]

ijk]^0 but equal to

1,

the required condition

/.

Also the equation of the plane containing them

(r-a^

(b 1

xb )=0
2

or

is

is

(r-aa).^;

or

=0

or

=0.

Mr. N. Sreekantr

O.U.

11.

Shortest distance between two non-intersect-

ing lines,
Let the equations of the
?i and t

(Agra
lines

be

37, 39, 42, 52)

178

Vector Analysis

Let

P& be the

Since

P^

At

whose module

Al

perpendi-

bjXb^n

to

Let

is

b2 and b 2

cular to both
parallel

__

between the given

distance
lines.

shortest

it is

A2

say

fig.

is n.

i.e.

A2

a x and

i.e.

any two points on the

a 2 be

given lines respectively, then shortest distance


a 2 on
tion of AtAz on PiP& i.e. projection of a x
/aj

Thus

in

intersecting

No 68

a.j) .

(aj

a 2^

order to find the

lines

parallel

(b,

S.

is

the projec-

D. between two nonto bj

respectively

and b 2 you
,

should find a point on each of the lines then find th


jection of the line joining these points on b 1 xb.2
;

pro-

Equation of the shortest distance.

The

equation of the shortest distance

intersection of the

is

the line of

planes through the given lines and the

shortest distance.

The plane containing


parallel to b, X b 2 is
(r-aO

The

t^3L 1 +tb 1 and

f^xfoxb^-O.

Note.

ween them
or

plane containing r=sa 2 +.sb 2 and S. D.

(r-a^.foxfoxbjM-O.

aa

S.

In case
is

as found in

is

2 III P. 161]
is

2 III P. 164].

the line be coplanar, then S. D. bet-

zero and as such (aj

(bj X b 2 )=a 2

D. which

(b x

x b 2 )=0

or

10.

Corresponding cartesian form.


Resolving in terms of unit vectors

i, j

and k as

in

10,

The
S

179

Plane and Sphere

b2]
'

b x xb 2

(b,

bjf-W sin

.'.

Alternative.

putting for

The equation

cos

b x xb 2

bX b

On

0-

(1)

X b2
b!Xb 2

of the S.

in

D.

is

<we get S. D.
the line of intersection of
(1),

planes.

.e.

-*i

and

- a2

bi
b2

b 1 xb 2 j=0
biXbaJasO,

=0

<;-

jr-j^

i.e.

m,
112/1

and

Ex.

7.

77z* shortest

angular parallelepiped

distances

diagonal of a rect-

between a

whose sides are

a,

b,

and the edges not

180

Vector Analysis

meeting

it

are
be

ca

ab

(Agra 33, 60)


[See Author's Solid Geometry]

Let the unit vectors along

OA OB and OC

Now

OC=c

CMssfl,

vectors of A,

OBssb and

B an i C

are

Fig. No

ai, b]

be

i, j

and

and therefore position

and ck respectively.

89

Now we have to find the shortest distance between


diagonal EC and the edge OB which does not meet it.
7

Shortest distance

is

aa0, a point
a'=ck, a point
Also

b and
on

the

a' are

b' are the directions

OB

C on EC.

bxb'==05xC*jX(ck-fli-6j)
k,

bxb'
:.

where a and

i~h~~h'~1

points on each of the lines and


of the lines.

and

k.

S.D.=

V jxk=iandixj=-jxi=k.

The

Plane and Sphere

181

abc

Similarly

we can

find the S.

D. between other diagonals

and the edges.


Exercises

Ex.

Prove that the equation of the plane through the point

1.

and perpendicular

(1, 2, 3)

to each

of the planes

+ j+k)=3 and r

.
(2i+3]+4k)=0 is r (i+6}+5k)=28.
n be the normal to the required plane, it is perpendicular to both b and c, the normals of other given planes, and
hence parallel to b and c. If a be the given point, then the
(r- a) (b X c)=0
[Cor. 2 P. 159]
plane is

(i

If

or

Ex.
(1 9 2,

2.

[rbc]=[abc],
Find the equation of the plane through

and perpendicular

1)

(3i- j+k) = J and

line

of intersection ofplanes

(i+4j-2k)=2.

The

given point
5 P. 173 the
line

Ex.

to the

the point

is

i4-2j

k=a

say

intersection

of

is

parallel

-tH+7j + 13k=n
plane

its

say and it being perpendicular


is
given by (r a) n=0.
equation

to

to

the

r.(2i-7j + 13k)-l.

Ans.
Ex.

from

and

3.

Find

the

(i-|-3j_k)=0 and

of the plane

line

containing

(~1 -1,-1).
The line of

intersection

of

(j+2k)=0 and

the above

line

of

the

planes

hence find the equation

and through

the

point

intersection

(i

since
line

both the
also

Ans.

parallel to

planes pass through origin, and hence the

passes through origin.

through (-1, -1, -1),

Now

is

+3 j_k)x(j+2k)=7i-2j+k

i. e.

The

-i-j-k.

use case V. P. 164.

r.(i+2j-8k)*0.

plane also

passes

182

Vector Analysis

Ex.
*

4.

Prove

(i- j-Hk)=2

the

that

and hence show that

the given planes

a point on
plane
perpendicular to the normal.
i.

Ex.

intersection lies in the

of

plane and

the

satisfies

Prove that the line of intersection

5.

the

that

it

e.

and r

r.(i+2j4-3k)=0
Show

(2i+5j+3k)=0 and

and the plane

have a common line of inter setion.


Hint. Prove that the line

third

planes

intersect in the line

1
angle \ sec"

line

is

3 with

(3i+2j+k)=0

Angle between

and

(Agra

j.

line

and

is

planes

rf (i-2j+k).

is

inclined to

equally

the

of

it is

and makes an

55, 61;

Utkal 53)

given by

= ab cos 0= v'(l+4+l) 1 cos


-2=V6cos0; ^ C os0=-2/\/6.
cos 20=2 cos 2 0-1=2 f-l=i
/.
sec 20=3 or 0= sec- 1 3 etc.
;.

(i-2j+k)
or

Ex.6.

[rnc]

Prove that the plane through the point a paralltl

r=b+/c

line

the

and

perpendicular

to

plane

t.n=q

to
is

[anc].
If

n x be the normal to the plane, then it is perpendicular


n and c and hence parallel to nxc and therefore

to both

the required plane

Ex.

7.

Ex.
the line

8.

of

a)

the

plane r

Find

equation

the equation

intersection

parallel to the line

of

of

of

the planes

intersection

r.n 8 =08
Any

(nxc)=0.

of the line through the point a


and
n=#
perpendicular to line ==b-f tc.
or
r=a-f-/nxc
(r-a)X(nxc)=0.

parallel to the

Ans.

Find

is (r

plane which passes through

the

ii,**^, r

n 2 =? 2

ana>

*5

of the planes
,

r.n 4 =04

plane through the line of

intersection of given

The

Plane and Sphere

183

%-^+A (r . n 2 -02)=0
*(!!!+ An,) saft+Aj,
(1)
Above plane is parallel to n3 Xn 4 i. e, the line of
intersection of r n 3 =0 3 and r n 4 =0 4 and hence perpendi.

planes
or

is

(r

cular to normal.

(iij+An^

/.
.

tij

(n 3 xn 4 )= -A

X n 4 )=*0,

(n s

{(n 2

(n 3

xn 4 )}

J W*]-A.
[WUl
r

Hence

the required plane

(r

Ex.

Find

9. (a)

parallel lines

is

nj-ft) [n 2 n 3n 4 ]=(r

r=a-f /

the equation

b,

r==c+/

n,- j,) [njn 3 n 4

of

b.

Clearly plane passes through a and

andb.

II

[Case

].

which contains two

the plane

is

Ans. r . {(a-c)xb}+[acb]=0.
Ex. 9. (b) Find the equation of the plane which
line

r=f a and

is

contains

the

parallel

to

158]

perpendicular to the plane containing

r=j b

snd

r=

c.

plane containing r=^ b and * =k c will be perpendicular to bxc. The required plane being perpendicular to above plane is therefore parallel to bxc. Also it
Hence it is perpendicontains r~t a, i. e. it is parallel to a.

The

cular to

ax (bxc).

equation

Ex.

Since
f

is

10.

Prove that

passes through origin; hence


a X (b X c)=0.
it

the lines

rxabxa and rxbaxb


intersect ,

and find

The

first

and parallel

their point

line is

to a

and

(r-b)XasO
its

will intersect

(a-b)

(Pb. 60)

of intersection.
i.e.

a line through

parametric equation

Similarly the 2nd line

They

it

if

(axb)0,

r=a+j

is

[a-b,
a

a,

is

fssb-H

b.

b]=0,

(axb)-b . (axb)0,

a.

Vector Analysis

184

which

as scalar triple product

true

is

zero

is

when two

vectors are equal.

For
value of
their

in

we should have identical


which on comparing the coefficients of a and b
=
equations w e have 1=^ and l f and we get the

their point of intersection


r for

a+b.

required point as

Ex.

Prove that the lines

11.

r=a-f/(bxc) and
will intersect ij a

Ex.
relative to

DC

c=b

t=b+s

(ex a)

c.

be the position vectors

an origin 0, then

interpret geometrically

12.

//a, b,

c,

of four points
the equations

(Pb. 60)
(c-d)x(axb)=0, (c-d).(axb)=0.
OAB
of
isr,(axb)=0. Also
plane
Clearly equation
Cross product of two vectors is zero when
is c
d.

they are parallel.

CD

.'.

is

parallel

to

axb

which

is

normal

to plane

OAB.

CD is

Hence

normal

Dot product
Therefore CD

is

plane OAB. or

is

Ex.
of

the

13.

squares

is

to plane

zero

when

OAB.
vectors are perpendicular.
it lies in the
i. e.

perpendicular to normal,
parallel to it.

Prove that the locus of a point such that the difference

of its distances from two given points

is constant is

plane perpendicular to the lines joining the points.

Ex.14.
point a and

Find

the

equation

intersecting both the lines

Let the equation

of the

r=c+/ d

and r=c'+/'

(Agra 46, 55, 61,


be

d'.

Delhi 51)

of the given line

r=a+$b
and the given

(Agra 43)

straight lines through the

(1)

lines are
(2)
'

(3)

The Plane and Sphere


Since

and

(1)

we have

(2) intersect,

and

also intersect

185

[b d

c-a]=0

[b d'

c'-a]=0
b . {dx(c-a)J0 and b (d'x(c'-a)}~0.
Above relation shows that b is perpendicular to both
dx(c-a)and [d'x(c'-a)].
bis parallel to [d x (c - a)] x [d' x (c' - a)]
/.
Hence required line is
Again

(I)

(3)

or

/.

<w

r-a+t[{dx(c-a)}x{d'x(c'-a)}].
(b)

Find

parallel to the

the

straight

plane r

asssfl,

through the point

line,

and

intersects the line

c,

which

is

r-a'=s/b.

(Agra 58)
Let the

line

be

rc+/d

It is parallel to r

a=sO

i.

and

(1)

[dba -c]-0
show that d

(2)

bx(a'-c) and hence


line is

/.

Ex. 15

normal

a.

r=a'+rt>.
f

.'.

JL to

e.

c.

d.a=0 ............ (1)

.'.

It intersects

passing through

or
is

[bx(a'-c)]=0

perpendicular to

...(2)

both a and

parallel to

ax{(bx(a'-c)}.
r=c4-*[ax{bx(a'-c)}].

Find

the

point

intersection

of

of the planes

t . ==&, r . n a =^ 2 r n 3 =$ 3
n lt n 2 n3 are three given non-coplanar vectors i.e. [n 1 n 2n 3 ]7^0.
Since n 1? n 2 n 3 are three given non-coplanar vectors,
ntXftg, n 2 xn s n 3 xn! are also non-coplanar Ex, 5. (2) P. 151
and we know that any vector can be expressed as a linear
,

where

combination of any three non-coplanar vectors. Let the


position vector p of the point of intersection be expressed as

where

/,

m, n are to be found.

of the three

product

Now p

satisfies the

equations

given planes and also noting that scalar

when two

vectors are equal

or

is

zero.

triple

186

Vector Analysis
Similarly
J

m=

Ex.

16.

the origin and

C planes
the locus

variable

Let

let

to

is at

Hence p

?.

is etc.

distance

constant

p from

Through A, B and
Prove that
the coordinate planes.

and C.

intersection is given by

[See Author's Solid Geometry, Q, 7 (b) P. 61]


the equation to the plane be
r

and

plane

parallel

of their point of

and ns=Y

axes in A,

meets the

drawn

are

n=q

...... (1)

where /?=

n be
on the axis

Its intercept

q
i

/.

A= ny

n~~i
i;

similarly
"

B= nq

a)

(r

Similarly

plane through

Now

1=

planes

C=-~

and

n=0

q-

or

is

or

through B,

and

parallel

to j-k

given by

n=a n

k.

n3

i.e.

plane whose normal will be along

any plane through A,

or

is

Now

of x

1=

parallel

to k-i-plane

and

parallel to i-j-plane are

x,y,z be the coordinates of the point of intersection then xi+y)+zk will satisfy the above three planes.
if

The
jy=~Similarly
* '

Plane and Sphere

187

(1).

Ex.

17.

of

the centroid

If

.v,

is at

variable plane

^4

and meets the axes

the origin

in A,

of the tetrahedron

distance

constant

OABC

is

[See Author's Solid Geometry, Q. 7


z be the centroid, then

}\

Equating i, j,
Ex. 18. Find

we

k, etc.

get

#,

y,

and

(a) P. 60]

etc.

locus of a point which

the

p from

Show that the locus


2
*- +Jr2 +C~ 2 ==</6/;- 2

and C.

is equidistant

from

the three planes

11!=^,

P 2 =? 2 r
,

n 3 =2 3

(Lucknow

51)

Ans.
"2

Wj

Ex.
lines;

show
is

19.

is the

that

W(b

OB

0^,

length of

and

OC

are

three

mutually perpendicular

the perpendicular from

2
2
2
/r =fl- -|-- +r2

c*+c a*+a*b*),

and

a, b, c being

the

the

to the

area of the
lengths

plane

ABC;

ABC
OB and

triangle

of OA,

OC.

Equation of the plane through a, b, c is


r , n=[abc] where n=bxc-f cxa + axb

and

its

module being twice the area of the triangle whose


b and c and perpendicular from is

vertices are a,

[abc]

TnT
Here a = <d,
Ex.20.

3i+9j+4k

and

Find a
triple product

b6j

Prove

and c=dk

that

the

-i-fj+k

d,
is

b
zero.

d,

etc.

four points

4i+5j+k,

-j-k

are coplanar.

c-d

and prove that

their scalar

Vector Analysis

188

Ex.21.

Prove that

the perpendicular

the plane through the points a,

from

b and c

[abc] -[abd]+[acd]- [bcdj-Mb

Volume

whose

of tetrahedron

distance

of a point

is

X c+c X a+a x b).

vertices are a, b,

c,

is

J [(abc)-(abd)+(acd)-(bcd)]=:J area of
through a, b, c
X perpendicular distance of the point d from the plane

through

a,

b,

c=J

.*.

/>=as given

Ex.

22.

edges of a

Prove

regular

(bXc+cXa+axb)x/>.

etc.

that

the

tetrahedron

shortest
is

distance
to

equal

between

half the

square described on an edge.

We

know

opposite

diagonal of the

(Agra

50, 59)

that a regular tetrahedron can be inscribed

in a cube.

N
Fig No 97

Let the unit vectors along OA, OB,

OC

be

i, j

and k

respectively.

/.

OD=i+j+k. BACD

hedron and we are to

find

pair of opposite edges say

and

AD^OD-OA-.

is

the shortest

BC and AD.

the regular tetradistance between

The Plane and Sphere

Module

Now B

of
is

BC*AD=

-2i

any point on

189

=2.

BC and 4

a point on AD.

A AB=OB-OA=}-\.
Required shortest distance

PCX. AD.

the projection of

is

S.D..

:.

AB

on

11 P. 178]

\BCxAD
(j-i).-2i
1,

Now

ABss
|

.*.

diagonal of the square with one side

.*,

S.D.=l=f =J

AB

the diagonal of square.

Alternative Method,

Let

OABC

tetrahedron

be the

and

of vectors with a,

vectors of

-4, J5,

and C.

opposite edgs are

The two

OA and BC.

The equations

rfa

regular

be the origin
b, c the position

of these are

and r=(l

The S.D. between them

is

ax(c~b).b

(ax(c-b)|

ax(c-b)

08
|

aXc-axTT

d)

190

Vector Analysis

axc^^! OAC

n x where n x

&OAC.
A
A
axb=2^2 OAB n a where n 2
pendicular to the &QAB.

is

the

is

the

unit

vector

per-

pendicular to the

unit vector per-

A
Module

of

axc-aXb~module

2j 1 n 1 -2J 2 n 2
= V14J 2 (n 1 2 +n 2 2 -2n 1 n t )],
of

as Jjss J 2 being areas of the faces of the regular tetrahedron

where
cos

the

is

0=.
/.

module

TT

Hence

angle

of

between the two faces and hence

axc-axb=V(B4 2

'--T-TVs-?"
*

f)= fo

yo

rr

...... (2)

nxl

[from()]

"4"
'

12

half the diagonal of the square described on an edge.


Ex. 23. Prove that the S. D. between pairs of opposite edges
isosceles tetrahedron lie along the join of their mid. points and
an
of
that the three S.D.'s are perpendicular.
i.e.

Take one vertex as


vectors of the other be

b and

and the position

origin

a,

c respectively.

"

2
s

Lt5TLa

2
.

Ftg.No.MI

The Plane and Sphere


If

191

OA and BC then PQ, should be


OA and -BC and it will be so if
tb+c-a).a=0 andi(b+c-a).(c-b)0
a
(1)
b+a . c=a2
2
2
c -b =a. c-a. b
(2)

P(l

is

D. between

S.

perpendicular to both
|

or

if

and

Since the tetrahedron

isosceles,

we have

OA**BC, AB^CO, CA=OB


2
a s=(c-b)
or a =b 2 +c 2 -2b
2

or

(4)

and

we get (2).
Hence PQ,

is S.

Adding

2c

2a

Similarly

and

2
c=(b +c -a
2

2b

or

(5)

is

a=*(c*+a

-b 2

$)
(4)

2
s
b=(a 2 +b -c
we get (1) and

(5)

(5),

D. between

b+c
will be

OA and

subtracting

2a

if

and

(H)

a-b,

If

we can

C be

point

of opposite

a+b-c

if

b=a 2 +b 2

etc.

c2 which

is

true

by

(5).

prove other pairs to be perpendicular

the centre

on

the

sphere from the

the

a sphere,
distance of

of

surface

.centre

is

of

the

equal to

a.

Let the position vector of any


be r with
point P on the surface
respect

pairs

General equation of a sphere.


(Agra 35, 38, 43, 52, 60)

then by definition,

radius

and

(4).

12.

any

c-f

(4)

BC.

(b+c-a) . (c+a-b)=0
c 2 -(a-b) 2 =0
Similarly

by

a,

perpendicular

if

or

Similarly we can prove for other


The three S. D.'s lie along
edges.

and these

to an

origin

and that

of

Vector Analysis

192

the centre be

Now

OP^t

c, i.e.

CP= radius

module of

square of a vector

is

square of

We may

know

that

......... (1)
V c 2 =*c*.

-2r.c+c2 -<z2

-fl 2 =fc

and

becomes
r2

is

and we

module.

its

for the sake of convenience put

the above equation

Above

then

2
2
(r-c) =fl

.'.

or

and 0(7

......... (2)

-2r.c+*=0

a relation between the position vector

point on the surface of the sphere and


of the sphere.

is

of

any

called the equation

Corresponding cartesian form.


If

in

be the point

(#, y, z)

terms of unit vectors

i,

and

and

(c l9 c.Zy 3 ),

then resolving

k,

and

Squaring both

sides,

we

get
r3)

which

whose

is

the well

centre

is

at

known

CP

2
fl

cartesian equation of the sphere


and radius is a.

(cly c29 cz )

Particular case,
1.

In case the origin of vectors lies on the sphere.

The

In
i. e.

Plane and Sphere

module

this case the

of

from

coma and hence

OC

be equal to radius

will

we

(1),

193

get the equation of the

sphere as

-2r.c=0 ............ (3)


*c2 -a 2 a2 =a2 =0.
r

V
Polar form.
C.

(3)

s=r 2

and

c=rc cos 0=r0 cos

gives r*-2rfl cos 0=*0

r=20

or

cos 6

is

the

required polar form.

Cartesian form,
Putting the
i, j,

k,

we

values of

get from

and c

terms of unit vectors

(3),

* i +J 1 +* i

or

in

which represents a sphere whose centre


which passes through origin.
2.

If

the centre

In this case

is

is at

is

at

(c l9

c2

3)

and

the origin.

at the point

equation of the sphere takes the

C and hence

form

clearly

the

r 2 s=a 2 .

(r-a).(r+a)*0 ............ (4)

or

-*-V

positon vector of

and we know

is

a,

dot product of
two vectors be zero, then they are
that

perpendicular.

if

Thus AP is perpendicular

diameter of a sphere subtends a

riglit

to BP, showing that

angle at the surface.

Vector Analysis

191

General

Method

for

If

CM=a,

Above

-2r

k*=c z ~a z

c+/i=0 where

...... (5)

then

gives us the posi-

A and

tion vectors of

B.

be the position vector of any point

If r

(Agra 52)

of the sphere be

Let the equation


r2

above property.

Pon

the circum-

ference, then

_
_* _
^P-= OP- 0.4 = r - (c+ a)

->->->
and

J3jPOP-OBt-(c-a).
-

Now

or

4P, BP=(r-c-a) . (r-c+a)


2
2
2
2
r 2 -9r
or
(r-c) -a
c-fc ~0

or

r2

Hence AP

2r

c+fc which

is

zero by

(5).

perpendicular to BP, showing that a diameter subtends a right angle at the circumference.

Cor.

is

Equation of a sphere on the join of two

given points as diameter.

(Benaras 48, 54)

g and h be the position vectors of the extremities G


and
of any diameter and P any point on the surface
whose position vector is r, then Z.GPH=7r/2, i. e. GP is
If

perpendicular to

HP.
-

.%

i.e.

dot product of

GP and GH

is

zero

(r-g).(r-h)0.

The
Above

Plane and Sphere

195

represents the required equation of the sphere.


Points of intersection of a line and a sphere.

13.

Let the equation of


the sphere be

2r

a 2.

(r)=r

c+A;=0
....(1)

where

k*=* c

Let
through

whose

the

line

pass

the

point
position vector

is

A
p and be

a parallel

to unit vector q, so that its equation

A
r=p+*q
t

will

stand for the distance of the point

point on the

[6,

(2)

is

P. 46]

P from any

line.

intersection we have to
and noting that square of a
on putting the value of r from

In order to find the points of


eliminate

unit vector

between

(1)

and

is

we

get,

unity,

(2)

(2) in (1),

A
or

or

t*+2t q
*

A
+2q

(p-c)

A
or

+2q.(p-c)/+F(p)=0

(3)

Above being a quadratic in / shows that every line


two points which will be real if

cuts the surface of sphere in

A
[q

(p-c)]*

> F (p),

t. e.

Ba

>

4 AC.

196

Vector Analysis

and

If Q,

are the points of intersection, then the roots


(3) will be the values of distances of

of the above equation

from

Q and
PQ

;.

Above

/?, i.e.

PQ

and PR.

P/?= Product

result

of

roots^F

(p).

dees not depend upon the direction of

A
as

lines

PQ
the

of

q showing thereby
P/J=F (p) for all lines drawn in any direction through
point Pto cut the surface of the sphere.
it

is

independent

Square of tangent from any point.


and R coincide at any point T, then PQR
()
becomes tangent line PT and both P) and PR become PT.
Cor. 2.
In

case

PT.PT=F(p)

:.

or

PT 2

Thus square of the tangent from any point


sphere is obtained by
sphere, which

is

F(p).

substituting that point in the

a result

that

identical

is

surface of a

to the

with

equation of the

the corresponding

result of coordinate geometry.

Cor.

3.

Tangent plane

at

Author's Coordinate Solid Geometry.]

given point.
The same procedure

[Refer
is

adopt-

ed for finding tangent planes at any point to a conicoid.

Now let us choose that the point P is on the surface of


the sphere so that F (p)=0 and then one root of (3) will be
If the line through P
zero as PQ, will be zero in this case.
is to

be a tangent line then the other root of

be zero, the condition

for

which

(3)

should also

is

q.(p-c)0from

(3)

(4)

Now q

being the direction of line which

is

now a

tan-

and p-cis the vector joining the centre to the


gent
point P and since their dot product is zero we conclude that
tangent line is perpendicular to the radius through that
line

point.

The Plane and Sphere


All such

tangent lines will

lie

in

197

a plane whose equa-

A
tion is obtained

by eliminating q between

geometry by eliminating

we

/,

m,

i.e.

w,

(2)

and

solid

(4) (in

direction cosines)

and

get the equation of the tangent plane as

(t-p).(p-c)-0.

(5)

Above equation clearly represents a plane through the


point p whose normal is p c i.e. the line joining the centre
and the point of contact.

Now we know

that

remains unchanged

(p)=*0

we add F

if

and hence the equation (5)


Thus the
(p) in L. H. S.

tangent plane becomes

(*-P).(P-c)+F(p)=0
r

or

p-p

or

r
r

Above
Rule.
change one of

is

c+p c+p 2 -2p c+&=0


.

ip-(r+p).c+fc==0

(6)

the equation of the tangent plane at p.

In

the

the t's

equation

of

the sphere

by the given point

2
replace t by r

and

p and replace 2r by r-f t and


p and this rule is identical

change one of the t's by the given point

with the corresponding rule of coordinate geometry.

Cor. 4. Condition for any plane to be a tangent plane.

We have seen

that tangent plane at any point is perradius through that point and as such if

pendicular to the
any plane is a tangent plane then its perpendicular distance
from the centre should be equal to radius.

Let the plane be

nj and c be

the centre and a the

radius.
..

/'q-c.nY

_,
[

5 P. 166]

198

Vector Analysis

Ex.

Find

1.

the

coordinates

of

inscribed in the tetrahedron bounded by


r

i=0,

j~0,

k=0

the

centre

and

(i-f-j

+ k)

Also write down the equation of the sphere.

Let

(*, y, z)

the

of

sphere

the planes

0.

(Benaras 53)

the coordinates of the centre so that the

be

position vector of the point

is

xi+yj+^k.

Since the given planes are all tangent planes, therefore,


perpendiculars from the centre to all the planes are equal.

"
111

|T|

'Since perpendicular distance

tr

(3-

is

V 3)

not

we

ive,

(i+j+k)

get

a,,

Hence on putting the values


vector of the centre

of x,y

and ^ the position

is

-~(3~V3)(i+j-fk).

Hence the equation


[t
(b)

(j-

of the sphere

is

(r-c)

(8-V3)(i+j+k)J.[-J

=a 8

(8-

V8)J.

Prove that the equation of the sphere


circumscribing the

tetrahedron of part (a) is

{t-a (i+j+k)}=:(X

The Plane and Sphere


Cor.

199

Condition for orthogonal intersection of two

5.

spheres.
In case the two spheres cut each other orthogonally,
then evidently the tangent plane to one of them at their
common point of intersection will pass through the centre

Hence

of the other.

radii,

and

equal

to

r2

-2r

r2

-2r

. CJL

of the squares of their

1
4-^=0, where A^ss^-a

c 2 +#2 =0, where


(c1 -c 2

then
or

c^+Ca

-^

or

'Jcj.

2
)

=c2a -0 2

=fl 1 2 +^ 2 2

a
c,-<? 1 -A;1 +f 1

-*j

Cj^fcj+fcg.

Corresponding cartesian

Two

sum

the two spheres be

if

i.e.

distance between

square of the

the

their centres should be

result.

spheres

xt+y^tf+

and
will cut

one another orthogonally

if

2^ + 2zv + 2^2 = d
2

[Refer Author's Solid Geometry P. 213]

Ex.

2.

The

gonally also cuts

sphere which cuts

(r)-AF fr)=0

(Alld.

Let

M.

F^tjO and F (r)=0 ortho2

orthogonally.

Sc. 1960,

Agra

38, 46;

F1(r)r ^2r.c1 +A;1 =0


F2 (r)^ra - 2r c 2 +A2 =0
2

.\

F^fJ-AFtCr)-^-

2r

Benaras 55)

...... (1)
...... (2)

Cj

=ta (l-A)-2t . (^(3)

200

Vector Analysis

Let the sphere


cut the spheres (1) and

ra -2r.

...... (4)

c+*=:0

orthogonally.

(2)

...... (5)
...... (6)
Multiplying

(6)

2c

by A and subtracting from


.

(5),

we

get

(Cj-

or

Above
cut sphere

is

(3)

evidently the condition that the sphere

The

Defi.

The

is

may

orthogonally.

14.

a sphere

(4)

polar plane.

polar plane of a given point with respect to


the locus of points the tangent planes at which

pass through the given point.


Let the equation of the sphere be
a
F(r)=r -2t

The

it

c+*=0.

tangent plane at any point


r

If

is

given by

~(

passes through a given point d say, then

d.p-(d+p).c+*-0.
The

locus of point

P is

therefore

...... (1)
or

it

can be written as

r.(d-c)(c.d-fc)

......... (2)

Above equation represents a plane which is clearly


perpendicular to the line joining the centre and the given
point.

Thus

the polar plane

joining the centre

oftfo

of a point

is

perpendicular

to the line

sphere to that point.

the polar plane of point d cut the line joining


then CT is the perpendicular distance
the centre to d in

Again

let

The
of c

Plane and Sphere

from the polar plane

d as

of

polar plane.

d)

___
""

where

CD

this line

(d

--5-d c

whose position vector

The two

CD.CT^a
D and T arc

easy to

prove from

that

(1)

d.

if

e,

inverse points

the polar plane

then the polar

through the point d.

Radical plane,

15.

The

is

called

point d passes through the point

plane of e will pass

-'--

points
with respect to the sphere.
it is

-~CZ>

Z) is the point

Also

normal to the

is

s P. 166]

/.

of a

201

radical

plane of any

two given spheres

is the

plane which

contains all such points the squares of the tangents from which to the

given spheres are equal.

Let the two spheres he


r2
r

-2r.

Cl

......... (1)
......... (2)

+*i=0

-2r.c 2 +/;2

=0

Let there be a point p such that squares of the tanit to (1) and (2) are
equal.

gents from

or

p -2p c 1 +^ 1 =p -2p c 8 +Aj-0


2p . (Cf^) ss &! &,.
Above shows that the point p lies on the plane
.'.

2r.(c 1 -c 2)
Clearly the above plane
the line joining the centres,

is

-A 2

......... (3)

perpendicular to c t

c2

i.e.

Rule. The radical plane of two given spheres is


obtained by subtracting the equations of the spheres as can
be seen from (1), (2) and (3).

202

Vector Analysis

Ex.

The

3.

of a point

locus

which moves

so

that

its

from two fixed points are in a constant ratio n : 1 is a


Prove also that all such spheres , for different values of n,

distances
sphere.

have a common radical plane

Let the middle point

A and B
of B is

of the line joining the fixed points

be origin so that position vector of

Let the point Pbe

a.

is

a and

r.

-*

,4P=r-a and

/.

4P2

/.

we

Also

(r-

:.

or,

and

P=r+a.
/>

=(r+a) 2

are given that

4J-2

a)

(r

or

AP*=n*.BP*.

-2r a+0 2 )n2 (r 2 +2r


.

(l-H )-2r

or

Above equation

clearly represents a sphere.


w x and 2 we g e t two spheres.
n
values
the
Giving
t

The

radical

subtracting

-2r .a

plane of the

them and

above sphere

2
}+.;j.o
l-"2 5

which

is

independent of

Above

n^

the

radical

and

w2

obtained by

is

the middle

plane

distance between the given

or

r.a^O

the radical

represents

through origin which


normal is along a.

Thus

is

is

bisects

points.

plane

which

passes

AB

and the

point of

perpendicularly the

The

203

Plane and Sphere

Exercises

Ex.

distances from a given point

of whose

squares

sphere whose centre

Ex,

is at the

Prove that

2.

of a given

centre

of a point the sum of

Prove that the locus

1.

the

constant is a

centroid of the given points.

distances

the

of two points from the

are proportional

sphere

is

to

the

distances of the

points each from the polar plane of the other.

For the sake

of convenience let us choose

of the sphere as r

points as a t and a 2

the equation
centre being at origin and the given
whose polar planes are

=a
,

Ex.

sphere.

From any

3.

are

drawn

sum of

a 2 =fl 2 etc.

on the surface of a sphere, straight

point

extremities

to

Prove that

2
aj=a and

lines

of any diameter of a concentric


on these lines

the squares

is constant.

(Agra 38)
Let the centre

of

the concentric

spheres be the point c so

equations of
spheres are
r2

that the

and

outer

the

inner

-2r.c+*!^0

r*_o r

(1)

C +A; 2 =0

(2)

where fc^c2 -^ 2 and fr 2 =c2 -0 2 2 .(2)


Let rj be any point on the outer
.

Fig.No.99

sphere so that
r1 2 -2r1

If

.c+fci=0
g and h be extremities of diameter

sphere, then

its

equation
r2

or.

Comparing

(2)

is (r

g)

(r

(4),

we

(4)

get

g+h2c and g h=*


.

Now

the inner

h)=0

-r.(g+h)+g.h=0

and

(3)

of

(6)

204

Vector Analysis

4c-2*i+2(t1 -2r 1

or

= 4c 2

2fc2

.c)

+ 2*!

[from ()]

=twice the sum

squares of the

the

of

radii and hence constant.

Ex.

4.

straight line

In

fixed sphere in P.

Prove that

fixed ratio.

rj,

is

the locus

of

is

taken so that

is

OP

a sphere.

/Ex.

5.

is

0(1

(Agra 35)

Q,

i.e. r a is

etc. etc.

through a fixed point (a, b, c) and


Prove by vectors that the locus of the

plane passes

axcsinA,B,C.

centre oj the sphere

OABC

is

+ + r2-

[See Author's Solid Geometry Q. 4

The

fixed point

plane be ns=w 1 i+

The

A=ai+b]i-ck and

j+

or

let

equation of the plane through

(fl 1

(a) P. 165]

the normal to the

3 k.

If

to meet
:

T!

sphere and hence the locus of

cuts the

drawn from a point

a point Q,

be the position vector of Q, on OP, so that P will


where n is constant and this point Plies on the

Let
be

OP

i+flftj+flsk)

x 1 be the intercept

is

given by

n=a n

s flfti+6"2+M3
on the axis

:ss
<7

of *,

say.

then

x^i lies

on

the plane.
(7

.%

point

The

point

is

is

i.

Similarly

origin.

is

-j

and

is

ni
k.

The

205

Plane and Sphere

Let the coordinates of the centre be P(x,y, z\

i.e.,

)+zk.

OP=APs=BP=CP=radms

.\

Equating

their modules,

we

+/+

n is
a2

.23=2*.

-~
*

Ex.

6.

Q
x*=-^-

or

<,

<i

to the sphere

// any tangent plane


a, b, c

etc.

V (^i+K+^ )= ^

+T
+T
y

makes intercepts

*.

,-,

Similarly,
"

q
-~

OABC

get

xor

of the sphere

on the axes, prove by vectors that

[See Q. 5 P. 182 of Author's Solid Geometry],

The
hence

its

centre of the sphere is origin


2
2
vector equation is r =d

and radius

plane r . n=q will be a tangent plane


cular from centre is equal to radius

Any

U,

^|
The

is

d and

intercepts

made by

q
.

.n

or

rf

q
""i*,
=0,

.'

.n

111.
2i

^2

\-\

q%

if

perpendi-

g=nd.

the plane on the axes are


q

-A
*^v

k n

"

Vector Analysis

206

= -yi

cos 2 a+fl2 cos 2 /3+n 1 ccs 1 y]

2
[fl

=~ V
a,

/?,

cos 2

a+cos 2 0+cos2

y-

being the angles which the normal makes with the

axes.

Ex.

The

7.

perpendicular

Ex.

ABCD

r;

six

is

edges of a tetrahedron

then prove that

is the centroid

The sum of the squares on

of the tetrahedron.

the line joining

the

centre

to

of the tetrahedron is 12r


The sum of the squares on the pairs of opposite edges

the vertices

(Hi)

16r*

of two spheres

intersection

the

of

points

a sphere of radius

Centre of the sphere

(ii)

is

The mid.

8.

lies on

(i)

through the

plane

to the line joining their centres.

and

these

that

edges are

perpendicular .

Let the centre of the sphere


be taken as origin and the position vector of the vertices be a,
b, c

and d respectively.

The mid.
(writen

in

points of the edges

groups

of

opposite

edges) are
'

2
i. e.

P and

Since these
origin

and radius

'

Q,

i. e.

points

lie

r,

c+d\2

(1)

b+c

c+d

a+b

'

+d
2

a
'

+c

"2

b+d
'

and S i. e. L and
on the sphere whose centre

is

we have

/b+c\

We have to prove

/a+d\ 2 /a+c\2

that centre of the sphere

is

the

The Plane and Sphere

we have

centroid or

have taken the centre

Now

----prove that

to

207

--,=0

as

we

at the origin.

or

+ b)2 =(c+d) 2 4r 2
(a+b + c+d) (a + b-c-d)=:0

or

(a+b+c+d). iQP-0.

(a

Similarly,

->
(a

+ b+c+d)

Above

%SR=*Q and

relations

show

->

to Q, P,

SR and

ML which

then

should be a zero vector.

(a+b+c+d)

a+b+c+d

that

is

JAfZ,=0.
perpendicular

are the joins of the middle points


of pairs of opposite edges and are non-coplanar vectors. Now
if a vector is
perpendicular to three non-coplanar vectors,
it

(2)

Now

have

prove that

to

OA 2 +OB 2 +OC*+OD*
a

or

+b +c +d

Adding the relations in

(1),

=a2r

we

12r 2
2
.

get

T_

6r

3Za +227a

or

2 /
(a-l-b\

b=*24r 2

. .

.(2)

Nowa+b+c+d=0.
Squaring,

we

get

or

^a 2 +227a b=0
^a 2 =227a b.
.

/.

3ra -27a =24f

(3)

The sum

of the squares

+CD

4B

Now

from

or

=12r 2 from

(2).

on the opposite edges is


2
2
=(b-a) +(d-c) =2:a -2 (a . b+c . d)

= 12r

or

-2(a.b+c.d)

(1),

aa +2

(a .

b+ c d)8r.
.

(3)

Vector Analysis

208
2

/.

(a

Again a

b+c

b+c

d)8r 2 -Za 2 =8r 2 - 12r2 =- -4r 2

d=

(a-c)-d

2r 2 s=b

c+a d
.

or

(a~c)=0

(a-c)

Ex.

Prove

9.

to intersect

a sphere

(b-d)=0

Hence proved.

CA.DB.

i. *.

drawn from a point


cut harmonically by the surface and the polar

that any
is

straight line

plane of 0.

(Agra 53,60)
2
2r . c+/c=0 and
sphere be r
be taken as origin. Therefore any line through

Let the equation


the point

to the

A
is

it

r=/ b where

from

0.

stands for the distance of any point on


intersection with

points of

Its

the sphere are

given by
*

If

the

it

A
-2b

A
.

meets the sphere

two values

of

c t+k*=Q',
in

P and

b2
Q,,

l.

13 P. 195]

OP and OQ

then

are

given by above

A
l

JL

Again polar plane

JL

2b
_'lft_
""
""

w.r.t. the

of

sphere

-? ......

is r
[

A
Again
t

now

if

r =*fb

c=sk.

14 P. 200]

A
cuts this plane in

-R,

then b

c=A; where

stands for OR.

A
2b.c

OR=T
*

m
u;

j_
OP

OR

OQ,

are

op+da

[from

nxl (1)]

The Plane and Sphere


or

OP, OR,

Hence

the

OQ

line is cut

209

are in H.P.

harmonically by the surface and

the polar plane of 0.

Volume

16.

Let the

position

the three

of

tors

of a tetrahedron.
vec-

cotermi-

OB and OC
tetrahedron OABC be

nous edges OA,


of the

b,

a,

respectively

with

respect to origin.

Now we
volume
?f

A from

that

a tetrahedron

of

area of base

of

know

is

OBCx height

the plane of base.

Area

of

&OBC %bxc (Ex.

6,

P. 128) which represents

a vector perpendicular to the plane of


/.

volume

of

tetrahedron=

&OBC.

(ibxc)

5, P. 130 that the volume


Again we know from
parallelepiped whose three coterminous edges are a,

of a
b, c

is [abc].
.'.

volume

of

tetrahedron=

volume

of paiallelo-

piped.
Cor.
of

1.

Volume of tetrahedron

the four vertices, neither oj

We

which

in

have seen that when one

origin, the

volume

is

at the

of the four points A, B,

C and

of the tetrahedron

Let the position vectors

terms oj position vectors

is at origin.

of the vertices
is

210

Vector Analysis

O be

a, b,

c and d with respect to any origin 0'.

(Agra 39, 51)

Similarly OJ3*=:b-d and

volume

/.

OC=c-d.

of tetrahedron

-i.[a-d b-d, c-d]


f

-J.(a-d).[(b-d)X(c-d)].

V dxd=0

I/^ffta-dMbXc-dxc-bXd}],

=i[a (bxc)-a.(dxc)-a (bxd)-d (bxc)].


product is zero when two vectors are
.

Scalar triple

(Cor. 2, P. 137)

equal.

=
V
Rule

{[abc]-[abd]+[acd]-[bcd]}.

-a

. (d x c)=a
The above form is

(c

X d)[acd].

quite convenient to rememd


and
strike
one letter from the
a, b, c,
end and then the next and soon. Then form the scalar
'

ber

i.e.

write

left and connect


and
ive
signs,
alternately +ive
Cor. 2. Condition for any four points to be coplanar.
In case the four points are coplanar, then the volume
of the tetrahedron should be zero or otherwise a- d, b-d,
c d are coplanar, i.e. [a d, b d, c d]=0 which when

triple

product of the three vectors thus

them with

expanded reduces to
[abc]

Cor. 3.

- [abd]+[acd] - [bcd]-0.

Volume

of

tetrahedron

in

terms

coordinates of the vertices*

Let
the

four

(x r ,yr ,

r),

where r=l,

vertices of

the

of the

(Agra 48)
2, 3,

4 be the coordinates of

tetrahedron so that the position

vectors of the four points in terms of unit vectors


a *ii+Jii+tk etc,,
are

i, j

and

The Plane and Sphere

211

so that

Now

the

volume

of the tetrahedron

is

(a-d, b-d, c-d]

F=~

or

The
make

it

[Cot. 4 P. 1381

value of this determinant remains unchanged

by adding one row of


as shown below

of fourth order

one column of

1, 0, 0,
1

^4

*4

*1~*4

J'l-J'i

*l-*4

-X 4

^-^4

^-^4

^3~^4

^3^4

^3-^4

*4

Ar

Adding the elements


rows,

we

1, #4,^4,

of

1st

row

to 2nd,

if

we
and

3rd and 4th

get

JL

Xi

*2

*3

Exercise

Ex.

1.

Prove that the volume

four planes r
r

(mj+k}=0,

of a tetrahedron bounded

(flk+/i)

0, r

by the

(li+m])=0 and

[Q, 7 P. 80 of Author's Solid Geometry]

(Agra

39, 45, 59;

Benaras 54; Lucknow 52)

Vector Analysis

212

Let us

taken three

at

planes are

and
find

of the

cartesian equations

ws+/#=0,

(ii)

lx+my=*Q

(iii)

lx+ny+nz**p.

(iv)

point of intersection of (i), (ii)


so that the position vector

the

(i)

and

we

(ii),

and

Hence

or

y^pjm

lx=sp

or

#=/>//

points of

Similarly
(iv) arc

[from

my*=*p

--

let

us

(iv).

Let

it

this

point

is

and
of

get

p+nzQ

or

Now

intersect at origin.

three

first

(x, y, z)

Adding

(iii),

The

time.

m^+w^O,

(i)

Clearly the

be

points of intersection of the four planes

find the

^n

[from

[from

z= -p\n.

.'.

(iv)];

(i)l
(ii)].

k=c (say).

intersection of

(i),

(iii),

and

(iv)

(ii)

(say,

and

Now

volume

the

at the origin

is

of a tetrahedron

whose one vertex

| [abc]

-Pll

pirn

pin

p* 1-1

pll

-pirn

pin

-1

pll

pirn

-p]n

tetrahedron

mutual

Pwtf

2.

in

^ following

terms

inclinations.

of lengths

formula for
of

-1

Ex.

is

the volume

three concurrent edges

Vofa

and

their

[Refer Author's Solid Geometry P. 75]

The

Plane and Sphere


1

F2 =

213

COS

COS

36
COS

<$>

COS

Jl

cos

COS

(Agta

57,

Luck. 55)

Let the three concurent edges concur at origin


the position vectors of the other vertices A, B,

Again a

b
c

b^a^+Ji^+^a^ cos

c=J!7# 2 *8=i

a sasZxiXjssca cos 0.

cos

0,

/I

a2

ab cos
ac cos

ab cos

<f>

6*

fa cos 6

ac cos
be cos

c*

be

and

Vectot Analysis

214
a

abc

b cos

<f>

cos

"36
b

a cos ^

cos

b cos B

cos

COS

a*b*c*

COS

<

3(3

cos

3.

Gj,

GJJ,

G 8 0r0

OBC, OCA, OAB of a


constructed uith

Ex.4.

/fo

centroids of (he

OG lf OG 2

Prove that

triangular faces

OABC.

Prove that the volume


is to the volume of the parallelepiped
and OG 3 as coterminous edges as 9 : 4.

tetrahedron

OABC

of the tetrahedron

cos

cos

Ex.

cos e

<

each

of the four faces

of a terahedron

subtends the same volume at the centroid.

Let

the centroid of

a,

b, c

and d be taken as

origin.

......... (1)

a+b+c+d=0

/.

Volume of tetrahedron GABC, G being origin=i-[abc].


Volume of tetrahedron GBCD= J [bed]
.

(cxd)=b {cx(-a-b-c)} from


-ib (cxa)
.

(1)

-*[abc].
Ex.

5.

given by the formula


distance between

From
that
is

AB

the

is

shortest

and OC.

the figure
parallel to

we observe

b-a

to

shortest distance

and

OC

is

paral-

(b-a)xc.

Also a
is

AB

volume

parallel to c.
/.

lel

is

OABC prove that the


AB OC p sin 6 where p is

In tetrahedron

is

a point on

OA and

a point on OC.
:.

shortest distance

A/o.98
is

projec-

The Plane and Sphere


tion of

215

a-c on (b-a)Xc.

If p

11 P. 177]

be the shortest distance, then

(bxc-axc)
AB OC sin

(a-c).

a .(bxc)

~AB OC
.

'

sin

when two

scalar triple product vanishes

vectors are

equal.

"
/.

Ex.
vertex

joining

is

F= J

Show

AB.OC

[abc]fclB

sinO
.

OC

/>

sin

0.

volume of a pyramid of which the


a given point (x,y,z) and the base a triangle formed by
6.

the

coordinates

points

that

the

(a, 0, 0), (0, b, 0)

and

(0, 0, c) in rectangular

is

(Agra 47)
that a triangular pyramid is a tetrahedron.
In terms of the unit vectors the given points are ai, b}, ck
and xi+y]+zk, say vectors A, B, C and D.

We

know

V=l\A-D
C-- D

-x

--\abc

BD

C-D\

*i-j1+(*-)k;

b-y

I-*/*

-z

-j/ft

l-y\b

[Cor.

IP.

188]

216

Vector Analysis

Adding column nos.

1, 2,

abc (1 - */a -y\b -

=f

afo;

Ex.

7.

O'A'B'C

is

are

all

3,

we

get

zjc)

(!-*/ -.?/*-

G w

the

centroid

of

the

tetraltedron

another tetrahedron such that 00', AA',

bisected at

tetrahedron

and

G;

O'A'B'C'.

show that

is

also

the

OABC

BB' and CC'

centroid

of the

AGRA UNIVERSITY SOLVED PAPERS


195*
Find the vector equation of a straight line passing
through two given points. Prove by vectorial methods that
1.

the following are coricurent


the bisectors of the angles of a triangle
(a)
:

medians of a

and

(b)

the

triangle.

Cor. 2 P. 48. (b) Ex. I P. 52, Ex* 2 P. 53.


Find the vector equation of a sphere.
(a)

(a)
2.

Show

that any diameter of a sphere


right angle at a point on the surface.
(b)

(i)

Prove

(ii)

vertices

of

that if a point is
a right-angled triangle,

point of the hypetenuse

is

subtends a

equidistant from the


join to the middle

its

perpendicular to the plane of the

triangle,

(a)

12 P. 191.

(b)

See general

(i)

method

P. 194. (ii)Q,.

Obtain the equation of a


two non-intersecting lines.

3. (a)

cular to

Prove that the locus

(b)

lines

straight

straight lines

terminated
is

a plane

straight

two

by

bisecting

line

the middle

of

fixed

their

n (b) P.

HI.

perpend'-

points of all

non-intersecting

common

perpendi-

cular at right-angles.
ii P. 177.
(a)

1953
1. (a)

Define 'Centroid'.

Show

that

the centroid

is

independent of the origin of vectors.


(b)

Prove that the

lines joining

the vertices of a tetra-

hedron to the centroids of area of the opposite faces are


concurrent.

218

Vector Analysis
2 P. 37,
4 P. 40.
6. P. 59.

(a)

Ex.

(b)
2. (a)

In a tetrahedron,

if

two

pairs of opposite edges


third pair are also per-

are perpendicular, prove that the


pendicular to each other, and the

sum

the squares on

of

two opposite edges is the same for each pair.


Prove that any straight line drawn from a point
(b)
a sphere is cut harmonically by the surface
intersect
to
and the polar plane of 0.
Ex. i P. 104, (b) Ex. 19 P. 208.
Establish the following relations

2. (a)
3.
(i)
(ii)

(iii)

(i)

(ii)

(iii)

ax(bXc)+bX(cXa)+cX(aXb)=0.
a
faXb bxc cxa]=[abc]
.

(bxc) . (aXd)+(cxa)
See bottom P. 142.
Cor. P. 146.

(bxd)+(aXb)

(cxd)0.

Q,. ii P. 154.

1954
1.
(a)

Prove the following by vector methods

The

internal

bisectors of the angles of a triangle

are concurrent.
(b)

ABCD

intersection of

is

parallelogram and

the diagonals.

Show

the

that

point

for

any

of

origin

plane of the figure) the sum of the


vertices
the
is equal to four times that
of
vectors
position

(not necessarily

in the

of 0.
(a)
2.
(a)

Ex. 2 P. 53,

(b)

Just as

Q,. 15 (b) P. 30.

Give vectorial solutions of the following

The

area of the triangle formed by joining the

middle point of one of the non-parallel sides of a trapezium


to the

extremities

trapezium.

of

the

opposite side

is

half that of the

219

Agra University Solved Papers


Find the coordinates

(b)

of

the centre of the sphere

the tetrahedron bounded by the planes


i0, r . j =0, r . k=0 and r . (i+j+k)=a.

inscribed in

Ex. 12 P. 132,

(a)

(b)

Ex.

P. 198.

Q,. 10 P. 154, (b) Ex. 2 P. 147.

1.

(a)

1955
Prove the following by vector methods
The medians of a triangle are concurrent.
:

The

four diagonals of a parallelepiped, and the


joins of the mid. points of opposite edges, are concurrent at
a common point of bisection.
(b)

(c)

o'a

- 2b

The

three points

whose position vectors are

a, b,

are collinear.

P, 52, (b) Ex. 7 P. 61.

(a)

Ex.

(c)

Q.26,(b)P.

2. (a)

77-

Find by vector method, the equation of the

line

two planes.
that the line of intersection of

of intersection of
(b)

Show
r

is

(i+2j+3k)=0 and r

equally inclined to

with

(3i+2j+k)=0

and k, and makes an angle J

sec"" 1

j.

(a)
3. (a)

8 P. 171, (b) Ex. 5 P. 182.


Prove the relation

ax{bx(cXd)}=b

daXc-b caxd,

and hence expand

ax[bx{cx(dxe)}].

veciur analysis

szu

Find the equation

(b)

of the straight line

through the

ooint c, intersecting both the lines

r-a=A and

(a) ft. 10 P. 154, (b)

1. (a)

r-a'=fb'.

Ex. 14 P. 184.

1956
Prove the following by vector methods

The internal bisector of the angle A of a triangle


divides the side BC in the ratio AB AC.

(i)

4BC

The

(ii)

ingle

is

mid. points of

of the

join

parallel to the third side,

What

(b)

vector

the

is

and

two

sides of a

tri-

of half its length.

of

equation

the straight line

hrough the points i-2j+k and 8k- 2j ?


Find where the line cuts the plane through the origin
ind the points 4j

and 2i+k.
(ii) Ex. 19 P. 72.

(a) (i) Ex. 3 P. 57,


(b) Q..2 P. go.

Find the equations of the planes bisecting the


ingles between the two given planes
r . n=*q and r n'==<?'.
2. (a)

Find the coordinates of the centre of the sphere


bounded by the planes

(b)

ascribed in the tetrahedron

i=0, r j=0, r .
6 P. 169, (b) Ex.
.

(a)
3. (a)

(i+j+k)=a,

P. 198.

Prove that

l.b

l.c

m .b

m .c

n a

n .b

n .c

l.a

[lmn]][abc]=

(b)

k0 and r

Prove the formula

[axb, cxd, cxf]=[abd] [cef]-[abc] [def]


[abe] [fcd]-[abf ] [ecd]

(a)

Ex. 2 P. 148,

~[cda][bef]-[cdb][aefj.
(b) Ex. 6 P. 153.

221

Agra University Solved Papers


*957
Prove the following by vector methods

1.

The

(a)

internal

bisectors of the angles of a triangle

area of

the

are concurrent.

The

(b)

formed by joining the

triangle

point of one of the non-parallel sides of a trapezium


the extremities of the opposite side is half that of the

mid,
to

trapezium.
(a)

Ex. 2 P. 53

in

terms

;
(b) Q,. 12 P. 132.
that any given vector r can be expressed
three given non-coplanar vectors a, ft y in the

Show

2. (a)

of

form

Prove that [axb, bxc, cXa]=[abc]2 and express

(b)

the result by means of determinants.


(a)
ft.

of

(a)

two

8 P. 143-1449 (b) Cor. P. 146.


Explain the terms scalar and vector products

vectors, giving illustrations.

Prove the following formula for the volume V of a


tetrahedron, in terms of the lengths of three concurrent
(b)

edges and their mutual inclinations


1
cos

<fr

cos

30

cos

<

cosJ*
(a)

(a)

2 P. 99

and

cos 6

cos

3 P. 118, (b) Ex. 2 P. 212.

Define 'centroid'.

Show

that the

centroid

is

independent of the origin of vectors.


(b)

Two

forces

act at the corner A of a

quadrilateral

222

Vector Analysis

ABCD,

represented by
-*

sented by

AB and AD

CB and CD.

Show

and two

that

at

repre-

resultant

their

is

represented by

4PQ where
,

P, Q, a * e

t' ie

mid. points of AC,

BD respectively.
aP.

(a)

4 P-

4-

5 (a) P. 30.

ft'

(*>)

37,

If any
within a tetrahedron ABCD is
pDint
joined to the vertices, and AO, BO, CO, DO are produced
to cut the opposite faces in P,
R, S respectively, then
show that
2. (a)

z
Z

b and 3a-2b are

(a)

Ex.

8. (a)

two

J/--

Prove that the three points whose position vectors

(b)

are a,

OP

colliuear.

P. 89; (b) Q,. 26 (b) P. 77.


Define (1) the scalar, ('2) the vector product of
i

vectors,

and give instances

of

their

application to

mechanics.

Find the straight line, through the point Cj which


parallel to the plane r . a=0, and intersects the line
(b)

is

r-a'^b.
2 P. 98

1.

(a)

and

3 P. 119

and see Q. 14

(b) P. 185*

Prove bo vector method the following


If a line be drawn parallel to the base of a

triangle,

the line which joins the opposite vertex to the intersection


of the diagonals of the trapezoid thus formed bisects the
base.

The

i- j-f 3k and B (i+j- k) are equidistant


from the plane r . (si+2j-7k)=0, and are on oppposite
(b)

sides of

(a)

points

it.

Ex. 22 P. 72. (b)

5 P. iC6

and Ex.

3. (b)

P. 168.

223

Agra University Solved Papers


Prove that

2.

(BxC)x A=A

BC-A

CB,
and that the volume of the tetrahedron bounded by the
.

four planes

r
r

(mj-fnk)=sO, r

(/i+mj)=0 and r

is

(li+wj+nk)=p

Zpsfilmn.

6 P. 140, Ex. i P. 211.


Prove that the shortest distance between the
two opposite edges of a regular tetrahedron is equal to
half the diagonal of the square described on an edge.
3. (a)

Establish the vector formula

(b)

[axpbxq cXr]+[aXq bxr cXp]


+[aXr bXp cxq]=0.
(a)

Ex. 22 (d) P. 188.

(b)

Ex. 7 P. 153.

1960
Find the vector equation to a sphere. Prove also
to intersect
that any straight line drawn from a point
a sphere is cut harmonically by the surface and the polar
1. (a)

plane of 0.
(b)

point

Find the equation of the straight

d and

line

through the

equally inclined to the vectors a, b, c in the

form

+T+T)
,,

and

---

*(cxa)+*(axbn
- ------

^
J.
Ex.
P.
12 P. 191,
208, (b) Ex. 4 P. 124.
19
(a)
What do you understand by a system of reciproB. (a)
cal vectors ? Show that any given vector r can be expressed
in terms of three given non-coplanar vectors a, /?, y in the
form
[rffy] a+[rya] /?4-[rafl y

r^d+5^

(b)

Prove that

the result
(a)

by means

[aXb bXc cXa]=[abc]2 and


of determinants.

9 P. 144* (b) Cor. P. 146.

express

224

Vector Analysis

Prove the formula

3. (a)

(bxc).(axd)+(cxa),(bxd)+(axb).(cxd)=0
and use

it

to

show

sin

that

(A+B)

sin

M-B)=sina

^4-sin 2 5.

Prove that the shortest distances between a diago(b)


nal of a rectangular parallelepiped whose sides are a, b, c
and the edges not meeting it, are
be

ab

ca

Ex. iz P. 154, (b) Ex. 7 P. 180.

(a)

1961

Prove by vector methods the following


If any
within a tetrahedron ABCD is
(a)
point
joined to the vertices, and AO, BO, CO, DO are produced to
cut the opposite faces in P, Q,, R, S respectively, then prove
1.

(b) If

through any point within a triangle, lines be drawn

sides, show that the


these lines to their corresponding sides

to the

parallel

Ex.

2. (a)

ratios of

(i+2j+Pk)*0 and r . (8i+2j+k)-0

equally inclined

with

of the

P. 89, Ex. 22 (b), P, 73.


Show that the line of intersection of

r
is

sum
is 2.

to i

k and makes

and

an angle i sec^1 3

j.

Find by vector method, the equation to the

(b)

intersection of
(a)

planes.
8 P. 171.
(b)

Ex. 5 P. 182,
Find the equation of the straight

3. (a)

the point

c,

line

intersecting both the lines

r-a=5b
(b)

and r

a'=*/b'.

Prove the formulae

[axb, cxd, exf][abe] [fcd]-[abf] [ecd]


[cda] [bef J-[cdb] [aef ].
(a)

line of

two given

Ex. 14 P. 184,

(b) Ex. 6 P. 153.

through

RAJPUTANA UNIVERSITY SOLVED PAPERS


1959
Define scalar triple product and prove the follow-

1.

ing

(a)

cyclic

change the value

permutation of
the

of

three

scalar triple product but

permutation changes the value

cyclic

vectors does not

in

sign

an

anti-

but not in

magnitude.

The

(b)

position of

changed without changing

a dot and cross can be


its

inter-

value.

4 and 5 page 134-136.


If p, q,

2.

r are three vectors defined by the relation


^

~(ibcj'

where

a,

b,

c are vectors and the scalar

(pqr)l and
q and r.

(abc)9^0, prove that (abc)


a,

b and
9

in

terms of p,

and Property

triple product
obtain the values of

page 144-145.
1960

1 00%

of the questions were set from this book.

Show

that every vector can be represented as a


linear combination of two non-collinear vectors coplanar with
1. (a)

the original vector.

Show by

method

vectors that the stiayht


line joining the middle points of two sides of a triangle is
parallel to the third side and is of half its length.
(b)

(a)

the

of

6 page 14, (b) Ex. 19 page 72.

PUNJAB UNIVERSITY SOLVED PAPERS


1960

100% of the questions were set from this book.


1.

The

(a)

and

necessary

points with position vectors


there exist four scalars x,y,

Show

sufficient condition for four

to be coplanar

a, b, c, d,
,

/,

not

all

is

that

zero, such that

the internal bisector of any angle of a


the
opposite side internally in the ratio of
triangle
the sides containing the angle.
(b)

that

divides

Show that the external bisectors of the three plane


(c)
angles of each trihedron of a given tetrahedron are coplanar.
10 page 86, (b) Ex. 3 page 57.

(a)

Define the

2. (a)

vector

product of two vectors and

prove that

a*\
(b)

-A /-> -\- /-> -VJ=\a c ) b-\a .b )

/-

bX.c

c.

Find the condition that

and interpret it geometrically.


Prove that
(c)

a,b
(&)

by

3 P.

(a), (c)

118,

+ b.c

a=*2c

b,

6 P. 140, (b)

Cor. P. 146.

Expand both

sides

Punjab University Solved Paperk

Show

3. (a)

227

that

-/- -\ -- -*a\ b+c J=a.b+a.c.


-

Find the distance of the point,

(b)

nq
(c)

measured

Show

a,

from the plane

rb+t c.

parallel to the line

that the middle point of the hypotenuse of a

right-angled triangle

is

equidistant from its vertices.

9 P. xoo, (b) Cor. P. 168, (c) Ex. 10 P. 109.

(a)
4. (a)

Show

and cross may

product, the dot


be interchanged without changing the result.

If a, b>

(b)

that in the scalar

c,

triple

d be four vectors, express d

combination of the three non-coplanar vectors

Hence prove

as

linear

a, b, c.

that

-->-->

P-P' P-9' P**'

where

q.p'

q.q'

jf

f
.

r'

r.r'

are any vectors.


6 (iii) P. 16, (c) Ex. 2 P. 148.
5 P. 135, 136, (b)
Show that the equation of the plane through two
5. (a)
/>,

r'
q' r'; p' q'

given points A,
-

given vector

c is

t(.

with position vectors

a,

and

parallel to

Vector Analysis

228
(b)

intersect
(c)

Show

and

find the point of intersection.

The

relative to

that the lines

position vectors

any

origin

four points

of

are denoted

by

0, b,

c,

d.

-4,

5, C,

Interpret

geometrically the equations

((i)

(ii)

(a)
'(c)

~*\

(-> ~*\

->

\a-bJx\cxdJ~Q,
\a-b

Result IV page 164,

Ex. 12 page 184.

(b)

Ex* 10 page 183,

S-ar putea să vă placă și